Patho/Pharm Final Exam Questions

Pataasin ang iyong marka sa homework at exams ngayon gamit ang Quizwiz!

The nurse recognizes that what system is being used when seeing a medication ordered in minims? A) Apothecary B) Metric C) Household D) Avoirdupois

A Feedback: The apothecary system used minims as the basic measure of liquid and is rarely used today. The metric system uses liters as the basic unit of measurement for fluid, while the household system uses ounces and the avoirdupois system uses ounces with a different conversion amount.

An adult patient with renal cancer, weighing 95 kg, is to receive vincristine 25 mcg/kg/day IV. What is the dosage of vincristine that the nurse should administer to the patient daily in mg? __________

Ans: 2.375 mg Feedback: This order requires 25 mcg of medication for every 1 kg of body weight. The patient weighs 95 kg. To determine total dosage multiply weight times mcg of medication: 25 × 95 = 2,375 mcg. Convert mcg to mg by moving the decimal three places to the left, or you can divide 2,375 by 1,000 because there are 1,000 mcg per mg.

The nurse is calculating the patient's intake and output record and converts ounces to milliliters. What systems is this nurse converting from and to? A) From household to metric B) From metric to household C) From household to apothecary D) From apothecary to Avoirdupois

Ans: A Feedback: Ounces can be either household or Avoirdupois, although they are not equal measurements and milliliters is a metric measurement. Since Avoirdupois to metric is not an available choice, the correct answer is household to metric.

A patient is going to have bowel surgery in the morning. The physician orders 500 mL of GoLytely PO to be administered at 5 PM this evening. How many liters will the nurse administer? A) 1 B) ¾ C) ½ D) ¼

Ans: C Feedback: Cross-multiply to learn 500 mL = ½ or 0.5 L.

The nurse administers a loop diuretic to the patient. In addition to sodium and water, what other electrolyte would the nurse expect to be excreted in significant amounts? A) Calcium B) Magnesium C) Potassium D) Zinc

Ans: C Feedback: Loop diuretics increase excretion of sodium, water, and potassium most significantly. Although other electrolytes may be excreted, loss of magnesium, calcium, and zinc are usually not significant.

A patient presents at the clinic complaining of vaginal itching and a clear discharge. The patient reports to the nurse that she has been taking an oral antibiotic for 10 days. The nurse is aware that the patient is experiencing what? A) An adverse reaction from the antibiotic B) A drug toxicity effect of the antibiotic C) An overdose of the drug that is damaging to more than one body system D) A superinfection caused by the antibiotic, which has destroyed normal flora

Ans: D Feedback: Superinfections often occur with antibiotic use because the drug kills normal bacterial flora. This is not a result of toxic levels of the antibiotic, but rather an effect of the medication that has killed normal flora, which it is designed to do. Vaginal itching and a clear discharge are not considered adverse effects of an antibiotic. An overdose of a drug that damages more than one body systems is considered drug poisoning.

New parents were just told by their physician that their son is two standard deviations above the mean. The parents later asked the nurse what that means. The nurse will explain by stating, A) "If your child is one standard deviation from the norm that translates to mean, he will be taller than 50% of his peers." B) "This is great news since it means you will have a larger baby than most." C) "Being two standard deviations above the mean translates into that your child will likely be taller than 95% of children in the population." D) "With the mean being average at 50%, two standard deviations means that your child will be at least 99.7% taller than his brother."

C

The nurse is preparing medication for a 30-month-old child with otitis media in the right ear. The child weighs 33 pounds. The physician has ordered Keflex, 50 mg/kg/d in equally divided doses every 8 hours. The medication concentration is 250 mg/5 mL. How many milliliters should the nurse give the toddler for each dose?

Ans: 5 mL Feedback: To calculate the correct dosage, the nurse first converts the child's weight from pounds to kilograms by dividing weight in pounds by 2.2 (2.2 lb = 1 kg). 33 pounds ÷ 2.2 pounds/kg = 15 kg. The child is to receive 50 mg for every kilogram. To determine this child's dosage multiply weight times daily dose (15 kg × 50 mg/kg = 750 mg). Thus, 750 mg is to be administered in equally divided dosages every 8 hours, or 3 times a day. 750 mg ÷ 3 = 250 mg/dose. There is 250 mg in 5 mL of medication so the patient would be given 5 mL.

The nurse receives a new medication order for a patient to administer 240 mg of medication per day in equally divided doses every 6 hours. How many mg of the drug should the nurse administer for each dose?

Ans: 60 mg Feedback: Because there are 24 hours in a day, giving a drug every 6 hours would mean giving the drug 4 times a day. Because the total daily dose is 240 mg, dividing that dose by 4 would mean each dose should be 60 mg.

The provider orders a maintenance dose of oral aminophylline, 3 mg/kg every 6 hour. The patient weighs 50 kg. How many mg should the nurse administer to the patient in a 24-hour period?

Ans: 600 milligram Feedback: The patient's weight times the number of milligram/kilogram will provide daily dosage of medication: 50 kg × 3 mg/kg = 150 mg per dose. The patient is to receive a dose every 6 hours. The number of hours in a day divided by the number of hours separating each dose supplies the number of dosages the patient receives per day: 24 hours ÷ 6 hours between doses = 4 doses per day. If each dose is 150 mg and the patient receives 4 of these doses a day, the total amount of medication received is 150 mg × 4 daily doses = 600 mg.

The mother of a 7-year-old boy who has recently been diagnosed with childhood asthma has come to the education center to learn more about her son's condition. Which of the following teaching points is most justifiable? A) "Research has shown that viruses may actually be a factor in many children's asthma." B) "The most reliable indicator that your child is having an asthma attack is audible wheezing." C) "Steroids that your child can inhale will likely be the first line of defense." D) "Your son will likely need to limit or avoid exercise and sports."

Ans: A Feedback: Viruses have been implicated as a contributing factor in childhood asthma. Wheezing may or may not be present in children, and inhaled corticosteroids are not common as an initial therapy. Current treatment guidelines do not advise the categorical avoidance of exercise.

When explaining how carbon dioxide combines with water to form carbonic acid as part of acid-base lecture, the faculty instructor emphasized that which enzyme is needed as a catalyst for this reaction? A) Carbonic anhydrase B) Phenylalanie hdroxylase C) Hydrolases D) Trypsin

Ans: A Feedback: Although CO2 is a gas and not an acid, a small percentage of the gas combines with water to form H2CO3. The reaction that generates H2CO3 from CO2 and water is catalyzed by an enzyme called carbonic anhydrase.

The first-time parents of an infant girl 2 days postpartum are distressed at the jaundiced appearance of her skin and are eager for both an explanation and treatment for the problem. Which of the following responses by their physician is most accurate? A) "Your daughter's young liver is unable to get rid of the waste products from old red blood cells." B) "Because your daughter's kidneys are so small, they have a hard time getting rid of the wastes that are always accumulating in her blood." C) "Nearly half of all infants have this problem, and while it is distressing to look at, it is largely harmless and will resolve in time." D) "This is a sign that your baby needs more milk than she is currently getting, and increased breast-feeding will act to flush these pigments out of her system."

Ans: A Feedback: Bilirubin is formed from the breakdown of hemoglobin in red blood cells. Normally about two thirds of the unconjugated bilirubin produced by a term newborn can be effectively cleared by the liver. However, the relative immaturity of the newborn liver and the shortened life span of the fetal red blood cells may predispose the term newborn to hyperbilirubinemia. Bilirubin clearance is not the domain of the kidneys, and treatment is often necessary. Jaundice can sometimes be addressed by increasing breast-feeding, but it is not a sign in and of itself of insufficient feeding.

A premature infant on mechanical ventilation has developed bronchopulmonary dysplasia (BPD) and is showing signs and symptoms of hypoxemia, low lung compliance, and respiratory distress. Which of the following is the most likely contributor to the infant's present health problem? A) High-inspired oxygen concentration and injury from positive-pressure ventilation B) Failure to administer corticosteroids to the infant in utero C) Insufficient surfactant production and insufficient surfactant therapy D) Insufficient supplemental oxygen therapy

Ans: A Feedback: Despite the administration of corticosteroids in utero to hasten alveolar maturation, premature infants suffering respiratory distress syndrome often must be treated with supplemental oxygen and mechanical ventilation. However, overly forceful positive- pressure ventilation (barotrauma) can lead to the chronic lung impairment of BPD. Surfactant therapy is a first-line defense against the development of RDS and is also used to treat cases of BPD; additional time on a ventilator is often required as well.

Which of the following patients who presented to a walk-in medical clinic is most likely to be diagnosed with a rhinosinusitis rather than a common cold? A) A man complaining of general fatigue, a headache, and facial pain with a temperature of 100.9°F B) A woman presenting with malaise, lethargy, and copious nasal secretions C) A man with a dry, stuffy nasopharynx, a sore throat, and temperature of 98.9°F D) A woman complaining of generalized aches and who has a hoarse voice and reddened, painful upper airways

Ans: A Feedback: Fever and facial pain are more commonly associated with rhinosinusitis rather than the common cold. The other noted symptoms are indicative of the common cold rather than rhinosinusitis.

A 52-year-old patient has just passed a kidney stone and has high levels of calcium in her urine. Blood tests show high levels of calcium in her blood as well. What subsequent lab results would be most likely to distinguish between primary hyperparathyroidism and hypercalcemia of malignancy? A) Parathyroid hormone level B) Bone scan C) Plasma phosphate levels D) Serum magnesium level

Ans: A Feedback: Hyperparathyroidism, in which parathyroid hormone is secreted in excess, may be caused by a parathyroid adenoma. Since parathyroid hormone mobilizes calcium from bone and promotes its transfer to the extracellular fluid, excess calcium is excreted in the urine (promoting the development of kidney stones) and is evident in the plasma. In primary hyperparathyroidism, antibody binding assays of intact PTH would reveal either normal or elevated parathyroid hormone in the face of hypercalcemia, whereas in hypercalcemia of malignancy, levels of intact PTH are suppressed.

When trying to explain the role of potassium and hydrogen related to acid-base balance, which of the following statements is accurate? A) Hypokalemia stimulates H+ secretion. B) Hyperkalemia will cause the reabsorption of HCO3. C) Acidosis causes an increase in K+ elimination. D) Alkalosis tends to increase H+ elimination.

Ans: A Feedback: Hypokalemia is a potent stimulus for H+ secretion and HCO3 reabsorption. Acidosis tends to increase H+ elimination and decrease K+ elimination, with a resultant increase in plasma potassium levels, whereas alkalosis tends to decrease H+ elimination and increase K+ elimination, with a resultant decrease in plasma K+ levels.

A midwife who is providing care for a woman during her first pregnancy is assessing for intrauterine growth retardation (IUGR) during an early prenatal checkup. Which of the following questions best addresses the risks for IUGR? A) "What does your typical diet look like over the course of a day?" B) "What is the highest level of education that you've finished?" C) "Are there many people in your life that you can count on for help and support?" D) "How would you describe your mood since you've been pregnant?"

Ans: A Feedback: Nutrition is a key aspect in the prevention of IUGR. Educational level, the presence or absence of support systems, and psychosocial health may all have ramifications that could affect fetal development, but they have a less direct bearing than maternal nutrition.

A nurse is providing care for a patient who has been admitted with a newly diagnosed bilateral pleural effusion. Which of the following findings from the nurse's initial assessment of the patient is incongruent with the patient's diagnosis and would require further investigation? A) The client complains of sharp pain exacerbated by deep inspiration. B) The client's breath sounds are diminished on auscultation. C) Pulse oximetry indicates that the client is hypoxemic. D) The client complains of dyspnea and increased work of breathing.

Ans: A Feedback: Pleural effusion is not normally associated with pain, and intense pain that is worsened by deep breathing would necessitate further investigation. Diminished breath sounds, hypoxemia, and dyspnea are common findings associated with pleural effusion.

A 55-year-old woman has presented to the emergency department following a panic attack. Her blood pressure, respiratory rate, and heart rate are all highly elevated, while her temperature and oxygen saturation are within normal ranges. What is the woman's body most likely doing to address the changes in pH associated with her situation? A) Her kidneys will limit the amount of bicarbonate that they reabsorb. B) She will be retaining Cl- ions in an effort to lower pH. C) Her respiratory center will attempt to lower her CO2 levels. D) The patient's kidneys will excrete more hydrogen ions than they normally do.

Ans: A Feedback: Renal compensation for respiratory alkalosis involves decreased bicarbonate reabsorption. Manipulation of Cl- ions is not a compensatory mechanism that the body is capable of, and increased CO2 levels and decreased H+ would compensate for her acid-base imbalance.

A patient with small cell lung cancer (SCLC) has developed a paraneoplastic syndrome called Cushing syndrome. Based on this new complication, the nurse will likely assess which of the following clinical manifestations of Cushing syndrome? A) Weight gain, moon face, buffalo hump, and purple striae on the abdomen B) Bilateral edema in the arms, swollen face, and protruding eyes C) Severe bone/joint pain, nausea/vomiting, and polyuria D) Tetany, new-onset seizure activity, emotional lability, and extrapyramidal symptoms

Ans: A Feedback: SCLS is associated with several types of paraneoplastic syndromes, including Cushing's. Answer choice B refers to superior vena cava syndrome; answer choice C refers to hypercalcemia; and answer choice D refers to tumor lysis syndrome. All of these are complications that can occur with cancer and treatment of cancer.

A family physician is performing patient teaching about the influenza virus with each patient who has come to the clinic to receive that year's vaccine. Which of the following statements by the patient best reflects an accurate understanding of the flu virus? A) "I could come down with viral or bacterial pneumonia as a result of a bad flu bug." B) "I know my vaccination is especially important since there aren't any drugs that can treat the flu once I get sick with it." C) "The emphasis on bundling up, staying warm, and drinking lots of fluids is outdated and actually ineffective." D) "Like all vaccines, it is ideal if everyone in a population gets immunized against the flu."

Ans: A Feedback: Viral and bacterial pneumonia are known sequelae of influenza. Antiviral drugs do exist for the flu, and the efficacy of staying warm and increasing fluid consumption have been demonstrated. The flu vaccine is recommended for higher risk individuals, and guidelines do not indicate the need for all individuals to be vaccinated.

The nursing instructor is teaching the class about how prescription drugs become over-the-counter (OTC) drugs and lists what factor as preventing a drug from becoming classified as OTC? A) If the patient cannot reliably self-diagnose the condition the drug is intended to treat B) If it would mask signs and symptoms of an underlying problem, the drug remains available by prescription only. C) If the drug would cause toxic effects if not taken as directed, it remains a prescription drug. D) OTC drugs must not have any adverse effects that could harm the patient.

Ans: A Feedback: If a diagnosis requires medical intervention, such as hyperlipidemia, which can only be diagnosed through laboratory studies, there is no point in making the drug an OTC medication. Most, if not all, OTC drugs have the capacity to mask signs and symptoms of an underlying disease so this is not a factor in deciding if a drug can be sold OTC. All drugs have the potential for toxic effects if not taken as directed and virtually all drugs have the potential for adverse effects.

The nurse administers a specific medication to an older adult patient every 4 hours. The patient has a history of chronic renal failure. Why would this patient be at risk for toxic drug levels? A) Cumulative effect B) First-pass effect C) Drug interactions D) Cross-tolerance effect

Ans: A Feedback: If a drug is taken in successive doses at intervals that are shorter than recommended, or if the body is unable to eliminate a drug properly, the drug can accumulate in the body, leading to toxic levels and adverse effects. This is a cumulative effect. First-pass effect addresses the reduction of available drug when taken orally due to metabolism in the liver before the drug reaches the bloodstream. Drug interactions occur when taken with other drugs, food, or complementary alternative therapies. Cross tolerance is resistance to drugs within the same class.

A nurse in a medical unit has noted that a client's potassium level is elevated at 6.1 mEq/L. The nurse has notified the physician, removed the banana from the client's lunch tray, and is performing a focused assessment. When questioned by the client for the rationale for these actions, which of the following explanations is most appropriate? A) "Your potassium level is high, and so I need you let me know if you feel numbness, tingling, or weakness." B) "Your potassium levels in the blood are higher than they should be, which brings a risk of changes in the brain function." C) "I'll need to monitor you today for signs of high potassium; tell me if you feel as if your heart is beating quickly or irregularly." D) "The amount of potassium in your blood is too high, but this can be resolved by changing the intravenous fluid you are receiving."

Ans: A Feedback: Paresthesia and muscle weakness are manifestations of hyperkalemia. Tachycardia and dysrhythmias are more commonly associated with hypokalemia, and the greatest risks associated with potassium imbalances are cardiac rather than neurological. Hyperkalemia is not normally resolved by correction using IV fluid.

The nurse, assisting with Phase I drug studies, is talking with a woman who asks, Why can't I participate in this study? What would be the nurse's best response? A) Drugs pose a greater risk to women of reproductive age. B) Drugs are only tested on men because they are stronger. C) Women are more prone to adverse effects from medications. D) Drugs affect women differently than they affect women.

Ans: A Feedback: Phase I drug trials usually involve healthy male volunteers because chemicals may exert an unknown and harmful effect on ova in women which could result in fetal damage when the woman becomes pregnant. Drugs are tested on both men and women, but women must be fully informed of risks and sign a consent stating they understand the potential for birth defects. Women are not more prone to adverse effects of medications. Although some drugs may affect women differently than men, this is a rationale for why drugs need to be tested on women, not an explanation of why women are not included in a phase I study.

In response to the patient's question about how to know whether drugs are safe, the nurse explains that all medications undergo rigorous scientific testing controlled by what organization? A) Food and Drug Administration (FDA) B) Drug Enforcement Agency (DEA) C) Centers for Disease Control and Prevention (CDC) D) Joint Commission on Accreditation of Healthcare Organizations (JCAHO)

Ans: A Feedback: The FDA is responsible for controlling and regulating the development and sale of drugs in the United States, allowing new drugs to enter the market only after being subjected to rigorous scientific testing. The DEA regulates and controls the use of controlled substances. The CDC monitors and responds to infectious diseases. The JCAHO is an accrediting body that inspects acute care facilities to ensure minimum standards are met.

What is the most accurate method for the nurse to use when determining a pediatric dosage? A) A nomogram using body surface area B) Young's rule C) Fried's rule D) Clark's rule

Ans: A Feedback: The most accurate means of determining a pediatric drug dosage is a nomogram using body surface area because the weight and body surface area of two children who are of the same age can be significantly different. Young's rule, Fried's rule, and Clark's rule are based on the child's age and the usual adult dosage and are rarely used today unless no other method will suffice for a specific drug.

The nurse receives an order to administer an unfamiliar medication and obtains a nurse's drug guide published four years earlier. What is the nurse's most prudent action? A) Find a more recent reference source. B) Use the guide if the drug is listed. C) Ask another nurse for drug information. D) Verify the information in the guide with the pharmacist. .

Ans: A Feedback: The nurse is responsible for all medications administered and must find a recent reference source to ensure the information learned about the medication is correct and current. Using an older drug guide could be dangerous because it would not contain the most up-to-date information. Asking another nurse or the pharmacist does not guarantee accurate information will be obtained and could harm the patient if the information is wrong

A nurse is caring for a patient taking multiple drugs and is concerned about a possible drugdrug interaction. What is the nurse's first and best means of avoiding this problem? A) Consult a drug guide. B) Call the pharmacist. C) Contact the provider. D) Ask another nurse.

Ans: A Feedback: Whenever two or more drugs are being given together, first consult a drug guide for a listing of clinically significant drug drug interactions. Sometimes problems can be avoided by staggering the administration of the drugs or adjusting their dosages. Consulting the pharmacist is not wrong, but it would not be the first action to take. The nurse holds responsibility for his or her own practice so asking a health care provider or another nurse is based on the assumption that that professional is knowledgeable about all drug drug interactions, which is likely not the case.

The student nurse asks the instructor why a patient with a central nervous system infection is receiving antibiotics that will not cross the blood brain barrier. What is the instructor's most correct response? A) A severe infection alters the blood brain barrier to allow the drug to cross. B) A medication that is water soluble is more likely to cross the blood-brain barrier. C) Antibiotics are the exception to the blood brain barrier and cross easily. D) An infection that spreads outside the central nervous system helps drugs cross the barrier.

Ans: A Feedback: Effective antibiotic treatment can occur only when the infection is severe enough to alter the blood brain barrier and allow antibiotics to cross. Lipid-soluble, not water-soluble, medications cross the blood brain barrier more easily and most antibiotics are lipid soluble, so they are not the exception. No matter where the infection originates, drugs must cross the bloodbrain barrier to treat central nervous system infections.

A male patient with a history of heavy alcohol use has been admitted to hospital for malnutrition and suspected pancreatitis. The patient's diagnostic workup suggests alcoholic ketoacidosis as a component of his current health problems. He is somewhat familiar with the effect that drinking has had on his nutrition and pancreas but is wholly unfamiliar with the significance of acid-base balance. How best could his care provider explain the concept to him? A) "The chemical processes that take place throughout your body are thrown off very easily when your body is too acidic or not acidic enough. B) "The multitude of chemical reactions that take place in your body depend on your body fluids being slightly acidic." C) "The healthy function of your kidneys and your lungs requires a specific level of pH in your body." D) "Your body is highly dependent on what food and fluid you consume to keep itself at a functioning level of slight nonacidity."

Ans: A Feedback: Metabolic activity is highly contingent on a narrow range of pH. Normal pH is slightly basic, not acidic, and appropriate pH is maintained by the lungs and kidneys, not vice versa. The action of the respiratory and renal systems, not particular food or fluid intake, has the most salient effect on the acid-base balance.

The nurse uses what term to describe the drug level required to have a therapeutic effect? A) Critical concentration B) Dynamic equilibrium C) Selective toxicity D) Active transport

Ans: A Feedback: A critical concentration of a drug must be present before a reaction occurs within the cells to bring about the desired therapeutic effect. A dynamic equilibrium is obtained from absorption of a drug from the site of drug entry, distribution to the active site, metabolism in the liver, and excretion from the body to have a critical concentration. Selective toxicity is the ability of a drug to attach only to those systems found in foreign cells. Active transport is the process that uses energy to actively move a molecule across a cell membrane and is often involved in drug excretion in the kidney

The nurse advises that patient to avoid long-term use of nasal decongestants because it may lead to what condition? A) Mucosal ulcerations B) Decreased drainage C) Increased risk of infection D) Asthma

Ans: A Feedback: Adverse effects associated with topical decongestants include local stinging and burning, which may occur the first few times the drug is used. If the sensation does not resolve, the drug should be discontinued, because it may indicate lesions or erosion of the mucous membranes. Nasal decongestants do not cause asthma or increased risk of infection. These medications do not decrease drainage from the nose as they shrink the nasal mucosa.

After administering a medication, for what would the nurse assess the patient? A) Drug effects B) Allergies C) Pregnancy D) Preexisting conditions

Ans: A Feedback: After the medication is administered, the nurse assesses the patient for drug affects, both therapeutic and adverse. The nurse would assess the patient for allergies, preexisting conditions, and pregnancy before administering a medication

The nursing instructor is discussing drug therapy in the older adult. What would the instructor tell the students about what could affect therapeutic dosing in an older adult? A) Changes in the gastrointestinal (GI) system can reduce drug absorption. B) In older adults, drugs enter into circulation more quickly. C) In older adults, drugs are distributed to a smaller portion of the tissues. D) In older adults, drugs have an increased action.

Ans: A Feedback: As patients age, the body undergoes many normal changes that can affect drug therapy, such as a decreased blood volume, decreased GI absorption, reduced blood flow to muscles or skin, and changes in receptor site responsiveness. They are not released more quickly into circulation; distributed to a smaller portion of tissue; nor do they have an increased action.

The physician orders acetaminophen (Tylenol) 15 mg/kg q4 hours PRN (as needed) for pain. The drug is supplied with 160 mg/5 mL. The infant weighs 12 pounds. How many milliliter of medication will the nurse administer? A) 2.5 mL B) 5.6 mL C) 12.4 mL D) 10.7 mL

Ans: A Feedback: Begin by converting the infant's weight to kilograms (12 pounds ÷ 2.2 pounds/kg = 5.45 kg). The order says to give 15 mg/kg. To determine this child's dosage multiply weight and 15 mg/kg (5.45 kg × 15 mg/kg = 81.75 mg/dose). Finally, use the ratio and proportion method to calculate how to prepare the medication: 160 mg/5 mL = 81.75 mg/X mL; cross-multiplying yields 2.5 mL.

The nurse is caring for a patient who needs education on his medication therapy for allergic rhinitis. The patient is to take clemastine (Tavist) daily. In providing educational interventions regarding this medication, what is the most important instruction on the action of the medication? A) It blocks the effects of histamine. B) It is used to treat atrial and ventricular dysrhythmias. C) It competitively inhibits the rate-limiting enzyme in the liver. D) It leads to bronchodilation and relaxes smooth muscle in the bronchi.

Ans: A Feedback: Clemastine blocks the effects of histamine at the histamine-1 receptor sites, decreasing the allergic response. It is prescribed to treat allergic rhinitis. Antiarrhythmic medications are used to treat atrial and ventricular dysrhythmias. Beta-hydroxy-beta-methylglutaryl coenzyme A reductase inhibitors are used to lower blood cholesterol by competitively inhibiting the rate-limiting enzyme in the liver. Beta- adrenergic agents lead to bronchodilation and stimulate beta2 -adrenergic receptors in the smooth muscle of the bronchi and bronchioles.

A patient arrives in the ED very hypovolemic related to excretion of "at least 3 gallon jugs of urine in the past 24 hours." He describes the urine as being clear-like water. The physician suspects diabetes insipidus. The nurse should be prepared to administer which of the following medications? A) Desmopressin acetate (DDAVP) B) Benadryl, an anticholinergic C) Calcium gluconate D) Prednisone

Ans: A Feedback: Diabetes insipidus is caused by a deficiency of or a decreased response to ADH. The preferred drug for treating chronic DI is desmopressin acetate (DDAVP).

The nurse is giving discharge instructions to the mother of a 3-month-old infant who has an upper respiratory tract infection and has been prescribed a pseudoephedrine nasal solution. What instructions are most important for the nurse to give to this mother? A) Instill the medication 20 to 30 minutes before feeding. B) Keep the baby on clear liquids until the nasal discharge has resolved. C) Start the baby on cereal, because she is having difficulty sucking right now. D) Give the medication immediately after feeding.

Ans: A Feedback: Oral decongestants are drugs that are taken by mouth to decrease nasal congestion related to the common cold, sinusitis, and allergic rhinitis. They are also used to relieve the pain and congestion of otitis media. Opening of the nasal passage allows better drainage of the Eustachian tube, relieving pressure in the middle ear. It should be given prior to a feeding so that infant is able to suck more effectively.

Parents who treat their children's cold and flu symptoms at home should be educated concerning the reading and understanding of over-the-counter (OTC) labels. Why is this statement true? A) Many of these preparations contain the same active ingredients so that inadvertent overdose is a common problem. B) Each product is best used for alleviating a particular symptom. C) Some of these products do not contain any drugs. D) Some of these products could interfere with breast-feeding.

Ans: A Feedback: Parents need to be educated to read the labels of any OTC preparation they give their children. Many of these preparations contain the same ingredients and inadvertent overdose is a common problem. Giving a child a drug should not interfere with the mother's milk production. All these products contain chemicals, which are drugs; many can be used to treat more than one symptom.

The family nurse practitioner is caring for a Hispanic woman who is 83 years old. The patient has been noncompliant with the care regimen the nurse practitioner has previously outlined for treatment of sinusitis. What should the nurse practitioner do that can assist the patient in being more compliant with the prescribed treatment regimen? A) Provide instructions in writing. B) Provide the instructions in large type. C) Give the treatment instructions to a member of her family. D) Give the treatment instructions to the woman's husband.

Ans: A Feedback: The nurse instructs the patient about signs and symptoms that require follow-up and provides these instructions verbally and in writing. Instructions in alternate formats (e.g., large type, patient's language) may be needed to increase the patient's understanding and adherence to the treatment plan. Option B is incorrect because the situation does not indicate that the woman cannot read regular-sized type print. Options C and D are incorrect because the situation does not indicate that any family members accompanied the patient to see the family nurse practitioner.

The patient is prescribed a medication that was just placed in Phase IV study. The patient tells the nurse, This medication is too expensive. Could the doctor order a generic form of this medication? What is the nurse's most accurate response? A) Medications are not produced in generic form until the patent expires, which normally takes several years. B) You can request the generic form but the binder used may make the drug less effective for this medication. C) The generic form of the medication would not be any less expensive because this is a relatively new medication. D) Generic medications are lower quality drugs and that would mean you would not be getting the best treatment available

Ans: A Feedback: When a new drug enters the market, it is given a time-limited patent; generic forms of the medication cannot be produced until the patent expires. Because no generic version of this drug will exist because it is so new, it is impossible to predict what binder will be used or what the cost would be.

A patient asks the nurse, What is a Drug Enforcement Agency (DEA) number? What is the nurse's best response? A) DEA Numbers are given to physicians and pharmacists when they register with the DEA to prescribe and dispense controlled substances. B) Physicians must have a DEA number in order to prescribe any type of medication for patients. C) DEA numbers are case numbers given when someone breaks the law involving a controlled substance. D) DEA numbers are contact numbers to talk with someone at the DEA when questions arise about controlled substances.

Ans: A Feedback: All pharmacists and physicians must register with the DEA. They are given numbers that are required before they can dispense or prescribe controlled substances. DEA numbers are only needed when prescribing controlled substances. A DEA number is neither a case number nor a phone number.

The nurse administers an anticholinergic medication to the patient. When assessing this patient, what finding will the nurse consider a secondary effect of the drug? A) Nasal congestion B) Tachycardia C) Hyperthermia D) Profuse sweating

Ans: A Feedback: Anticholinergic secondary effects include dry mouth, altered taste perception, dysphagia, heartburn, constipation, bloating, paralytic ileus, urinary hesitancy and retention, impotence, blurred vision, cycloplegia, photophobia, headache, mental confusion, nasal congestion, palpitations, decreased sweating, and dry skin. Tachycardia, hyperthermia, and profuse sweating would not be expected findings or consistent with anticholinergic effects and would require further assessment.

A 68-year-old patient who must take antihistamines for severe allergies is planning a vacation to Mexico. The nurse will encourage the patient to do what? A) Avoid sightseeing during the hottest part of the day. B) Discontinue the antihistamines if he becomes extremely restless. C) Decrease the dosage of the drugs if he experiences excessive thirst. D) Continue taking the antihistamines even if he begins to hallucinate.

Ans: A Feedback: Antihistamines can cause anticholinergic effects, which would result in decreased sweating and place the patient at high risk for heat stroke. Avoiding the hottest part of the day will help prevent dehydration and heat prostration. Extreme restlessness could indicate Parkinson-like syndrome not usually associated with antihistamines. Excessive thirst is characteristic of hyperglycemia. Hallucinations are associated with drugs that affect neurologic functioning. Further, nurses should never tell patients to decrease or discontinue a drug unless the prescriber has instructed them to do so

A student nurse is doing research consisting of chart audits of 25 patients with diagnosed chronic pharyngitis; she is attempting to determine what medication has been prescribed most frequently. What type of medication would this nurse expect to find in most of the chart audits? A) Antitussives B) Nasal sprays C) Oral decongestants D) Mucolytics

Ans: A Feedback: Antitussives are drugs that suppress the cough reflex. Many disorders involving the respiratory tract, including the common cold, sinusitis, pharyngitis, and pneumonia are accompanied by an uncomfortable, nonproductive cough. Persistent coughing can be exhausting and can cause muscle strain and further irritation of the respiratory tract. Nasal sprays, oral decongestants, and mucolytics are not generally prescribed for chronic pharyngitis, however.

A nurse is caring for a 77-year-old patient. The nurse plans care for this patient based on the knowledge that the aging process impacts drug therapy in what important way? A) Blood volume decreases B) Subcutaneous tissue increases C) Total body water increases D) Muscle mass increases

Ans: A Feedback: As patients age, the body undergoes many normal changes that can affect drug therapy, such as a decreased blood volume, decreased gastrointestinal (GI) absorption, reduced blood flow to muscles or skin, and changes in receptor site responsiveness. As a person ages, subcutaneous tissue decreases, total body water decreases, and muscle mass decreases.

The nurse is assessing a patient new to the clinic. The patient says she is allergic to penicillin. What would be the nurse's appropriate next action? A) Ascertain the exact nature of the patient's response to the drug. B) Document the patient is allergic to penicillin. C) Mark the patient's chart in red that she has a penicillin allergy. D) Continue to assess the patient for other allergies.

Ans: A Feedback: Ask additional questions of patients who state that they have a drug allergy to ascertain the exact nature of the response and whether it is a true drug allergy. Patients may confuse secondary actions of the drug with an allergy. Only after it was determined the action was truly an allergy would the nurse document the allergy, mark the patient's chart, and continue to assess for other allergies.

The patient is a 34-year-old man who recently started taking theophylline. The nurse knows that medication teaching has been successful when he agrees to what activity? A) Avoiding caffeine B) Eating foods high in potassium C) Limiting fluid intake to 1,000 mL a day D) Taking the medicine on an empty stomach

Ans: A Feedback: Both theophylline and caffeine are xanthenes. Theophylline increases cardiac output and heart rate. Caffeine also stimulates heart rate. This can have an additive effect. Eating foods high in potassium, limiting fluid intake, or taking the medicine on an empty stomach are not indications that the patient has understood the nurse's teaching.

Student nurses are learning to weigh patients and do vital signs. How does a correct weight impact administering medication? A) Proper dosage calculation B) Assessing changes in fluid balance C) Assessing changes in nutritional status D) Caloric needs

Ans: A Feedback: Dosage of medication is often calculated based on the patient's weight, so getting patients' weight wrong could cause a medication error. The patient's weight gives information into fluid balance, nutritional status, and caloric needs but this is not associated with drug therapy. However, a patient's weight is most important in determining the appropriateness of drug dosage.

The nursing instructor is discussing the off-label use of drugs. What group of drugs would the instructor tell the nursing students is often used for off-label indications? A) Drugs used to treat psychiatric problems B) Drugs used to treat gastrointestinal (GI) problems C) Drugs used to treat cardiovascular problems D) Drugs used to treat musculoskeletal problems

Ans: A Feedback: Drugs often used for off-label indications include the drugs used to treat various psychiatric problems. Drugs used to treat GI, cardiovascular, or musculoskeletal problems do not fall in the category of frequent off-label uses.

What cardiac effect would the nurse be prepared to see in the patient with an extremely high potassium level? A) Arrhythmia B) Tachycardia C) Sudden death D) Bradycardia

Ans: A Feedback: Elevated potassium levels irritate cardiac cells and increase the likelihood of a cardiac arrhythmia. Tachycardia, sudden death, and bradycardia would be the result of an arrhythmia if they were to occur.

The nurse provides teaching to the patient using herbal therapies and includes what important information related to the effects of the herbal therapy? A) They can interact with prescription drugs. B) They always contain known ingredients. C) They are natural so they are effective and safe. D) The ingredients are natural, meaning toxicity is not a concern.

Ans: A Feedback: Herbal therapies can produce unexpected adverse effects and toxic reactions, can interact with prescription drugs, and can contain various unknown ingredients that alter the therapies' effectiveness and toxicity.

A physician orders 500 mL of IV solution be administered over 8 hours. If the IV infusion set delivers 15 drops per mL, how many drops per minute should the nurse administer to the patient? A) 15 drops/min B) 20 drops/min C) 32 drops/min D) 64 drops/min

Ans: A Feedback: If a patient was to receive 500 mL in 8 hours, dividing 500 by 8 would mean that the patient would receive 62.5 mL in 1 hour, or 60 min. Setting up the equation, 15 drops/mL/X equals 62.5 mL/60 min; cross-multiplying, the answer will be 15 drops/min.

The triage nurse in the emergency department sees a patient suspected of abusing amphetamines brought in by friends. While assessing this patient, what would the nurse be likely to find if steroids are being abused? A) Hypertension B) Bradycardia C) Drowsiness D) Elated mood

Ans: A Feedback: Increases in blood pressure, tachycardia, and insomnia are symptoms of amphetamine abuse. Elation can indicate abuse of cannabis.

Ipecac, formerly used as the drug of choice by parents for treatment of suspected poisoning in children, was tested by the Food and Drug Administration (FDA) in 2003. What was the finding of this testing? A) Ipecac is ineffective for its intended use. B) Ipecac is the safest treatment for poisoning in children. C) Ipecac was grandfathered in as an ineffective drug. D) Ipecac induces vomiting.

Ans: A Feedback: Ipecac, a formerly standard over-the-counter drug, was used for many years by parents to induce vomiting in children in cases of suspected poisoning or suspected drug overdose. The drug was finally tested and in 2003, the FDA announced that it was not found to be effective for its intended use. Although it was grandfathered in as an effective drug, this was not what the study researched. Ipecac is not effective and does not consistently induce vomiting.

The patient is taking a drug that affects the body by increasing cellular activity. Where does this drug work on the cell? A) Receptor sites B) Cell membrane C) Golgi body D) Endoplasmic reticulum

Ans: A Feedback: Many drugs are thought to act at specific areas on cell membranes called receptor sites. After the receptor site is activated, this in turn activates the enzyme systems to produce certain effects, such as increased or decreased cellular activity, changes in cell membrane permeability, or alterations in cellular metabolism. Receptor sites are generally located on the outside of cells and allow the drug to bypass the cell membrane. The Golgi body and endoplasmic reticulum are not involved in this process.

When evaluating information accessed over the Internet, an important question the nurse should teach the patient to ask is what? A) Is the information anecdotal? B) Where has this information been obtained? C) Is this information paid for by the drug company? D) How many patients have had input into the information?

Ans: A Feedback: Many people do not know how to evaluate the drug-related information that they can access over the Internet. Is it accurate or anecdotal is an important concept for the nurse to teach the patient to assess to verify the accuracy of the information. Where the information came from is unimportant. It would be expected that all drug advertising is paid for by the drug company and this is not an important concern. Number of patients with input into the information is most likely none because information is gathered from health care professionals.

A patient presents at the clinic with signs and symptoms of seasonal allergic rhinitis. The patient is prescribed a nasal steroid to relieve symptoms. Two days later, the patient calls the clinic and tells the nurse that he is frustrated and wants a new drug. What is the most appropriate response by the nurse? A) It may take up to 2 weeks to get the full clinical effect. Try to keep using the drug as ordered. B) The drug must not work for you. Let's change to an oral steroid. C) You probably are administering the drug incorrectly. Come in and we can review the process. D) You probably need to try a different nasal steroid. This one should be effective by now.

Ans: A Feedback: Nasal steroids require about 2 weeks to reach their full clinical effect so the patient should be encouraged to use the drug for that length of time before changing drugs or giving up. The other responses could be appropriate if after 2 weeks the patient is still not getting relief.

The post-anesthesia care unit nurse is serving a patient after a right knee arthroscopy. As the patient begins to wake up from anesthesia, the nurse assesses rigidity, involuntary movements, hyperthermia, and tachycardia. What would the nurse suspect is causing these effects? A) Neuroleptic malignant syndrome B) Parkinson-like syndrome C) Malignant tachycardia D) Anaphylactic shock

Ans: A Feedback: Neuroleptic malignant syndrome is a generalized syndrome that includes extrapyramidal symptoms such as slowed reflexes, rigidity, involuntary movements; hyperthermia; autonomic disturbances (e.g., hypertension, fast heart rate); fever may be noted as well. This is most often seen after administering general anesthesia or drugs with central nervous system (CNS) effects. This syndrome was once known as malignant hyperthermia. These symptoms are not consistent with Parkinson-like syndrome or anaphylactic shock. Malignant tachycardia is a distracter.

While collecting a medication history, the patient admits to doubling the recommended dosage of an over-the-counter (OTC) medication, saying It's harmless or they would require a prescription. What is the nurse's best response? A) OTC drugs are serious medications and carry serious risks if not taken as directed. B) Taking medications like that is careless and you could kill yourself doing it. C) Sometimes you need to take more than the package directs to treat the symptoms. D) Did you notify your doctor of the increased dosage you were taking?

Ans: A Feedback: OTC drugs are no less a medication than prescription drugs and carry the same types of risks for overdosage and toxicity if directions are not followed. Although increasing the dosage is careless and dangerous, it is important to use the information as a teaching opportunity rather than scolding the patient. Agreeing with the patient or asking her if she talked to the doctor misses the teaching opportunity, which could be harmful for the patient

After admitting a patient to the unit, the nurse is organizing times to administer ordered medications. What important consideration will guide the nurse's timing of each medication? A) Comfort B) Ethnicity of patient C) Gender D) Age

Ans: A Feedback: Organizing the day and the drug regimen to make it the least intrusive on a patient's comfort can help to prevent errors and improve compliance. Ethnicity, gender, and age should be a consideration when ordering the drug, but after the drug is chosen it should be administered in a way that will maintain the patient's quality-of-life.

With the need to protect our environment, what is it now important for the nurse to teach patients to do? A) Dispose of drugs no longer used on an annual basis. B) Flush drugs down the toilet. C) Bury unused in the yard. D) Throw unused pill bottles in the trash in original containers.

Ans: A Feedback: Patients should go through their medicine cabinet annually and dispose of drugs no longer used. Unused drugs should not be flushed down the toilet or buried in the yard because they seep into the community water supply. Pills should be removed from their bottle and mixed with an undesirable substance to prevent someone from using the medication if found.

A nurse is providing teaching to a group of patients who are beginning drug therapy for acquired immunodeficiency syndrome (AIDS). What should be included in her instructions to the group? A) Take your medications as directed. Poisoning occurs with overdosage causing damage to more than one body system. B) Renal injury results from first-pass effect when the drug is excreted from the system. C) A blood dyscrasia due to drug therapy can be serious. Call us if your skin looks yellowish or you experience itching. D) Most drugs are metabolized in the liver and the first indication of damage is dark red papules, which should be reported immediately.

Ans: A Feedback: Poisoning resulting from overdosage can lead to the potential for fatal reactions when more than one body system is affected. Liver, not kidney, injury can be caused by the first-pass effect and can cause the skin to have a yellow appearance. Most drugs are metabolized in the liver but liver damage causes jaundice, manifested as a yellow tinge to the skin and sclera. Dark red papules appearing on limbs are characteristic of Stevens-Johnson syndrome, a potentially fatal erythema multiforme exudativum, which should be reported but is not due to liver damage

The pharmacology instructor explains to students that adverse effects can be extensions of what? A) Primary action of a drug B) Anaphylaxis C) Secondary action of a drug D) Anticholinergic responses to the drug

Ans: A Feedback: Primary action adverse effects are extensions of the therapeutic action and are usually the result of overdosage, essentially too much of the therapeutic effect. Anaphylaxis is not an extension of the therapeutic action of the drug but a histamine reaction to an allergen. Secondary actions of a drug are negative effects of the drug that occur even when the drug is in the therapeutic range. Anticholinergic responses occur in response to drugs that block the parasympathetic nervous system.

A patient has been diagnosed with a brain tumor that cannot be removed surgically. During each office visit, the nurse will be assessing the patient for syndrome of inappropriate antidiuretic hormone (SIADH). Which of the following assessments would alert the clinic nurse that the patient may be developing this complication? A) Complaints that his urine output is decreased, no edema noted in ankles, and increasing headache B) Elevated blood glucose levels, dry mucous membranes, and severe projectile vomiting C) Fever, diarrhea, and nausea D) Muscle cramps, pins and needle sensation around the mouth/lips, and unexplained bruising

Ans: A Feedback: SIADH manifests as a dilutional hyponatremia. Decrease urine output, absence of edema, and headaches are signs of this. Answer choice B relates to s/s of diabetes insipidus; answer choice C is indicative of common flu s/s; answer choice D is relates to s/s of hypocalcemia.

A patient is prescribed salmeterol with dosage on a 4 to 6 hour schedule for treatment of exerciseinduced asthma. What is the recommended dosing schedule of asthma experts regarding this drug? A) 30 minutes before exercise to prevent dyspnea during exercise B) Every 15 minutes during exercise to prevent dyspnea C) As needed to treat or prevent dyspnea during exercise D) Every 1 to 2 hours to treat or prevent dyspnea during exercise

Ans: A Feedback: Salmeterol (Serevent) adult and pediatric (12-year-old and older): two puffs every 12 hours; or two puffs 30 to 60 minutes before exercise. Therefore, options B, C, and D are incorrect.

The nurse explains the Drug Enforcement Agency's (DEA's) schedule of controlled substances to the nursing assistant who asks, Do you ever get a prescription for Schedule I medications? What is the nurse's best response? A) Schedule I medications have no medical use so they are not prescribed. B) Schedule I medications have the lowest risk for abuse and do not require a prescription. C) Schedule I medications are only prescribed in monitored units for patient safety. D) Schedule I medications are found in antitussives and antidiarrheals sold over the counter.

Ans: A Feedback: Schedule I medications have no medical use and are never prescribed. Schedule V medications have the lowest risk for abuse and are found mostly in antitussives and antidiarrheals but they are not sold over the counter.

What can make a nurse or any health care provider lose credibility with the patient? A) Being unprepared to deal with the disease of the week B) Refusing to write prescriptions for the drug the patient requests C) Not being knowledgeable about diseases described on House D) Being prepared to discuss the role of concierge doctor

Ans: A Feedback: Some health care providers have learned to deal with the disease of the week as seen on talk shows; others can be unprepared to deal with what was presented and may lose credibility with the patient.

Recognizing the prevalence and incidence of dehydration among older adults, a care aide at a long-term care facility is in the habit of encouraging residents to drink even though they may not feel thirsty at the time. Which of the following facts underlies the care aide's advice? A) Older adults often experience a decrease in the sensation of thirst, even when serum sodium levels are high. B) The metabolic needs for both fluid and sodium in older adults differ from those of younger individuals. C) Regulation and maintenance of effective circulating volume by the kidneys is less effective in the elderly. D) The renin-angiotensin-aldosterone system (RAAS) is less able to facilitate sodium clearance in older adults.

Ans: A Feedback: The elderly are prone to hypodipsia even when osmolality and serum sodium levels are elevated, a fact that is compounded by sensory and/or neurological deficits. Hypodipsia in the elderly is not related to differing metabolic needs, ineffective kidney function, or compromise of the RAAS.

The nurse determines a child's body surface area is 0.4 m2 and the average adult dosage of the medication is 500 mg. The medication is supplied in liquid form with 500mg/5 mL. How many milliliter will the nurse administer? A) 3.46 mL B) 1.73 mL C) 0.5 mL D) 12 mL

Ans: A Feedback: The formula for calculating the child's dose using body surface area is surface area in m2 divided by 1.73 and then multiplied by adult dosage. Using the information supplied in this problem (0.4 ÷ 1.73) × 500 mg = 346 mg. Using the ratio and proportion method, 500 mg/5 mL = 346 mg/X mL; cross- multiplying yields 3.46 mL for the nurse to administer.

The nursing students are learning about the half-life of drugs. A student asks the instructor to explain half-life. What is the instructor's best response? A) Half-life of a drug is the time it takes for the amount of drug in the body to decrease to half of the peak level it previously achieved. B) Half-life is the amount of time it takes for the drug to be metabolized by the body. C) Half-life is the amount of time it takes for half of the drug to reach peak level in the body. D) Half-life of a drug is the time it takes for the drug to reach half its potential peak level in the body.

Ans: A Feedback: The half-life of a drug is the time it takes for the amount of drug in the body to decrease to half the peak level it previously achieved. Therefore Options B, C, and D are not correct

The parent of a 2-year-old child is visiting his or her pediatric health care provider and shows the nurse the advertisement for allergy medication found in a magazine in the waiting room saying, This drug sounds like it would be far more effective to treat my son's asthma and I'd only have to give it once a day. What is the nurse's best response? A) Talk with your health care provider about this drug, but be aware that advertisements do not always provide all the important information you need to know. B) Oh, I need to throw that magazine away because so many people show me that ad and it is all complete nonsense with no truth to it at all. C) I've been seeing amazingly positive results from that medication so you are absolutely right to want to give it to your child. D) That drug is dangerous and should not be given to children under the age of 5 unless there are no other good options.

Ans: A Feedback: The health care provider should make the decision about what medications are to be prescribed, not the nurse. However, the nurse can make the mother aware of the fact that there is often more that goes into choosing the correct drug than the bit of information disclosed in the advertisement. Becoming upset with the mother, agreeing with the mother, or frightening the mother about the medication is the wrong approach for the nurse to use.

The nurse is calculating a drug dosage and converting from milligrams to grams. What measurement system is the nurse using? A) Metric system B) Apothecary system C) Household system D) Avoirdupois system

Ans: A Feedback: The metric system is the most widely used system of measurement in the world; it is based on the decimal system. The gram is the basic unit of solid measure, and the liter unit of liquid measure. The apothecary system uses the grain as the basic unit of solid measure. The household system uses the pound as the basic unit of measure. The avoirdupois system uses ounces and grains, but it is mostly used by drug manufacturers for bulk medications.

The nurse is preparing a medication that is new to the market and cannot be found in the nurse's drug guide. Where can the nurse get the most reliable information about this medication? A) Package insert B) Another nurse C) Drug manufacturer D) Physician

Ans: A Feedback: The most reliable information about the drug can be found on the package insert supplied by the manufacturer because it was prepared according to strict Food and Drug Administration (FDA) regulations. Asking another nurse or the physician is not reliable and cannot be verified as accurate. It would not be realistic to call the drug manufacturer for information.

When moving to another state, what is the nurse responsible for becoming familiar with? A) Local policies and procedures for controlled substance administration B) Local provider's Drug Enforcement Agency (DEA) number for prescribing controlled substances C) The agency monitoring controlled substances in the new state D) Board of Nursing regulations of controlled substances in the new state

Ans: A Feedback: The nurse needs to learn local policies and procedures for controlled substance administration because they can vary with some local governments more rigorous than others. Nurses do not memorize a provider's DEA numbers. The DEA is a federal agency that monitors controlled substances in all states. State boards of nursing do not regulate controlled substances but may regulate how controlled substances are administered by nurses.

The local news has been discussing a specific rare disorder that killed a child in the community this week, describing the symptoms of the disease as including nasal congestion, ear pain, and a cough. The pediatrician's office is receiving numerous calls asking to make appointments to rule out this rare disease. What is the nurse's best action? A) Prepare a handout that describes the disorder discussed in the news in greater detail. B) Tell parents their child is experiencing the common cold and do not need to be seen. C) Direct all calls to the local news agency to answer questions and provide details. D) Become familiar with the disorder and screen each call for more specific symptoms.

Ans: A Feedback: The nurse needs to not only become more familiar with the disorder in the news, but also needs to be prepared to teach parents about the disease of the week to allay fears so a handout with detailed information would allow the parents to have something to consult after leaving the provider's office. Turning parents away without seeing their child will increase fears and the office will lose credibility for lack of interest in their child's well-being. Directing calls to the news agencies will not provide parents with essential information. Screening calls without seeing the child could be potentially very dangerous.

The nurse is reviewing the patient's admission medication orders. What order would the nurse to question? A) Digoxin .5 mg orally now give one dose only B) Lasix 20 mg. IV every 4 hours times 3 C) Gentamicin 80 mg IV to infuse over 1 hour every 12 hours D) Acetaminophen 650 mg PO every 4 hours as needed for pain

Ans: A Feedback: The nurse should question the order for Digoxin.5 mg to make sure 0.5 mg is meant versus 5 mg, or what should have been ordered which is 0.05 mg. There should always be a 0 before a decimal point if no other number is present to make sure the decimal point is seen. The remaining orders could be administered as written.

A patient with chronic bronchial asthma is prescribed montelukast (Singulair). What will the nurse instruct the patient to avoid taking? A) Aspirin B) Penicillin C) Sertraline (Zoloft) D) Nifedipine (Procardia)

Ans: A Feedback: The nurse would instruct the patient to avoid aspirin, which might cause an increased montelukast level and toxicity. The other options do not cause drugdrug interactions with montelukast.

A 71-year-old man with a history of heart disease and diabetes has had an antihistamine prescribed. The nurse is concerned with this prescription because of the risk for what? A) Cardiac arrhythmias B) Increased salivation and choking C) Severe constipation D) Insomnia

Ans: A Feedback: The patient has history of heart disease. Antihistamines have been associated with prolongation of the QT interval, which can lead to potentially fatal cardiac arrhythmias. Antihistamines dry the mucosa and are not associated with increased salivation or choking, can cause drowsiness, and are not associated with insomnia or severe constipation.

The nurse begins administering 500 mL of 5% dextrose and water solution at 01:00 to run over 4 hours. At 02:00, the nurse administers 80 mg gentamicin in 50 cc normal saline to infuse over 30 minutes. How many mL of fluid will the nurse administer to the patient between 02:00 and 03:00? A) 175 mL B) 150 mL C) 125 mL D) 100 mL

Ans: A Feedback: The patient is receiving 500 mL over 4 hours. To determine how much fluid is infusing per hour = 500 mL ÷ 4 = 125. In addition to the 125 mL of IV solution, the patient also receives 50 mL of gentamicin during the 02:00 to 03:00 hour. 125 mL + 50 mL = 175 total mL of fluid received during this hour

A nurse is caring for a child on the pediatric unit. A drug is ordered for the child, but no pediatric dose is listed for the drug. To make sure that the right dose has been ordered, what will the nurse use to calculate the correct dose? A) Surface area B) Height C) Birth date D) Adult dosage

Ans: A Feedback: The surface area of a child is calculated using height and weight. It is the most accurate way to determine an appropriate dosage for that child. Age does not take into consideration variations in growth. Height alone does not take into account the mass of the child. Gestational age is simply a distracter.

The nurse is teaching the patient how to safely use the Internet for health information and includes what information in the teaching plan? A) The Web site where information is obtained needs to be evaluated for credibility. B) Most information found on the Internet is accurate. C) Information on the Internet is most reliable when people give their reviews of the drug. D) Only a health care professional can tell whether a Web site is reliable.

Ans: A Feedback: There are excellent sites for reliable drug information, but each site must be evaluated for credibility and the nurse can teach the patient things to look for to increase confidence in the site. However, a lot of information on the Internet is not accurate; the patient needs to learn how to recognize unreliable information when he or she comes across these sites. Just because a person reviews a drug and gives it multiple stars or a thumbs up does not mean the drug is any more effective or useful in the patient's care.

The processes involved in dynamic equilibrium are key elements in the nurse's ability to determine what? A) Dosage scheduling B) Amount of solution for mixing parenteral drugs C) Timing of other drugs the patient is taking D) How long the patient has to take the drug

Ans: A Feedback: These processes are key elements in determining the amount of drug (dose) and the frequency of dose repetition (scheduling) required to achieve the critical concentration for the desired length of time. The processes in dynamic equilibrium are not key elements in determining the amount of diluents for intramuscular (IM) drugs; they do not aid in the timing of the other drugs the patient is taking or how long the patient has to take the drug.

A 77-year-old female hospital patient has contracted Clostridium difficile during her stay and is experiencing severe diarrhea. Which of the following statements best conveys a risk that this woman faces? A) She is susceptible to isotonic fluid volume deficit. B) She is prone to isotonic fluid volume excess. C) She could develop third-spacing edema as a result of plasma protein losses. D) She is at risk of compensatory fluid volume overload secondary to gastrointestinal water and electrolyte losses.

Ans: A Feedback: This woman is at risk of isotonic fluid volume deficit and sodium imbalances as a result of her diarrhea. She is not likely to develop fluid volume excess or third spacing as consequences of diarrhea.

The nurse is to infuse 100 mL of 5% dextrose and water solution containing an IV antibiotic over 30 minutes. The infusion set delivers 10 gtt/mL. How many drops per minute will the nurse administer? A) 33 gtt/min B) 30.3 gtt/min C) 30 gtt/min D) 3 gtt/min

Ans: A Feedback: Use the following ratio to determine how many drops of fluid to administer per minute: Using the information from this problem: Because it is not possible to deliver 0.3 drops, round 33.3 to 33 gtt/min.

A patient is admitted with a deep vein thrombosis in his or her left calf. The physician orders Heparin, 7,500 units subcutaneously every 12 hours. The medication vial reads Heparin, 10,000 units/mL. How many milliliters does the nurse administer? A) 0.5 mL B) 0.75 mL C) 1 mL D) 1.25 mL

Ans: A Feedback: Using the information supplied by the problem: 10,000 units/1 mL = 7,500/X. Cross-multiplying yields X = 0.75 mL for each dose.

Why are inhaled steroids used to treat asthma and chronic obstructive pulmonary disease (COPD)? A) They act locally to decrease release of inflammatory mediators. B) They act locally to improve mobilization of edema. C) They act locally to increase histamine release. D) They act locally to decrease histamine release.

Ans: A Feedback: When administered into the lungs by inhalation, steroids decrease the effectiveness of the inflammatory cells. This has two effects, which are decreased swelling associated with inflammation and promotion of beta-adrenergic receptor activity, which may promote smooth muscle relaxation and inhibit bronchoconstriction. Options B, C, and D are incorrect.

The nurse is preparing to administer a medication from a multidose bottle. The label is torn and soiled but the name of the medication is still readable. What is the nurse's priority action? A) Discard the entire bottle and contents and obtain a new bottle. B) Find the drug information and create a new label for the bottle. C) Ask another nurse to verify the contents of the bottle. D) Administer the medication if the name of the drug can be clearly read.

Ans: A Feedback: When the drug label is soiled obscuring some information the safest action by the nurse is to discard the bottle and contents because drug labels contain a great deal of important information, far more than just the name of the drug. Concentration of the drug, expiration date, administration directions, and precautions may be missing from the label and so put the patient at risk. Looking up drug information in a drug handbook or consulting with another nurse will not supply the expiration date or concentration of medication. Be safe and discard the bottle and its contents.

A 42-year-old male patient is brought to the emergency department by ambulance. The patient is in distress. The nurse suspects an anaphylactic reaction resulting from taking oral penicillin. What assessment findings are important in making this diagnosis? A) Blood pressure (BP): 186/100, difficulty breathing B) Hematocrit (Hct): 32%, decreased urine output C) Temperature: 102º, swollen joints D) Profuse sweating, Blood Pressure: 92/58

Ans: A Feedback: An anaphylactic reaction is an immune reaction that causes a massive release of histamine, which results in edema and swelling that can lead to respiratory distress and increased blood pressure. A decreased hematocrit and decreased urine output suggests a cytotoxic reaction. An increased temperature and swollen joints could suggest serum sickness. Profuse sweating and decreased blood pressure may indicate cardiac-related issues.

What concept is considered when generic drugs are substituted for brand name drugs? A) Bioavailability B) Critical concentration C) Distribution D) Half-life .

Ans: A Feedback: Bioavailability is the portion of a dose of a drug that reaches the systemic circulation and is available to act on body cells. Binders used in a generic drug may not be the same as those used in the brand name drug. Therefore, the way the body breaks down and uses the drug may differ, which may eliminate a generic drug substitution. Critical concentration is the amount of a drug that is needed to cause a therapeutic effect and should not differ between generic and brand name medications. Distribution is the phase of pharmacokinetics, which involves the movement of a drug to the body's tissues and is the same in generic and brand name drugs. A drug's half-life is the time it takes for the amount of drug to decrease to half the peak level, which should not change when substituting a generic medication

The patient, diagnosed with cancer, is receiving morphine sulfate (a potent narcotic pain reliever) to relieve cancer pain. Approximately every 7 days the medication is no longer effective in controlling the patient's pain and a larger dose is needed to have the same effect. How might the nurse explain why this is happening? A) Tolerance B) Cumulation C) Interactions D) Addiction

Ans: A Feedback: The body may develop a tolerance to some drugs over time. Tolerance may arise because of increased biotransformation of the drug, increased resistance to its effects, or other pharmacokinetic factors. When tolerance occurs, the amount of the drug no longer causes the same reaction. Therefore, increasingly larger doses are needed to achieve a therapeutic effect. Cumulative effect occurs when the drug is not properly eliminated and more of the drug is administered, resulting in toxic levels accumulating. Interactions occur when the drug reacts badly with another substance such as food, another drug, or an alternative or complementary therapy. Addiction is the psychological need for a substance.

The nurse administers a medication to the patient that induces the secondary action of hypoglycemia. What organ will be most acutely impacted by inadequate circulating glucose? A) Brain B) Heart C) Lungs D) Skin

Ans: A Feedback: While all cells require glucose to function, the brain uses the greatest amount. As a result, hypoglycemia has the greatest impact on the brain, which explains why hypoglycemia has so many neurological signs and symptoms including fatigue, drowsiness, anxiety, headache, shaking, lack of coordination, numbness and tingling of the mouth, tongue, and /or lips; confusion, and in severe cases, seizures or coma may occur because the brain cannot function without adequate supplies of glucose.

A COPD patient asks the nurse what medications are prescribed to help his breathing. The nurse, looking at the list of medications, will educate the patient about which of the following medications to help his COPD in the long term? Select all that apply. A) Salmeterol (Serevent), a bronchodilator B) Tiotropium (Spiriva), anticholinergic C) Alprazolam (Xanax), a benzodiazepine D) Sildenafil (Viagra), a vasodilator E) Ketorolac (Toradol), an NSAID

Ans: A, B Feedback: Pharmacologic treatment of COPD includes the use of bronchodilators (Serevent) and anticholinergic drugs (Tiotropium). Benzodiazepines are used for anxiety, and sildenafil is a vasodilator commonly prescribed not only for erectile dysfunction but also for patients with pulmonary hypertension. Toradol (ketorolac) is an NSAID for pain and inflammation.

The ICU nurse is concerned with her patient's arterial blood gas (ABG) results especially the pH 7.30; and PCO2 49 mm Hg. The nurse interprets these ABG results to mean respiratory acidosis. The nurse knows which of the following are clinical manifestations of respiratory acidosis? Select all that apply. A) Headache with complaints of blurred vision B) Muscle twitching C) Hyperactive deep tendon reflexes D) Complaints of paresthesia sensations around the lips/mouth E) Numbness in the fingers and toes

Ans: A, B Feedback: Carbon dioxide readily crosses the blood-brain barrier, exerting its effects by changing the pH of brain fluids. Elevated levels of CO2 produce vasodilation of cerebral blood vessels, causing headache, blurred vision, irritability, muscle twitching, and psychological disturbances. Distracters C and D are related to hypocalcaemia. Numbness in the fingers and toes correlates with respiratory alkalosis.

The neonatologist suspects an infant has developed sepsis with multiorgan system illness. The nurse caring for this infant will note which of the assessment findings support this diagnosis. Select all that apply. A) Decreasing BP with increase in heart rate indicative of shock B) Prolonged PT and PTT and decrease in platelet count C) Frequent voiding of a small amount of light-colored urine D) Bilateral warm feet but pedal pulses hard to palpate E) Positive Moro reflex when loud noise made at crib side

Ans: A, B Feedback: Premature infants' health is severely impacted by early-onset infections and progressive multiorgan system illness. Infants with sepsis frequently present with respiratory failure, shock, meningitis, DIC, acute tubular necrosis, and symmetrical peripheral gangrene. Positive Moro reflex is normal for this infant.

A patient who has had a prolonged period of nasogastric (NG) suctioning following colon surgery is experiencing electrolyte imbalances. The magnesium level is low (1.2 mg/dL). Knowing that magnesium deficiency occurs in conjunction with low calcium levels, the nurse should assess the patient for which of the following clinical manifestations of hypocalcaemia? Select all that apply. A) Personality changes B) Hyperactive reflexes C) Increase in ventricular arrhythmias D) Increase in bouts of atrial fibrillation E) Symptomatic hypotension

Ans: A, B, C Feedback: Hypocalcaemia may be evidenced by personality changes and neuromuscular irritability along with tremors, choreiform movements, and positive Chvostek or Trousseau signs. Cardiovascular manifestations include tachycardia, hypertension, and ventricular dysrhythmias.

Which of the following individuals are at risk of developing metabolic alkalosis? Select all that apply. A) A 70-year-old woman who has taken two tablespoons of baking soda to settle her "sour stomach" B) A hospital patient who is on nasogastric suction following gastric surgery C) A 20-year-old male who has been regularly inducing himself to vomit following binge eating D) A 33-year-old male patient who is on mechanical ventilation in the intensive care unit following a head injury E) A 58-year-old alcoholic male who has been foregoing food for several weeks while drinking heavily F) A 60-year-old female who has chronic renal failure secondary to hypertension

Ans: A, B, C Feedback: Ingestion of bicarbonate, gastric suction, and vomiting are causes of metabolic alkalosis. Patients on mechanical ventilation are at risk of respiratory alkalosis, while heavy alcohol use and renal failure are associated with acidosis.

The nurse caring for a male child with respiratory problems is concerned he may be developing respiratory failure. Upon assessment, the nurse knows that which of the following are clinical manifestations of respiratory failure? Select all that apply. A) Severe accessory muscle retractions B) Nasal flaring C) Grunting on expiration D) Inspiratory wheezes heard E) Swollen glottis

Ans: A, B, C Feedback: Children with impending respiratory failure due to airway or lung disease have rapid breathing; exaggerated use of the accessory muscles; retractions, which are more pronounced in the child than in an adult because of more compliant chest; nasal flaring; and grunting during expiration. Inspiratory wheezes are usually associated with asthma. Swollen glottis can occur with strep throat.

A patient arrives in the ED after an automobile accident. Which of the following clinical manifestations lead the nurse to suspect a pneumothorax? Select all that apply. A) Respiratory rate 34 B) Asymmetrical chest movements, especially on inspiration C) Diminished breath sounds over the painful chest area D) Pulse oximetry 98% E) ABG pH level of 7.38

Ans: A, B, C Feedback: Manifestations of pneumothorax include increase in respiratory rate, dyspnea, asymmetrical movements of the chest wall, especially during inspiration, hyper resonant sound on percussion, and decreased or absent breath sounds over the area of pneumothorax. The pulse oximetry reading is normal. ABG pH level of 7.38 is a normal finding.

It is often necessary to obtain baseline data prior to initiating many forms of drug therapy. These baseline data include what? (Select all that apply.) A) Education level B) Allergies C) Drug use D) Number of members in family E) Father's occupation

Ans: A, B, C Feedback: Assessing educational level allows the nurse to plan an effective teaching plan. Allergies must be fully assessed before administering any medication to prevent allergic responses. Understanding the patient's current drug use informs the nurse about drugs that may interact or be impacted. Knowing the number of family members and father's occupation would not promote safer drug administration.

What is the responsibility of the nurse related to the patient's drug therapy? (Select all that apply.) A) Teaching the patient how to cope with therapy to ensure the best outcome B) Providing therapy as well as medications C) Evaluating the effectiveness of therapy D) Altering the drug regimen to optimize outcome E) Recommending appropriate over-the-counter medications to treat adverse effects of prescription drug therapy

Ans: A, B, C Feedback: A nurse is, therefore, a key health care provider who is in a position to assess the whole patient, to administer therapy as well as medications, to teach the patient how best to cope with the therapy to ensure the most favorable outcome, and to evaluate the effectiveness of the therapy. Nurses do not alter drug therapy or recommend over-the-counter medications because prescribing is outside the nurse's scope of practice.

The nurse learns that a drug needed by the patient is classified as an orphan drug and recognizes what as a reason for this classification? (Select all that apply.) A) The drug is rarely prescribed. B) The drug has dangerous adverse effects. C) The drug treats a rare disease. D) The patent on the medication is still effective. E) Production by a company that only manufactures drugs.

Ans: A, B, C Feedback: Drugs are classified as orphan drugs when they are not financially viable for a drug company to produce either because of risk for lawsuits about adverse effects or because the drug is not prescribed, which is often seen in rare diagnoses. Generic drugs are not produced until the patent expires, but this has no impact on classifying a particular drug as an orphan drug. Generic drugs are often produced by companies that only manufacture drugs without conducting research, but this has no bearing on the classification of orphan drugs.

The patient asks the nurse, Is it safe to take over-the-counter (OTC) medications with prescription medications? What is the nurse's best response? (Select all that apply.) A) OTC medications can interact with prescription medications. B) It is important to tell your doctor all medications you take, including OTC. C) OTC medications could mask or hide signs and symptoms of a disease. D) You should avoid taking any OTC medication when taking prescription drugs. E) Taking OTC medications can make your prescription medication more effective.

Ans: A, B, C Feedback: OTC medications can interact with prescription medications or other OTC so it is always important to consult your pharmacist and provider for advice. To provide the most accurate instruction, the health care provider must know all medications taken including dietary supplements, OTC, and prescription. OTC medications could mask or hide symptoms of a disease so it is always important to consult a physician if symptoms persist. OTC medications are not prohibited when taking prescription drugs as long as no drug interaction occurs. How an OTC will impact a prescription medication varies depending on the medications involved, so it is incorrect to say it will make the prescription drug more effective.

Before administering a prescription medication, what information does the nurse find on the drug label? (Select all that apply.) A) Brand name B) Generic name C) Drug concentration D) Expiration date E) Adverse effects

Ans: A, B, C, D Feedback: Prescription drug labels will contain the brand name, generic name, drug concentration, and expiration date. Adverse effects will not be listed on drug labels.

A 55-year-old male client with a history of cardiovascular disease has been admitted to the intensive care unit after recovering from cardiogenic shock. In the hours since admission, the client's arterial blood gases indicate acidosis, most likely acute lactic acidosis. Which of the following signs, symptoms, and diagnostic findings might his care team anticipate before the acid-base balance is restored? Select all that apply. A) Decreased pH B) Cardiac dysrhythmias C) Decreased alertness and cognition D) Hypoventilation E) Nausea and vomiting

Ans: A, B, C, E Feedback: As with any form of acidosis, pH is apt to be lower than normal. Metabolic acidosis is also associated with dysrhythmias, decreased alertness, and nausea and vomiting. Respiration is likely to be increased in both rate and depth.

The nurse promotes optimal drug effectiveness by doing what? (Select all that apply.) A) Incorporate basic history and physical assessment factors into the plan of care. B) Evaluate the effectiveness of drugs after they have been administered. C) Modify the drug regimen to modify adverse or intolerable effects. D) Minimize the number of medications administered to patients. E) Examine factors known to influence specific drugs if they are to be effective.

Ans: A, B, C, E Feedback: Incorporate basic history and physical assessment factors into any plan of care so that obvious problems can be identified and handled promptly. If a drug simply does not do what it is expected to do, further examine the factors that are known to influence drug effects. Frequently, the drug regimen can be modified to deal with that influence. Minimizing the number of medications administered is usually not an option because each drug is ordered for a reason of necessity for the patient

A nurse is providing care for a client who has been admitted to a medical unit with a diagnosis of bronchiectasis. Which of the following signs and symptoms should the nurse expect to find during physical assessment of the client and the review of the client's history? Select all that apply. A) Recurrent chest infections B) Production of purulent sputum C) A barrel chest D) Low hemoglobin levels E) Recent surgery

Ans: A, B, D Feedback: Chest infections, copious production of purulent sputum, and anemia are all associated with bronchiectasis. A barrel chest is more commonly evident with emphysema, and recent surgery is not a noted factor.

Several processes enable a drug to reach a specific concentration in the body. Together they are called dynamic equilibrium. What are these processes? (Select all that apply.) A) Distribution to the active site B) Biotransformation C) Absorption from the muscle D) Excretion E) Interaction with other drugs

Ans: A, B, D Feedback: The actual concentration that a drug reaches in the body results from a dynamic equilibrium involving several processes: Absorption from the site of entry (can be from the muscle, the gastrointestinal (GI) tract if taken orally, of the subcutaneous tissue if given by that route); Distribution to the active site; biotransformation (metabolism) in the liver; excretion from the body. Interaction with other drugs is not part of the dynamic equilibrium.

A patient with asthma is going to begin taking an inhaled steroid. The nurse teaching the patient that what adverse effects may occur when using this drug? (Select all that apply.) A) Headache B) Rebound congestion C) Sepsis D) Epistaxis E) Depression

Ans: A, B, D Feedback: Adverse effects associated with the use of inhaled steroids include irritability, not depression, headache, rebound congestion, local infection, not sepsis and epistaxis.

The nurse administers antipsychotic medications to the patient who has taken these same drugs for many years. What signs and symptoms would the nurse attribute to secondary actions of the drug? (Select all that apply.) A) Muscular tremors B) Drooling C) Changes in gait D) Yellow discoloration of skin and sclera E) Fine red rash on the trunk

Ans: A, B, D Feedback: Drugs that affect the dopamine levels in the brain (e.g., antipsychotic drugs), cause a syndrome that resembles Parkinson's disease including lack of activity, akinesia, muscular tremors, drooling, changes in gait, rigidity, extreme restlessness or jitters (akathisia), or spasms (dyskinesia). Yellow discoloration of the skin and sclera indicate jaundice and would suggest liver damage. A fine red rash on the trunk would be a dermatologic reaction unrelated to an antipsychotic agent's secondary effects.

The nurse needs to ask what specific questions when collecting a drug history? (Select all that apply.) A) Do you take any over-the-counter medications? B) Do you take any herbal supplements? C) Do you use any alternative therapies? D) Do you take any natural supplements or vitamins? E) What unusual therapies do you take?

Ans: A, B, D Feedback: The nurse needs to specifically question the patient's use of over-the-counter drugs, herbal supplements, natural supplements, and vitamins. Use of terms like alternative therapies or unusual therapies is too vague and may not elicit the kind of information needed.

An individual calls the nurse help line and asks what the drug diphenhydramine is used for. The nurse knows that the medication is prescribed for which conditions? (Select all that apply.) A) Urticaria B) Vasomotor rhinitis C) Productive cough D) Motion sickness E) Angioedema

Ans: A, B, D, E Feedback: Diphenhydramine is used for the symptomatic relief of perennial and seasonal rhinitis, vasomotor rhinitis, allergic conjunctivitis, urticaria, and angioedema; it is also used for treating motion sickness and parkinsonism, as a nighttime sleep aid, and to suppress cough. It would not be used to treat a productive cough, because it is not an expectorant.

Vitamin D is integral to the regulation of calcium and phosphate levels. Put the following steps in the action of vitamin D into the correct sequence. Use all the options. A) Vitamin D is present in the skin or intestine. B) Vitamin D is concentrated in the liver. C) Absorption of calcium from the intestine increases. D) Vitamin D is transported to the kidneys. E) Calcitriol is produced.

Ans: A, B, D, E, C Feedback: Vitamin D is either synthesized in the skin by ultraviolet exposure or obtained from the intestines following ingestion. It is then concentrated in the liver and transported to the kidneys

A patient has been using guaifenesin for a cough that accompanied a common cold. The patient calls the nurse help line and states that she thinks she is having an adverse reaction to the medication. The nurse knows that which symptoms are adverse effects of this medication? (Select all that apply.) A) Nausea B) Rash C) Constipation D) Bleeding E) Headache

Ans: A, B, E Feedback: Adverse effects of guaifenesin are nausea, vomiting, headache, dizziness, and rash. Constipation and bleeding are not adverse effects of guaifenesin.

A patient with pulmonary hypertension may display which of the following clinical manifestations? Select all that apply. A) Shortness of breath B) Decreased exercise tolerance C) Nasal flaring D) Grunting on expiration E) Swelling (edema) of the legs and feet

Ans: A, B, E Feedback: Symptoms of PAH typically progress from shortness of breath and decreasing exercise tolerance to right heart failure, with marked peripheral edema and functional limitations. Other common symptoms include fatigue, angina, and syncope (fainting) or near-syncope. Nasal flaring and expiratory grunting are usually seen in infants experiencing respiratory distress.

The nurse has just admitted a patient with asthma and the emergency department doctor has ordered the patient to begin taking zafirlukast (Accolate). The nurse would hold the medication and contact the physician if the patient reported taking which medication at home? (Select all that apply.) A) Propranolol B) Warfarin C) Acetaminophen D) Ampicillin E) Terfenadine

Ans: A, B, E Feedback: Use zafirlukast with caution if propranolol, theophylline, terfenadine, or warfarin is taken at the same time because increased toxicity can occur. Toxicity may also occur if these drugs are combined with calcium channel blockers, cyclosporine, or aspirin. No reported change of toxicity occurs when the patient is currently taking acetaminophen or ampicillin.

When a drug is ordered off-label, what must the nurse be clear about before administering the drug? (Select all that apply.) A) Why the drug is being given B) Its potential for problems C) The research that has been done D) The age group it was pretested on E) The intended use

Ans: A, B, E Feedback: Liability issues surrounding many of these uses are very unclear, and the nurse should be clear about the intended use, why the drug is being given, and its potential for problems. Knowing the age group it was pretested on and knowing the research that has been done are not factors the nurse needs to know before administering the drug.

A patient who overdosed on aspirin is brought to the emergency department. The nurse caring for this patient should anticipate which of the following clinical manifestations? Select all that apply. A) Respiratory rate of 40 B) BP 100/72 C) ABG report: pH 7.50, PCO2 31 mm Hg, and HCO3 level 19 mmol/L. D) Urine output approximately 100 mL/hour E) Bilateral crackles (fluid) in the lungs

Ans: A, C Feedback: The salicylates cross the blood-brain barrier and directly stimulate the respiratory center, causing hyperventilation and respiratory alkalosis (answer choices A and C). The blood pressure is at normal range, and the urine output is normal or excessive depending on fluid intake. Bilateral crackles (fluid) in the lungs are usually a sign of heart failure.

What action does the nurse take during the intervention state of the nursing process related to drug therapy? (Select all that apply.) A) Administer the medication. B) Determine medication effectiveness. C) Document the medication. D) Analyze the data collected. E) Collect a nursing history.

Ans: A, C Feedback: During the implementation phase, the nurse administers and documents the medication. Effectiveness of the medication is determined during the evaluation phase. Analyzing data occurs when assigning appropriate nursing diagnoses. Collecting a nursing history is part of the assessment stage of the nursing process.

A patient who has been on a high-protein diet comes to the emergency department with respiratory symptoms. Upon analysis of arterial blood gases (ABGs), the patient is diagnosed with hypercapnia. The nurse will note the ABG results that confirm this diagnosis include: Select all that apply. A) pH 7.31 (normal 7.35 to 7.45). B) PO2 of 97%. C) PCO2 of 58 mm Hg (normal 38 to 42). D) Serum HCO3of -33 mEq/L (normal 22 to 28). E) Serum K+ (potassium)of 3.6 mmol/L (normal 3.5 to 5.0).

Ans: A, C, D Feedback: Hypercapnia affects a number of body functions, including acid-base balance and renal, neurological, and CV functions. Elevated levels of PCO2 (38 to 42) produce a decrease in pH (7.35 to 7.45) and respiratory acidosis. Compensatory mechanisms result in an increase in serum HCO3 (22 to 28). In this example, the PO2 level is within normal range. Serum K+ is not part of the ABG analysis.

During a prenatal education class, a participant has related a story about how her friend's infant died of sudden infant death syndrome (SIDS). What can the educator tell the group about how they can prevent SIDS when they have their babies? Select all that apply. A) "The best sleeping position for your baby is on his back." B) "Children are at particular risk of SIDS when they have a cold or flu, so these times require extra vigilance." C) "Using drugs during pregnancy has been shown to be associated with SIDS after birth, which is one more reason for mothers to avoid them." D) "It's important if anyone in your home smokes to make sure they only do it outside." E) "The exact cause of SIDS still isn't known, so there's little that you can do to prevent this tragic event."

Ans: A, C, D Feedback: Prone or side-lying position, intrauterine drug exposure, and postnatal exposure to cigarette smoke are all associated with SIDS. Upper respiratory infections are not noted to present a particular risk, and though the exact etiology is not known, preventative measures do exist.

A premature infant who is receiving care in a neonatal intensive care unit (NICU) has just been identified as having necrotizing enterocolitis (NEC). Of the following clinical manifestations, identify those most likely to contribute to the diagnosis of NEC. Select all that apply. A) Feeding intolerance B) Inability to pass stool within the first 10 days of life C) Hard, taut abdomen with increasing distention D) Blood noted in stools E) Hypoactive bowel sounds on right lower quadrant

Ans: A, C, D Feedback: Immature immunity, shunting of circulation away from the GI tract, and infectious processes have all been implicated in the etiology of NEC. The classic initial symptoms are usually feeding intolerance, abdominal distention, and bloody stools shortly after the first week of life.

A patient has been prescribed an antihistamine for treatment of allergic rhinitis. What statements by the patient indicate an understanding of this medication? (Select all that apply.) A) This medication will work best if I take it before I eat anything. B) I need to drink less fluid while I take this medication to help reduce the amount of mucus I have. C) I will use sugarless candies to help with the feelings of a dry mouth. D) I will use a humidifier in the bedroom while I sleep. E) This medication will probably cause my appetite to increase.

Ans: A, C, D Feedback: Antihistamines should be taken on an empty stomach and the patient should force fluids, not drink less fluids. The patient may use sugarless candy to help with dry mouth and should increase room humidity. The patient may experience nausea or anorexia but not increased hunger. Options B and E are not correct.

What concerns might the nurse legitimately have related to the use of alternative therapies? (Select all that apply.) A) The Food and Drug Administration (FDA) does not test or regulate active ingredients. B) The incidental ingredients are clearly marked on the label. C) The dosage contained in each tablet may vary greatly. D) No alternative therapies have been found to be effective. E) Advertising of alternative products is not as restrictive or accurate.

Ans: A, C, E Feedback: Alternative products are not controlled or tested by the FDA and advertising is not as restrictive or accurate as with classic drugs. Incidental ingredients are often unknown and strength of tablets may vary within the bottle depending on the conditions under which they were grown. While some alternative therapies have been found to be effective, there are others who have not been studied.

Which of the following individuals would be considered to be at risk for the development of edema? Select all that apply. A) An 81-year-old man with right-sided heart failure and hypothyroidism B) A 60-year-old obese female with a diagnosis of poorly controlled diabetes mellitus C) A 34-year-old industrial worker who has suffered extensive burns in a job-related accident D) A 77-year-old woman who has an active gastrointestinal bleed and consequent anemia E) A 22-year-old female with hypoalbuminemia secondary to malnutrition and anorexia nervosa

Ans: A, C, E Feedback: Right-sided heart failure, burns, and low levels of plasma proteins are all associated with the development of edema. Diabetes and GI bleeds are not identified as contributors to edema.

The nursing instructor is talking to a group of nursing students about the treatment regimen for children with asthma. The students indicate they understand the information when they identify which class of drugs that comprise this regimen? (Select all that apply.) A) Long-acting inhaled steroids B) Xanthines C) Leukotriene-receptor antagonists D) Topical steroid nasal decongestants E) Beta-agonists

Ans: A, C, E Feedback: Antiasthmatics are frequently used in children. The leukotriene-receptor antagonists have been found to be especially effective for long-term prophylaxis in children. Acute episodes are best treated with a beta-agonist and then a long-acting inhaled steroid or a mast cell stabilizer. Xanthines (e.g., theophylline) have been used in children, but because of their many adverse effects and the better control afforded by newer agents, its use is reserved for patients who do not respond to other therapies. Topical steroid nasal decongestants may be used for symptom relief for nasal congestion but are not a regular part of asthma therapy in children.

Which of the following clients are displaying known risk factors for the development of pulmonary emboli? Select all that apply. A client who is: A) immobilized following orthopedic surgery. B) experiencing impaired Cl- and Na+ regulation. C) taking amiodarone for the treatment of premature ventricular contractions. D) a smoker and who takes oral contraceptives. E) undergoing radiation therapy for the treatment of breast cancer.

Ans: A, D Feedback: Postsurgical immobility, smoking, and the use of oral contraceptives are all identified risk factors for the development of pulmonary emboli. Impaired Cl- and Na+ regulation are associated with cystic fibrosis, while amiodarone and radiation therapy are linked to interstitial lung diseases.

Teaching the patient/caregiver about her or his medications is an important step in reducing the risk of medication errors. What is an important teaching point about medications? (Select all that apply.) A) Speak up and ask questions. B) Store medications in a warm humid place. C) Adjust your medication according to how you feel. D) Keep a list of your prescribed medications. E) Take all medications together in the morning.

Ans: A, D Feedback: Appropriate patient teaching will reduce the risk of medication errors and complications. Nurses teach patients to speak up, ask questions, and act as his or her own advocate when medications are being prescribed. He should keep a complete list of medications and have a copy available at all times in case of accident. Store drugs in a dry, cool place away from children and pets that could be harmed. Take medications as they have been prescribed and do not adjust dosage without authorization from the prescriber. Take medications at the time they are prescribed to be taken, always being aware that some drugs cannot be taken together.

A terminally ill cancer patient with metastasis to the bone has been admitted with elevated calcium levels (hypercalcemic crisis). The patient is very lethargic and exhibiting muscle flaccidity. The nurse should be prepared to administer (Select all that apply.) A) pamidronate, a bisphosphonate. B) intravenous drip of insulin. C) furosemide, a loop diuretic. D) gallium nitrate, a gallium salt of nitric acid. E) prednisone, a corticosteroid.

Ans: A, D, E Feedback: The bisphosphonates (e.g.,pamidronate, zoledronate), which act mainly by inhibiting osteoclastic activity, provide a significant reduction in calcium levels with relatively few side effects. Calcitonin also inhibits osteoclastic activity. Gallium nitrate is highly effective in the treatment of severe hypercalcemia associated with malignancy. Prednisone, a corticosteroid, inhibits bone resorption.

The nurse is caring for a patient who is scheduled to receive acetylcysteine because of an acetaminophen overdose. The nurse would notify the physician before administering the medication if the patient had which condition? (Select all that apply.) A) Bronchospasm B) Hypertension C) Nephrotic syndrome D) Peptic ulcer E) Esophageal varices

Ans: A, D, E Feedback: Before administration, assess for possible contraindications or cautions: any history or allergy to the prescribed drugs and the presence of bronchospasm, which are contraindications to the use of these drugs, as well as findings of peptic ulcer and esophageal varices, which would require careful monitoring and cautious use. Options B and C are not correct.

A patient who has just had her first postoperative dinner out to celebrate her recovery from an intestinal bypass is brought to the emergency room by her spouse. He reports that the patient seems disoriented and is slurring her words. The patient did not have any alcohol with her pasta dinner. Which of the following might be the cause of her symptoms? A) Ketoacidosis B) Lactic acidosis C) Hypercapnia D) Hypothalemia

Ans: B Feedback: A unique form of lactic acidosis, called D-lactic acidosis, can occur in persons with intestinal disorders that involve the generation and absorption of D-lactic acid. D-lactic acidosis can occur in persons with jejunoileal bypass, in which there is impaired reabsorption of carbohydrate in the small intestine. Persons with D-lactic acidosis experience episodic periods of metabolic acidosis often brought on by eating a meal high in carbohydrates. Manifestations include confusion, cerebellar ataxia, slurred speech, and loss of memory. They may complain of feeling (or may appear) intoxicated.

A 31-year-old client with a diagnosis of end-stage liver failure has been admitted to the intensive care unit of a hospital. Arterial blood sampling indicates that the man has an acid-base imbalance. Which of the following situations is most likely to result in an inappropriate pH? A) Conservation or formation of new HCO3- by the kidneys B) Low albumin and plasma globulin levels C) Transcompartmental exchange of H+ and potassium ions D) Renal excretion of HCO3- in the presence of excess base

Ans: B Feedback: Albumin and plasma globulins are key protein buffers in the vascular compartment; consequently, a low albumin level, as is common in liver failure, is apt to result in acid-base imbalances. Answer choices A, C, and D all convey normal physiological processes that help to maintain pH.

A 51-year-old female client who is 2 days postoperative in a surgical unit of a hospital is at risk of developing atelectasis as a result of being largely immobile. Which of the following teaching points by her nurse is most appropriate? A) "Being in bed increases the risk of fluid accumulating between your lungs and their lining, so it's important for you to change positions often." B) "You should breathe deeply and cough to help your lungs expand as much as possible while you're in bed." C) "Make sure that you stay hydrated and walk as soon as possible to avoid us having to insert a chest tube." D) "I'll proscribe bronchodilator medications that will help open up your airways and allow more oxygen in."

Ans: B Feedback: Atelectasis is characterized by incomplete lung expansion and can often be prevented by deep breathing and coughing. Pleural effusion, not atelectasis, is associated with fluid accumulation between the lungs and their lining, and neither chest tube insertion nor bronchodilators are common treatments for atelectasis.

Which of the following phenomena is most likely occurring during a child's alveolar stage of lung development? A) Terminal alveolar sacs are developing, and surfactant production is beginning. B) A single capillary network exists, and the lungs are capable of respiration. C) The conducting airways are formed, but respiration is not yet possible. D) Primitive alveoli are formed, and the bronchi and bronchioles become much larger.

Ans: B Feedback: During the alveolar stage of lung development from late fetal to early childhood, a single capillary network appears, and the lungs are ready to perform respiration. The development of alveolar sacs and production of surfactant are associated with the saccular period, and formation of the conducting airways occurs during the pseudoglandular period. Formation of primitive alveoli takes place during the canalicular period.

In the grocery store, a nurse overhears a teenage mother intentionally shaming and verbally reprimanding a child in public. The mother also grabbed the child's stuffed animal and tore the limbs off. From what the nurse remembers about abuse, this would be classified as a form of A) physical abuse. B) emotional abuse. C) sexual abuse. D) neglect.

Ans: B Feedback: Emotional abuse or psychological maltreatment includes methods of verbal abuse, shaming, destruction of child's personal property, harming or killing child's pet, and bullying.

What topic should health promotion initiatives emphasize if the target audience is parents of preschoolers and the goal is to minimize mortality? A) Handwashing as an infection control measure B) Injury prevention especially when the child is near water C) Identifying signs of child abuse and neglect D) The importance of good nutrition

Ans: B Feedback: Injuries are the leading cause of death in children aged 1 to 4. While handwashing does prevent many infections, these are not commonly fatal. Likewise, child abuse and poor nutrition are valid educational topics, but they do not relate as directly and frequently to childhood death as do injuries.

An 81-year-old female has a long-standing diagnosis of hypocalcemia secondary to kidney disease. She will be moving into an assisted living facility shortly. Which of the following clinical manifestations would the nursing staff at the facility likely observe in this patient? A) Loss of appetite and complaints of nausea B) Muscular spasms and complaints of tingling in the hands/feet C) High fluid intake and copious amounts of dilute urine output D) Lethargy and change in level of consciousness

Ans: B Feedback: Muscular spasms and cramping are common manifestations of low serum calcium. Polydipsia, polyuria, anorexia, lethargy, and stupor are associated with hypercalcemia.

A 56-year-old female hospital patient with a history of alcohol abuse is receiving intravenous (IV) phosphate replacement. Which of the following health problems will this IV therapy most likely resolve? A) The client has an accumulation of fluid in her peritoneal cavity. B) The client is acidotic and has impaired platelet function. C) The client has an irregular heart rate and a thread pulse. D) The client has abdominal spasms and hyperactive reflexes.

Ans: B Feedback: Phosphate is necessary for the normal function of platelets and the excretion of hydrogen ions that contribute to acidosis. Phosphate replacement would be unlikely to resolve ascites and cardiac anomalies, while abdominal spasms and hyperactive reflexes are more likely consequences of low calcium levels.

A client with a newborn infant is also the caregiver for her 75-year-old mother, who lives with them and who has diabetes. The client requests pneumonia vaccinations for her entire household. Which vaccine is most likely to be effective for the baby? A) Since the baby's immune system is mature at birth, regular vaccine is appropriate. B) There is no effective vaccine for newborn infants. C) The 23-valent vaccine will be effective. D) No vaccine is necessary for the baby if the nursing mother is immunized.

Ans: B Feedback: S. pneumoniae capsular polysaccharides would be especially appropriate for the client and her diabetic, elderly mother but is not effective in the immune system of anyone younger than 2 years old. Fortunately, a newer, 7-valent vaccine was designed to protect infants as young as 7 months. However, because their immune system is immature, the antibody response to most flu shots is poor or inconsistent in children younger than 2 years of age.

A 3-year-old boy has developed croup following a winter cold. His care provider would recognize that which of the following microorganisms and treatments is most likely to be effective? A) Respiratory syncytial virus treated with intubation B) Parainfluenza virus treated with a mist tent and oxygen therapy C) Haemophilus influenza treated with appropriate antibiotics D) Staphylococcus aureus treated with bronchodilators and mist tent

Ans: B Feedback: The majority of croup cases are caused by parainfluenza viruses, and common treatment modalities are humidified air or mist tents as well as supplementary oxygen. Respiratory syncytial virus accounts for some croup diagnoses, but intubation is not normally required. Haemophilus influenza is responsible for epiglottitis, while Staphylococcus aureus is not commonly responsible for croup.

A patient with ESRD comes into the emergency department in severe acidosis. The nurse notes that the respiratory rate is 36 breaths/minute. The nurse understands the pathophysiology of this response and explains to the student nurse that the patient's A) anxiety level is high, and the body is trying to release endorphins. B) chemoreceptors in the carotid and aortic bodies have noticed the pH change and altered the ventilator rate. C) kidneys are not able to buffer the acid and require the help from the lungs D) lungs are trying to excrete excess hydrogen.

Ans: B Feedback: The second line of defense against acid-base disturbances is the control of extracellular CO2 by the lungs. Blood PCO2 and pH are important regulators of ventilation. Chemoreceptors in the brain stem and the peripheral chemoreceptors in the carotid and aortic bodies sense changes in PCO2 and pH and alter the ventilatory rate.

A nurse is providing care for an older, previously healthy adult male who has been diagnosed today with pneumococcal pneumonia. Which of the following signs and symptoms is the nurse most likely to encounter? A) The man will be hypotensive and febrile and may manifest cognitive changes. B) The patient will have a cough producing clear sputum, and he will have faint breath sounds and fine crackles. C) The patient will have copious bloody sputum and diffuse chest pain and may lose his cough reflex. D) The patient will lack lung consolidation and will have little, if any, sputum production.

Ans: B Feedback: The typical onset of pneumococcal pneumonia involves production of clear sputum, along with faint breath sounds and fine crackles. The patient is less likely to be hypotensive, have copious bloody sputum, or have chest pain. A lack of lung consolidation or sputum production is more closely associated with atypical pneumonias.

A patient is admitted for a relapse for sarcoidosis. Knowing this is usually caused by an inflammatory process, the nurse can anticipate administering A) a bronchodilator. B) a corticosteroid. C) aspirin. D) an albuterol inhaler.

Ans: B Feedback: Treatment is directed at interrupting the granulomatous inflammatory process that is characteristic of the disease and managing the associated complications. When treatment is indicated, corticosteroid drugs are used. Bronchodilators may be used if there is wheezing, but this is not a normal medication for this disease. Aspirin is a blood thinner. Albuterol is a short-term bronchodilator for acute asthma.

A nurse who works in a neonatal intensive care unit is providing care for an infant born at 26 weeks' gestation. Which of the following assessments would lead the nurse to suspect that the infant has developed respiratory distress syndrome (RDS)? A) The infant's blood pressure and temperature are normal measurements as expected. B) Infant is grunting and has notable intercostal retractions with respirations. C) Infant has poor motor skills and limited limb range of motion. D) Infant has apnea lasting 5 to 10 seconds with a decrease in heart rate, which reverses with tactile stimulation.

Ans: B Feedback: While premature birth is associated with numerous potential health problems, including variations in vital signs, impaired motor function, and neurological deficits, the most common complications of prematurity involve respiratory function.

A nurse is caring for a patient who is supposed to receive two drugs at the same time. What is the nurse's priority action? A) Wash her hands before handling the medications. B) Consult a drug guide for compatibility. C) Question the patient concerning drug allergies. D) Identify the patient by checking the armband and asking the patient to state his name.

Ans: B Feedback: A nurse should first consult a drug guide for compatibility when two or more drugs are being given at the same time. After compatibility is determined the medication can be administered. The nurse will perform hand hygiene, check for patient allergies, and ensure the right patient receives the medication by using two identifiers.

The patient is taking low dose aspirin daily for his heart. The nurse knows only a portion of the medication taken actually reaches the tissue due to what process? A) Distribution B) First-pass effect C) Reduced absorption D) Gastrointestinal circulation

Ans: B Feedback: Drugs that are taken orally are usually absorbed from the small intestine directly into the portal venous system and then delivers these absorbed molecules into the liver, which immediately break the drug into metabolites, some of which are active and cause effects in the body, and some of which are deactivated and can be readily excreted from the body. As a result, a large percentage of the oral dose is destroyed at this point and never reaches the tissues. This process is not caused by distribution, absorption, or gastrointestinal circulation.

A nurse is planning patient teaching about a newly prescribed drug. What is a priority teaching point included by the nurse to improve compliance and safety? A) List of pharmacies where the drug can be obtained B) Measures to alleviate any discomfort associated with adverse effects C) The cost of the brand name drug compared with the generic form D) Statistics related to Phase III testing for the prescribed drug

Ans: B Feedback: If a patient is aware of certain adverse effects and how to alleviate or decrease the discomfort, he or she is more likely to continue taking the medication. A list of pharmacies can be useful information but will not improve safety or compliance. Knowing the cost of the brand name versus the generic form could also be helpful to the patient. However, a substitution may not be allowable and the cost of a drug does not improve patient safety. Most patients are not concerned with the statistics related to drug testing and it would not improve compliance or safety even if the patient was interested in the information.

A newly admitted patient has orders to receive 1,000 mL of normal saline IV over 8 hours. If the IV infusion set is a microdrip set that delivers 60 drops per mL, how many drops per minute should the nurse administer to the patient? A) 60 drops/min B) 125 drops/min C) 240 drops/min D) 480 drops/min

Ans: B Feedback: If a patient was to receive 1,000 mL in 8 hours, dividing 1000 by 8 would mean that the patient would receive 125 mL in 1 hour, or 60 minutes. Setting up the equation, 60 drops/mL ÷ X = 125 mL/60 minutes; cross-multiplying, the answer is 125 drops/min.

A client is brought to the emergency department with complaints of shortness of breath. Assessment reveals a full, bounding pulse, severe edema, and audible crackles in lower lung fields bilaterally. What is the client's most likely diagnosis? A) Hyponatremia B) Fluid volume excess C) Electrolyte imbalance: hypocalcemia D) Hyperkalemia

Ans: B Feedback: Peripheral and pulmonary edema as well as a bounding pulse and dyspnea are indicators of fluid volume overload.

What would the nurse provide when preparing a patient for discharge and home medication self- administration? A) Personal contact information to use if the patient has questions B) Thorough medication teaching about drugs and the drug regimen C) Over-the-counter medications to use to treat potential adverse effects D) A sample size package of medication to take home until prescription is filled

Ans: B Feedback: The nurse is responsible for providing thorough medication teaching about drugs and the drug regimen to ensure the patient knows how to take the medication and when to notify the provider. The nurse never provides personal contact information to a patient. If adverse effects arise, the patient is taught to call the health care provider and should not self-medicate with over-the-counter drugs, which could mask serious symptoms. The nurse never dispenses medication because it must be properly labeled for home use; this is done by the pharmacy.

The patient returns from the postanesthesia care unit (PACU) with the following order: morphine 3 mg IV every 2 hours as needed for relief of pain. The vial reads morphine, 4 mg/mL. How many milliliters of morphine will the nurse administer? A) 1 mL B) 0.75 mL C) 0.5 mL D) 0.25 mL

Ans: B Feedback: Using the figures from this problem: 4 mg/1 mL = 3 mg/X. Cross-multiply yielding 0.75 mL.

A pneumonia that occurs 48 hours or more after admission to the hospital is considered A) community-acquired pneumonia. B) hospital-acquired pneumonia. C) viral pneumonia. D) immunocompromised pneumonia.

Ans: B Feedback: Hospital-acquired pneumonia is defined as a lower respiratory tract infection that was not present or incubating on admission to the hospital. Usually, infections occurring 48 hours or more after admission are considered hospital acquired. Community- acquired pneumonia is diagnosed within 48 hours after admission. Most hospital- acquired pneumonia is bacterial.

A nurse is providing care for a client who has been diagnosed with metabolic alkalosis after several days of antacid use. Which of the following treatments should the nurse prepare to give? A) Intravenous or oral administration of free hydrogen ions B) Intravenous administration of KCl solution C) Administration of oxygen and NaHCO3 solution D) Supplementary oxygen and possible mechanical ventilation

Ans: B Feedback: KCl administration facilitates the renal retention of hydrogen ions, resulting in lowering of pH. It is not possible to administer free H+ ions, and sodium bicarbonate would exacerbate her condition. Mechanical ventilation is indicated in cases of respiratory acidosis.

The nurse is caring for a patient with ketoacidosis, who is complaining of increasing lethargy and occasional confusion following several weeks of rigid adherence to a carbohydrate-free diet. The nurse understands which of the following phenomena is most likely occurring? A) High-fat, low-carbohydrate dietary intake is associated with respiratory acidosis. B) In the absence of carbohydrate energy sources, her body is metabolizing fat and releasing ketoacids. C) Metabolism of dietary fats without the buffer action of carbohydrates results in the catabolism of ketoacids. D) Decreased carbohydrate intake induces insulin deficiency and consequent ketoacidosis.

Ans: B Feedback: Low-carbohydrate diets can induce the fat metabolism and consequent metabolic acidosis that is more commonly associated with diabetic ketoacidosis. The acidotic state is not classified as respiratory in nature and does not involve a buffer role for carbohydrates or insulin deficiency.

A 62-year-old female smoker is distraught at her recent diagnosis of small cell lung cancer (SCLC). How can her physician most appropriately respond to her? A) "I'm sure this is very hard news to hear, but be aware that with aggressive treatment, your chances of beating this are quite good." B) "This is very difficult to hear, I'm sure, and we have to observe to see if it spreads because that often happens." C) "I'm very sorry to have to give you this news; I'd like to talk to you about surgical options, however." D) "This is a difficult diagnosis to receive, but there is a chance that the cancer may go into remission."

Ans: B Feedback: Metastases are common with SCLC. Survival rates are very low; surgical options do not exist; and remission is very unlikely

A 41-year-old male client has presented to the emergency department with an acute onset of increased respiratory rate and difficulty breathing. STAT chest x-ray indicates diffuse bilateral infiltrates of his lung tissue, and ECG displays no cardiac dysfunction. What is this client's most likely diagnosis? A) Cor pulmonale B) Acute lung injury C) Pulmonary hypertension D) Sarcoidosis

Ans: B Feedback: Rapid onset of respiratory distress accompanied by diffuse bilateral infiltrates of lung tissue and an absence of cardiac changes are associated with acute lung injury/acute respiratory distress syndrome. These particular signs and symptoms are not as closely associated with cor pulmonale, pulmonary hypertension, or sarcoidosis.

The pharmacology instructor is discussing the adrenergic drug ephedrine with the nursing students and lists an adverse reaction of this drug as what? A) Bronchoconstriction B) Hyperglycemia C) Cardiac arrhythmias D) Severe constipation

Ans: B Feedback: Ephedrine (generic), a drug used as a bronchodilator to treat asthma and relieve nasal congestion, can break down stored glycogen and cause an elevation of blood glucose by its effects on the sympathetic nervous system. Ephedrine does not cause bronchoconstriction, cardiac arrhythmias, or severe constipation.

A patient presents to the emergency department with a drug level of 50 units/mL. The half-life of this drug is 1 hour. With this drug, concentrations above 25 units/mL are considered toxic and no more drug is given. How long will it take for the blood level to reach the non-toxic range? A) 30 minutes B) 1 hour C) 2 hours D) 3 hours

Ans: B Feedback: Half-life is the time required for the serum concentration of a drug to decrease by 50%. After 1 hour, the serum concentration would be 25 units/mL (50/2) if the body can properly metabolize and excrete the drug. After 2 hours, the serum concentration would be 12.5 units/mL (25/2) and reach the nontoxic range. In 30 minutes the drug level would be 37.5 units/mL, whereas in 3 hours the drug level would be 6.25.

The nurse is caring for a patient who is receiving acetylcysteine (Mucomyst) by face mask. What would be an appropriate nursing diagnosis? A) Impaired swallowing B) Risk for impaired skin integrity C) Risk for falls D) Sleep deprivation

Ans: B Feedback: A patient receiving acetylcysteine by face mask should have the residue wiped off the face mask and her face with plain water to prevent skin breakdown. The appropriate nursing diagnosis would be risk for impaired skin integrity. Acetylcysteine does not cause impaired swallowing, produce any CNS effects that could increase the risk for falls, or impair the patient's ability to sleep

The nurse is giving discharge instructions to a patient with an upper respiratory infection who has been advised to take an over-the-counter (OTC) topical nasal decongestant. The nurse advises the patient about what common adverse reaction to these medications? A) Diarrhea B) Rhinitis medicamentosa C) Rash D) Headache

Ans: B Feedback: An adverse effect that accompanies frequent or prolonged use of topical nasal decongestants is rebound congestion, technically called rhinitis medicamentosa. Other adverse reactions include disorientation, confusion, nausea, vomiting, fever, and dyspnea. Diarrhea, rash, and headache are not commonly associated with these drugs, however.

The nurse is caring for a patient with chronic obstructive pulmonary disease. The plan of care will focus on what patient problem? A) Pain B) Obstructed airway C) Activity intolerance D) Adverse effects of medication therapy

Ans: B Feedback: Asthma, emphysema, chronic obstructive pulmonary disease (COPD), and respiratory distress syndrome (RDS) are pulmonary obstructive diseases. All but RDS involve obstruction of the major airways. RDS obstructs the alveoli. Pain, activity intolerance, and adverse effects of medication therapy are conditions identified to detect, manage, and minimize the unexpected outcomes the nurse should be especially aware of the potential for an obstructed airway in these patients.

A patient has been prescribed a nasal steroid and asks the nurse what the most common reason that this medication is used to treat. What is the nurse's best response? A) Nasal steroids are only used to treat a sinus infection. B) Nasal steroids are used to treat allergic rhinitis. C) Nasal steroids are used for an infection in the adenoids. D) Nasal steroids are used for all acute upper respiratory infections.

Ans: B Feedback: Because nasal steroids block the inflammatory response, their use is contraindicated in the presence of acute infections. The most common reason they are prescribed is for the treatment of allergic rhinitis or to relieve inflammation after the removal of nasal polyps. Nasal steroids are not used for a sinus infection, an infection in the adenoids, or any other acute upper respiratory infection.

A patient comes to the clinic with symptoms as seen in a cold. What group of upper respiratory drugs causes local vasoconstriction, which decreases blood flow and shrinks swollen membranes to improve air flow? A) Antitussives B) Decongestants C) Expectorants D) Mucolytics

Ans: B Feedback: Decongestants cause local vasoconstriction that decreases blood flow to irritated and dilated capillaries of the mucous membranes lining the nasal passages and sinus cavities. This vasoconstriction leads to a shrinking of swollen membranes and opens clogged nasal passages promoting drainage of secretions and improved air flow. Antitussives either work directly on the medullary cough center of the brain or act as a local anesthetic on the respiratory passages blocking the effectiveness of the stretch receptors that stimulate a cough reflex. Expectorants liquefy lower respiratory tract secretions, reducing the viscosity of the secretions and so making it easier to cough them up. Mucolytics break down mucus to aid a person in coughing up thick tenacious secretions by separating cells that hold mucous material together.

The nurse is caring for a patient who is taking dextromethorphan for cough suppression. The nurse will assess this patient for hypotension if he also takes which other medication? A) Calcium-channel blockers B) Monoamine oxidase (MAO) inhibitors C) Beta-blockers D) Thiazide diuretics

Ans: B Feedback: Dextromethorphan should not be used in conjunction with MAO inhibitors because hypotension, fever, nausea, myoclonic jerks, and coma could occur. No known drugdrug interaction exists between dextromethorphan and calcium-channel blockers, beta-blockers, and thiazide diuretics.

Epinephrine, formerly the drug of choice for acute attacks of bronchoconstriction, has been replaced by what? A) Short-acting bronchodilators are the drug of choice in this situation. B) Nothing has replaced epinephrine as drug of choice in this situation. C) Self-administered metered-dose inhalers (MDIs) D) Long-acting beta2 -adrenergic agonists (LABAs)

Ans: B Feedback: Epinephrine, the prototype drug, is the drug of choice in adults and children for the treatment of acute bronchospasm, including that caused by anaphylaxis; it is also available for inhalation therapy. Because epinephrine is associated with systemic sympathomimetic effects, it is not the drug of choice for patients with cardiac conditions. Options A, C, and D are not correct.

The nurse is reviewing the patient's medication orders and finds an order stating amoxicillin 250 mg every 8 hours. What would the nurse question regarding this order? A) Dose B) Route C) Frequency of administration D) Allowance for generic substitution

Ans: B Feedback: For the nurse to administer a medication, all essential components of a medication order must be written by the prescriber including drug name, dosage, route, frequency, and patient name. This order is missing the route and the drug could be given IV, IM, or PO. The nurse should call the ordering health care provider and clarify what route the medication is to be administered.

The nurse explains that what drug resource book is compiled from package inserts? A) Nurses Drug Guide B) Physicias's Desk Reference (PDR) C) Drug Facts and Comparisons D) AMA Drug Evaluations

Ans: B Feedback: The PDR is a compilation of information found on package inserts. The Nurses Drug Guide uses more easily understood language and incorporates nursing considerations and patient teaching points. Drug Facts and Comparisons includes cost comparison, often not found in other drug resource guides. The AMA Drug Evaluations is far less biased than the PDR and includes drugs still in the research stage of development.

The patient is taking a 2-mg dose of ropinerol XR. The drug has a half-life of 12 hours. How long will it be before only 0.25 mg of this drug remains in the patient's system? A) 24 hours B) 36 hours C) 48 hours D) 60 hours

Ans: B Feedback: The half-life of a drug is the time it takes for the amount of drug in the body to decrease to half of the peak level it previously achieved. At 12 hours there will be 1 mg of the drug available to the body. At 24 hours there will be 0.5 mg; at 36 hours there will be 0.25 mg; at 48 hours there will be 0.125 mg, and at 60 hours there will be 0.0625 mg.

The clinic nurse is caring for a patient who has just been diagnosed with chronic obstructive pulmonary disease (COPD). The patient asks the nurse what COPD means. What would be the nurse's best response? A) It is an umbrella term for diseases like acute bronchitis. B) It means that the lungs have been damaged in such a way that there airflow is limited in and out of the lungs. C) It means your lungs can't expand and contract like they are supposed to, which makes it hard for you to breathe. D) It is a term that covers so many lung diseases I can't list them all.

Ans: B Feedback: The obstruction of asthma, emphysema, and COPD can be related to inflammation that results in narrowing of the interior of the airway and to muscular constriction that results in narrowing of the conducting tube. With chronic inflammation, muscular and cilial action is lost, and complications related to the loss of these protective processes can occur, such as infections, pneumonia, and movement of inhaled substances deep into the respiratory system. In severe COPD, air is trapped in the lower respiratory tract, the alveoli degenerate and fuse together, and the exchange of gases is greatly impaired. Options A, C, and D are not incorrect, but option B is the best response.

The nursing instructor observes several nursing students administer medications to their assigned patients. The instructor would stop what student from administering the medication until the error was corrected? A) The student checks the label on the medication against the administration record three times. B) The student enters the patient's room and asks the patient Are you Mr. Jones? C) The student checks the drug's brand name and generic name before taking it to the patient. D) The student documents the medication upon exiting the patient's room.

Ans: B Feedback: When the student asks the patient, Are you Mr. Jones the patient may nod in agreement, even if that is not his name. Perhaps he misheard, or maybe he wasn't paying attention to the name, or he may just want to be agreeable but this manner of ensuring the right patient receives a medication often results in errors. The student is correct to check the medication name against the medication administration record (MAR) three times to ensure the right drug is administered. The student who checks brand name and generic name is accurate in making sure the correct drug is prepared. Medication should be documented as soon as they are given so this student is also correct.

A student nurse asks her study group how to define a drug allergy. What would be the peer group's best response? A) A second effect of the body to a specific drug B) The formation of antibodies to a drug protein causing an immune response when the person is next exposed to that drug. C) A serum sickness caused by a reaction to a drug D) Immediate systemic reaction to the drug when exposed to the drug the first time.

Ans: B Feedback: A drug allergy is the formation of antibodies to a drug or drug protein; causes an immune response when the person is next exposed to that drug. A drug allergy does not occur at the first exposure to a drug. A second action of a specific drug is an adverse response that the drug causes in addition to the therapeutic effect. Serum sickness is one type of allergic reaction but does not define allergic reaction. An immediate systemic reaction to a drug, usually not on first exposure, is an anaphylactic reaction.

What would be the best source of drug information for a nurse? A) Drug Facts and Comparisons B) A nurse's drug guide C) A drug package insert D) The Physicians' Drug Reference (PDR)

Ans: B Feedback: A nurse's drug guide provides nursing implications and patient teaching points that are most useful to nurses in addition to need-to-know drug information in a very user friendly organizational style. Lippincott's Nursing Drug Guide (LNDG) has drug monographs organized alphabetically and includes nursing implications and patient teaching points. Numerous other drug handbooks are also on the market and readily available for nurses to use. Although other drug reference books such as Drug Facts and Comparisons, PDR, and drug package inserts can all provide essential drug information, they will not contain nursing implications and teaching points and can be more difficult to use than nurse's drug guides.

The nurse caring for a 38-year-old patient started on albuterol (Proventil) should advise the patient that he or she may experience what adverse effect? A) Polydipsia B) Tachycardia C) Hypotension D) Diarrhea

Ans: B Feedback: Adrenergic agents stimulate beta1 -adrenergic receptors in the heart as well as beta2 -adrenergic receptors in the lungs. Adrenergic agents do not cause polydipsia, hypotension, or diarrhea.

Which statement best describes drug efficacy/toxicity in pediatric patients? A) Drug requirements for infants have been extensively studied. B) Drug dosage is altered by age and weight in children. C) Children always need smaller doses of medication than adults. D) Infants and children are not at risk for drug toxicity if the dosage is correct.

Ans: B Feedback: All aspects of pediatric drug therapy must be guided by the child's age, weight, and level of growth and development. Drugs are generally studied using healthy young men and are never studied in infants because they are not old enough to give consent for themselves. Drug dosages are very specific and a big child may weigh more than an adult and require a larger dose. Even when medications are given accurately, adverse effects can occur.

When taking a medication history on a patient why should the nurse ask about the use of complementary or alternative therapies? A) Patients starting on new drugs are usually not compliant with medical regimens. B) Many drug-alternative therapy interactions can cause serious problems. C) Natural products may be more effective and the prescribed drug may not be needed. D) The cost of the drug and the alternative therapy may be too expensive for the patient to handle.

Ans: B Feedback: Alternative therapies often involve the use of herbal products, which contain natural chemicals that affect the body. Many drug-alternative therapy interactions have been reported that could cause serious adverse effects, but patients often don't think to mention these therapies when asked about the medications they are taking. The health care provider needs to be alert to these possible interactions and to adjust treatment appropriately. Cost and effectiveness may be factors, but the balancing of these therapies in the drug regimen to prevent interactions is the main concern of the nurse.

A patient with seasonal allergies is taking an antihistamine to relieve itchy, watery eyes, and a runny nose. When planning teaching for this patient, the nurse would include what teaching point? A) Advise the patient to limit fluid intake to dry out mucous membranes. B) Advise the patient to avoid driving or operating machinery. C) Advise the patient to report strange dreams or nightmares. D) Advise the patient to decrease dietary fat.

Ans: B Feedback: An adverse effect of antihistamines is drowsiness, so that injury to the patient or others can occur if driving or operating machinery. An increase in fluids would be indicated to help keep nasal membranes moist. It is common for dreams to occur when taking medication and it is not necessary to report them. Dietary fat should not interfere with the drug metabolism of antihistamines.

A 76-year-old man with asthma is being treated with an anticholinergic. What will the nurse be careful to assess for? A) Cardiac arrhythmias B) Prostatic hypertrophy C) Thyroid conditions D) Parkinsonism

Ans: B Feedback: Anticholinergics can produce urinary hesitancy and urinary retention, conditions that would aggravate the signs and symptoms of prostatic hypertrophy. Older patients given anti-cholinergics should be encouraged to empty the bladder before taking the drug. These drugs are used to treat parkinsonism. Thyroid conditions and cardiac arrhythmias are not cautions or contraindications to the use of these drugs.

A patient presents at the clinic with a dry nonproductive cough. The patient is diagnosed with bronchitis and it has been determined that assistance is needed in thinning the sputum so the cough can become productive. What does the nurse expect the physician will prescribe? A) Benzonatate (Tessalon) B) Guaifenesin (Mucinex) C) Dextromethorphan (Benylin) D) Hydrocodone (Hycodan)

Ans: B Feedback: Because this patient needs to cough up respiratory secretions, he would likely be prescribed guaifenesin. This drug is an expectorant that liquefies lower respiratory secretions by reducing their viscosity and so making it easier for a patient to cough them up. Benzonatate, dextromethorphan, and hydrocodone are antitussives and are given to suppress the cough reflex

A nurse is instructing a pregnant patient concerning the potential risk to her fetus from a Pregnancy Category B drug. What would the nurse inform the patient? A) Adequate studies in pregnant women have demonstrated there is no risk to the fetus. B) Animal studies have not demonstrated a risk to the fetus, but there have been no adequate studies in pregnant women. C) Animal studies have shown an adverse effect on the fetus, but there are no adequate studies in pregnant women. D) There is evidence of human fetal risk, but the potential benefits from use of the drug may be acceptable despite potential risks

Ans: B Feedback: Category B indicates that animal studies have not demonstrated a risk to the fetus. However, there have not been adequate studies in pregnant women to demonstrate risk to a fetus during the first trimester of pregnancy and no evidence of risk in later trimesters. Category A indicates that adequate studies in pregnant women have not demonstrated a risk to the fetus in the first trimester or in later trimesters. Category C indicates that animal studies have shown an adverse effect on the fetus, but no adequate studies in humans. Category D reveals evidence of human fetal risk, but the potential benefits from the use of the drugs in pregnant women may outweigh potential risks

A patient is taking chloroquine (Aralen) for rheumatoid arthritis. What problem reported by the patient would the nurse suspect may be an adverse reaction of the medication? A) I have to urinate all the time. B) Sometimes I have blurred vision. C) I have tingling in my arms and legs. D) Sometimes I feel like I am off balance.

Ans: B Feedback: Chloroquine (Aralen) can cause ocular toxicity with blurring of vision, color vision changes, corneal damage, and blindness. Increased urination, tingling, and numbness are signs of hyperkalemia and hypokalemia. Loss of balance can be caused by auditory damage due to drug toxicity.

What factor influences drug absorption? A) Kidney function B) Route of administration C) Liver function D) Cardiovascular function

Ans: B Feedback: Drug absorption is influenced by the route of administration. IV administration is the fastest method; drug absorption is slower when given orally. Kidney function impacts excretion, liver function impacts metabolism, and cardiovascular function impacts distribution.

The nursing instructor is discussing bronchodilators with a group of nursing students. The students understand the instruction when they identify what drug is most effective in treating acute bronchospasm? A) Ipratropium bromide (Atrovent) B) Epinephrine (Adrenalin) C) Cromolyn (Intal) D) Ephedrine

Ans: B Feedback: Epinephrine may be injected subcutaneously in an acute attack of bronchoconstriction, with therapeutic effects in 5 minutes that last 4 hours. It is considered the drug of choice for the treatment of acute bronchospasm. Ipratropium bromide has an onset of action of 15 minutes when inhaled with a duration of 3 to 4 hours. Cromolyn is not for use during acute times of bronchospasm but is used to help prevent bronchospasm. Ephedrine can be used in acute bronchospasm but epinephrine remains the drug of choice.

What action by the patient would indicate that the patient understands how to use an inhaler? A) The patient inhales as soon as the inhaler enters his or her mouth. B) The patient holds his or her breath for several seconds after releasing the medication. C) The patient administers three doses of medication within a 1-minute time frame. D) The patient exhales as soon as he or she compresses the inhaler.

Ans: B Feedback: Holding the breath prevents exhalation of medication still remaining in the mouth. The patient should inhale when the canister is compressed, not as soon as the inhaler enters his or her mouth. The patient should only administer one dose of medication at a time and the patient should wait to exhale until after the breath has been held as long as possible.

A nurse is preparing a drug for administration to a patient. The drug does not have an indicated use for the patient's medical diagnosis. What should the nurse do? A) Administer the drug as ordered. B) Question the prescriber concerning the ordered drug. C) Ask a coworker his or her thoughts about the ordered drug for the patient. D) Ask the patient why the drug has been prescribed for him or her.

Ans: B Feedback: If the nurse is not sure about giving a drug, the order should be questioned. The nurse should never give a medication that is not clear. Mistakes do happen and the drug ordered, if not approved for the condition that the patient has, could be an error on someone's part. The person who wrote the order should be questioned, not a co worker, who probably does not know why an off-label drug is being used. It would be unprofessional and inappropriate to ask the patient about the drug.

A 29-year-old female patient has sinusitis, so the physician orders a topical nasal decongestant. What instructions should be given? A) Avoid becoming pregnant during decongestant therapy. B) Increase fluids to 2 L/d. C) Restrict fluids to 500 mL/d. D) Take the medication with meals.

Ans: B Feedback: Institute other measures to help relieve the discomfort of congestion (e.g., humidity, increased fluid intake, cool environment, avoidance of smoke-filled areas) as appropriate. The medication does not need to be taken with meals or to restrict fluids. It would be inappropriate to tell the patient to avoid becoming pregnant.

The nurse receives a call from a frantic mother saying, My child swallowed some of my birth control pills. Should I give Ipecac? What is the nurse's best response? A) Yes, give Ipecac and follow the dosage directions on the bottle. B) Ipecac is not effective for this use so you should not give it to your child. C) Give the Ipecac only if you are absolutely sure your child swallowed the pills. D) No, don't give Ipecac because it will cause your child to vomit and make a mess.

Ans: B Feedback: Ipecac is a drug that the Food and Drug Administration tested in 2003 and found, despite its use for many years, that it was not effective in inducing vomiting in children suspected of poisoning. As a result, it is no longer used. The mother should be instructed not to give it and to call poison control to get up-to-date instructions on how to deal with this emergency. Whether the pills were swallowed, this child requires appropriate intervention because it is better to err on the side of caution. Making a mess is not a concern.

A patient calls the clinic and asks to speak to a nurse. The patient questions the nurse about the use of a drug that was advertised on TV. The patient tells the nurse he or she is sure that the drug will make him or her feel the same way as described in the commercial. What response is most appropriate for the nurse to make? A) I'm glad that you want to be involved in treatment decisions but you are not qualified to decide what medications are best for your condition. B) It's important to remember that drug advertisements emphasize the positive effects of drug therapy and not the adverse effects or contraindications. C) You need to remember that the drugs being advertised are much more expensive than other drugs that have the same effect. D) I've seen those advertisements and I would want to take that medication too if I had the condition it was designed to treat.

Ans: B Feedback: It would be important for the nurse to remind the patient that advertisements always emphasize the positive effects of drug therapy. The patient should not be discouraged from contributing to the plan of care by being told she is not adequately qualified to make decisions because no one is more qualified to make decisions about her own body. Although the drug may be more expensive, this is not a reason to choose or avoid a medication that could be more effective. Agreeing with the patient is not meeting the nurse's obligation to teach and inform.

Before administering a macrolide antibiotic the nurse would question the order for what patient? A) An 82-year-old woman with hypertension B) A 12-year-old boy with hearing loss C) A 30-year-old pregnant patient D) A 51-year-old man after myocardial infarction

Ans: B Feedback: Macrolide antibiotics can cause severe auditory nerve damage so the nurse would question administration of this drug to the child with hearing loss because another antibiotic may be indicated to preserve remaining hearing. This drug is not contraindicated in older adults, although a lower dosage may be indicated. It may safely be given in pregnancy and after myocardial infarction.

When patients do not understand the information provided with their medication, whose responsibility is it to help them sort through and comprehend the meaning? A) Care giver B) Nurse C) Patient D) Physician

Ans: B Feedback: Many pharmacies provide written information with each drug that is dispensed, but trying to organize these sheets of information into a usable and understandable form is difficult for many patients. The nurse is often the one who needs to sort through the provided information to organize, simplify, and make sense of it for the patient.

A patient is admitted to the intensive care unit with hyperkalemia. The nurse caring for the patient knows that the most severe adverse effect of hyperkalemia is what? A) Renal failure B) Cardiac emergency C) Liver failure D) Respiratory distress

Ans: B Feedback: Monitor for cardiac irregularities because potassium is an important electrolyte in the action potential, needed for cell membrane stability. When potassium levels are too high, the cells of the heart become very irritable and rhythm disturbances can occur. Be prepared for a possible cardiac emergency. Hyperkalemia is often found in the patient with renal failure. Liver failure and respiratory distress are not generally caused by hyperkalemia.

A physician has ordered intramuscular (IM) injections of morphine, a narcotic, every 4 hours as needed for pain in a motor vehicle accident victim. The nurse is aware this drug has a high abuse potential. Under what category would morphine be classified? A) Schedule I B) Schedule II C) Schedule III D) Schedule IV

Ans: B Feedback: Narcotics with a high abuse potential are classified as Schedule II drugs because of severe dependence liability. Schedule I drugs have high abuse potential and no accepted medical use. Schedule III drugs have a lesser abuse potential than II and an accepted medical use. Schedule IV drugs have low abuse potential and limited dependence liability.

The home health nurse is caring for a 77-year-old patient with congestive heart failure. What would the nurse consider a priority to assess to develop the most effective plan of care related to medication administration? A) Description of the patient's living environment B) Required lifestyle changes C) Family members in the community D) Compliance with therapy to reduce risk of skin breakdown

Ans: B Feedback: Nurses must consider how a person responds to disease and its treatment, including the changes in lifestyle that may be required. By recognizing required lifestyles during the home visit the nurse can teach the patient how to make healthy choices and support the process of changing to new choices. Although the environment would impact the physical plan of care, it would not be a factor in the administration of medications. Assessment of family members in the community would not be a necessary part of the assessment in relation to the patient's drug therapy. Nothing suggests the patient is at risk for skin breakdown so this would not factor into the medication regimen.

The nurse has admitted a patient (who takes ipratropium) to the respiratory unit with an acute exacerbation of chronic obstructive pulmonary disease (COPD). While writing a plan of care for this patient, what would be the most appropriate nursing diagnosis to use? A) Deficient knowledge regarding alternative therapy B) Imbalanced nutrition: Less than body requirements C) Acute pain related to renal effects of the drug D) Disturbed thought processes related to central nervous system (CNS) effects

Ans: B Feedback: Nursing diagnoses related to drug therapy might include acute pain related to CNS, gastrointestinal (GI), or respiratory effects of the drug; imbalanced nutrition: Less than body requirements, related to dry mouth and GI upset; and deficient knowledge regarding drug therapy. Options A, C, and D are not correct.

The nurse is writing a care plan for a patient who has been prescribed a nasal steroid. What would be an appropriate nursing diagnosis for this patient? A) Disturbed sensory perception (kinesthetic) related to CNS effects B) Risk for injury related to suppression of inflammatory reaction C) Ineffective airway clearance related to bronchospasm D) Ineffective airway related to nasal obstruction

Ans: B Feedback: Nursing diagnoses related to drug therapy might include acute pain related to local effects of the drug, risk for injury related to suppression of inflammatory reaction, and deficient knowledge regarding drug therapy. Nursing diagnosis for this patient does not include disturbed sensory perception, ineffective airway clearance, or ineffective airway.

A nurse working in radiology administers iodine to a patient who is having a computed tomography (CT) scan. The nurse working on the oncology unit administers chemotherapy to patients who have cancer. At the Public Health Department, a nurse administers a measles-mumps-rubella (MMR) vaccine to a 14-month-old child as a routine immunization. Which branch of pharmacology best describes the actions of all three nurses? A) Pharmacoeconomics B) Pharmacotherapeutics C) Pharmacodynamics D) Pharmacokinetics

Ans: B Feedback: Pharmacology is the study of the biologic effects of chemicals. Nurses are involved with clinical pharmacology or pharmacotherapeutics, which is a branch of pharmacology that deals with the uses of drugs to treat, prevent, and diagnose disease. The radiology nurse is administering a drug to help diagnose a disease. The oncology nurse is administering a drug to help treat a disease. Pharmacoeconomics includes any costs involved in drug therapy. Pharmacodynamics involves how a drug affects the body and pharmacokinetics is how the body acts on the body

The nurse is acting as the triage nurse in the emergency department when a 16-year-old boy is brought in by his friends. The patient is in respiratory distress, he is vomiting, and blood is noted in the vomitus. He is somnolent and his electrocardiogram shows an arrhythmia. The boy's friends tell you he was taking a bunch of little green pills he got from the cupboard at his grandparent's house. The nurse suspects what? A) Overdose of sleeping pills B) Poisoning C) Anaphylactic shock D) Allergic reaction to Dyazide

Ans: B Feedback: Poisoning occurs when an overdose of a drug damages multiple body systems, leading to the potential for fatal reactions. The situation described does not indicate what was contained in the little green pills so it is not possible to say whether the drugs were sedatives or triamterene and hydrochlorothiazide (Dyazide). The symptoms do not indicate an anaphylactic reaction, which would not normally include bloody vomitus.

A patient has an acute asthma attack. A bronchodilator is used to bring the exacerbation under control. What drug would be used to prevent acute bronchoconstriction? A) Salbutamol (Ventolin) B) Salmeterol (Serevent) C) Fenoterol (Berotec) D) Terbutaline (Bricanyl)

Ans: B Feedback: Salmeterol (Serevent) and formoterol (Foradil) and are long-acting beta2 -adrenergic agonists used only for prophylaxis of acute bronchoconstriction. Options A, C, and D are all short-acting beta2 -adrenergic agonists and not used for prophylaxis.

What classification of drug allergy would be described as an immune system reaction to injected proteins used to treat immune conditions? A) A cytotoxic reaction B) Serum sickness C) A delayed reaction D) An anaphylactic reaction

Ans: B Feedback: Serum sickness is an immune system reaction to certain medications, injected proteins used to treat immune conditions, or antiserum, the liquid part of blood that contains antibodies that help protect against infectious or poisonous substances. An allergic reaction can occur with any drug, not just those containing protein. Anaphylaxis is an acute, systemic, life-threatening allergic reaction. A cytotoxic reaction is one in which antibodies circulate and attack antigens on cell sites, causing death of that cell.

The nurse is caring for a patient receiving an antineoplastic medication who reports fever, chills, sore throat, weakness, and back pain. What type of adverse effect does the nurse suspect this patient is experiencing? A) Dermatologic reaction B) Blood dyscrasia C) Electrolyte imbalance D) Superinfection

Ans: B Feedback: Symptoms of blood dyscrasias include fever, chills, sore throat, weakness, back pain, dark urine, decreased hematocrit (anemia), low platelet count (thrombocytopenia), low white blood cell count (leukopenia), and a reduction of all cellular elements of the complete blood count (pancytopenia). Dermatologic reactions would be reflected in skin alterations, electrolyte imbalances would result in differing symptoms depending on the electrolyte involved but would not cause chills and fever, and a superinfection could cause a fever but would not cause a sore throat, weakness, or back pain unless the infection involved those body parts.

A renal failure patient with severe hyperkalemia (K+ level 7.2 mEq/L) has just been admitted to the nursing unit. Given the severity of this situation, the nurse should be prepared to administer which intravenous infusion stat? A) Lactated Ringer solution at 150 mL/hour to maintain blood glucose levels B) Regular insulin infusion, rate dependent on lab values C) Infusion of Solu-Medrol to decrease irritation to the intravascular system D) Dilaudid via patient-controlled device (PCA) to control pain

Ans: B Feedback: The administration of sodium bicarbonate,-adrenergic agonists, or insulin distributes potassium into the ICF compartment and rapidly decreases the ECF concentration. Lactated Ringer solution, steroids, or narcotics will not help to lower potassium levels.

A nurse in a three drops with respiratory problems has received orders for four patients. What medication order will the nurse question? A) Tetrahydrozoline (Tyzine) three drops in each nostril for an 8-year-old child B) Hydrocodone (Hycodan) 10 mg PO for a 5-year-old child C) Pseudoephedrine (Dorcol) 15 mg PO for a 2-year-old child D) Diphenhydramine (Benadryl) 5 mg/kg IM for a 10-year-old child

Ans: B Feedback: The nurse should question the order for hydrocodone. The ordered dose, 10 mg, is an adult dose and should not be given to a 5-year-old. Patients from 2 to 12 years of age should be given between 1.25 and 5 mg/dose. The other medications are all correct dosages for the age of the child for whom they were prescribed.

Today, an abundance of information is available in the health care arena for consumers, resulting in the nurse encountering patients who have a much greater use of what? A) Over-the-counter (OTC) therapies B) Alternative therapies C) Prescription drugs D) Off-label drugs

Ans: B Feedback: The patient now comes into the health care system burdened with the influence of advertising, the Internet, and a growing alternative therapy industry. Many patients no longer calmly accept whatever medication is selected for them. Indeed, an increasing number of patients are turning to alternative therapies with the belief that they will treat their disorder and reduce risk of adverse effects. Although more prescription drugs are used today, that is not related to abundant information. No indication exists of an increase in use of OTC or off-label drugs.

An adolescent is admitted to the intensive care unit with diabetic ketoacidosis. The nurse prepares a continuous insulin infusion of 100 units (U) regular insulin in 500 milligram normal saline. When documenting this medication, how many units of regular insulin will this patient receive per milligram of IV solution? A) 0.175 U/milligram B) 0.2 U/milligram C) 0.25 U/milligram D) 0.5 U/milligram

Ans: B Feedback: The problem tells us there is 100 U/500 milligram. To determine how many units are in each milligram, divide both numbers by 500: 100 U ÷ 500/500 milligram÷ 500 = 0.2 U/1 milligram.

A patient is experiencing pain, so the physician orders codeine ½ grain every 4 hours. How many milligrams of codeine would the nurse administer? A) 15 mg B) 30 mg C) 60 mg D) 120 mg

Ans: B Feedback: The simplest way to convert measurements from one system to another is to set up a ratio and proportion equation. The ratio containing two known equivalent amounts is placed on one side of an equation, and the ratio containing the amount you wish to convert and its unknown equivalent is placed on the other side. 60 mg/1 grain = flexion range of motion. Cross-multiplying yields 30 mg.

A patient is in the clinic to have blood drawn to assess his or her theophylline level. The patient is taking theophylline (Theo-Dur) and appears to being doing well on the drug. He or she reports no problems. What serum level will the nurse expect the patient to have? A) Between 0.5 to 5 mcg/mL B) Between 10 to 20 mcg/mL C) Between 25 to 35 mcg/mL D) Between 40 to 50 mcg/mL

Ans: B Feedback: Therapeutic theophylline levels should be between 10 and 20 mcg/mL. A level between 0.5 and 5 mcg/mL would be low and would not produce a therapeutic effect. Levels between 25 and 50 mcg/mL would be too high and could cause serious adverse effects.

A 35-year-old male patient is admitted to the hospital with pneumonia. He was originally being treated at home, but became worse when he quit taking his antibiotic prematurely. What is an appropriate nursing diagnosis for this patient? A) Deficient knowledge: monitoring temperature B) Noncompliance C) Risk for injury related to hypoxia D) Non-adherence: overuse

Ans: B Feedback: This patient did not take his antibiotic as directed. He quit taking his medication too soon, probably when he began to feel better so he was noncompliant with care. Why he quit taking the medication is unknown; it could have been a knowledge deficit or inability to pay for the full prescription. This will require further assessment to determine. Further data would need to be collected to determine whether the patient is experiencing hypoxia. He did not overuse his medication so option D is inaccurate.

An 80-year-old patient with internal bleeding is admitted through the emergency room after a motor vehicle accident. The physician has ordered 2 units of packed red blood cells (1 unit is 250 mL) to infuse over 1 hour each. The drip rate on the blood administration set is10 gtt/mL. The nurse administers how many drops per minute to infuse the blood as ordered? A) 47 B) 42 C) 37 D) 32

Ans: B Feedback: Use the following ratio to determine how many drops of fluid to administer per minute: Using the information from this problem: Because it is not possible to deliver 0.7 of a drop, round 41.7 to 42 minutes

A nurse is administering digoxin to a patient. To administer medications so that the drug is as effective as possible, the nurse needs to consider what? A) Pharmacotherapeutics B) Pharmacokinetics C) Pharmacoeconomics D) Pharmacogenomics

Ans: B Feedback: When administering a drug, the nurse needs to consider the phases of pharmacokinetics so that the drug regimen can be made as effective as possible. Pharmacogenomics is the area of study that includes mapping of the human genome. Pharmacotherapeutics is the branch of pharmacology that deals with the uses of drugs to treat, prevent, and diagnose disease. Pharmacoeconomics includes all costs involved in drug therapy.

A patient has orders to receive 2 L of IV fluid over a 24-hour period with ½ this amount to be infused in the first 10 hours of treatment. How many milliliters per hour will the nurse administer during the first 10 hours of the infusion? A) 50 mL/h B) 100 mL/h C) 83 mL/h D) 200 mL/h

Ans: B Feedback: ½ of the 2 liters is to infuse in the first 10 hours. ½ of 2 = 1 L; 1 L = 1,000 mL to infuse over 10 hours = 100 mL/L.

The nurse teaches a young mother the importance of administering appropriate dosages of acetaminophen (Tylenol) and determines further teaching is needed when the mother makes what statement? A) The children's dosage will change with time as they grow. B) My baby's dose of Tylenol is about 1 half an adult dose. C) It is important to give the right dose to prevent toxic effects of the medication. D) My children's dose of Tylenol should be based on their weight or age.

Ans: B Feedback: A child's dose is never based on an adult's dose. A child's dosage is based on weight and age and will change with age as they grow. Larger than directed dosages can result in toxic effects of this medication

The patient looks at the prescription provided by the doctor and asks the nurse whether he can request a generic substitution. The nurse answers No when noting what on the prescription? A) No refills B) DAW C) Brand name used on prescription D) Patient older than 65 years of age

Ans: B Feedback: DAW stands for dispense as written and means that the doctor does not want a generic substituted for the prescribed medication. Requesting no refills does not preclude the substitution of a generic medication. Even when the brand name is ordered, the pharmacist can substitute a generic equivalent so long as the prescriber does not write DAW. Generic substitutions are not impacted by the patient's age.

The nurse has provided health teaching for a 15-year-old boy newly diagnosed with asthma. What statement, made by the patient, indicates that he has a good understanding of the teaching the nurse has done regarding inhalers? A) I should hold my breath when administering a puff. B) The aerosol canister should be shaken well before using. C) I need to take three short quick breaths when I administer the inhaler. D) A second aerosol medication cannot be administered until 30 minutes after the first aerosol medication.

Ans: B Feedback: Inhalers should be shaken well, immediately before each use. It would not be appropriate to teach the patient to hold his breath when administering a puff, to take three short quick puffs when administering the inhaler, or that a second aerosol medication should not be administered until 30 minutes after the first dose of aerosol medication.

The nurse applies the nursing process in medication therapy to ensure what? A) That medications are given at the right time B) That care is efficient and effective C) That the right dose of the drug is given to the patient D) That the right drug is given to the right patient at the right time

Ans: B Feedback: Nurses use the nursing process as a decision-making, problem-solving process to improve the efficiency and effectiveness of care. Options A, C, and D are incorrect and do not describe why the nursing process is important as much as they explain how to give medications safely.

The nurse is teaching a diabetic patient to self-administer Humulin insulin, supplied in a vial labeled 100 units/mL. The provider has ordered 32 units of Humulin insulin to be taken each morning. How many mL of insulin would the patient prepare for one dose? A) 0.032 mL B) 0.32 mL C) 3.2 mL D) 0.64 mL

Ans: B Feedback: There are 100 units in each mL. Divide that amount by 32 units for the answer (0.32 mL).

The nurse is conducting an admission assessment on a patient. When collecting data related to medications the nurse asks What medications are you currently taking? After collecting this information, what other questions should the nurse ask? (Select all that apply.) A) Do you take any medications? B) What over-the-counter (OTC) medications do you take? C) Do you take an herbs, vitamins, or supplements? D) Do you take medications safely when you take them? E) Why do you take this medication?

Ans: B, C Feedback: Patients often neglect to mention OTC drugs or alternative therapies (e.g., herbals) because they do not consider them to be actual drugs or they may be unwilling to admit their use to the health care provider. Ask patients specifically about OTC drug and alternative therapy use. The question What medications are you taking? has already been asked so there is no need to ask if they take any medications. Often, patients may take medications unsafely, so do not ask whether the patient takes drugs safely but instead assess exactly how they take medications to determine whether they are being taken safely. Patients should always know why they are taking each medication to understand whether they are getting the desired effect.

A patient has come to the clinic and been diagnosed with Lyme disease. The physician has ordered oral tetracycline. What is important for the nurse to include in the teaching plan about tetracycline? (Select all that apply.) A) Do not take the drug with anything high in sodium content to keep from producing a state of hypernatremia in the body. B) Do not take the drug with foods or other drugs that contain calcium. C) Do not take the drug at the same time you take an iron supplement or with foods that are high in iron content. D) Avoid exposure to the sun when taking this drug as it can turn your skin purple. E) Avoid eating bananas at the same time you take this drug as the potassium content of the tetracycline can produce hyperkalemia in the body.

Ans: B, C Feedback: The antibiotic tetracycline is not absorbed from the gastrointestinal (GI) tract if calcium or calcium products (e.g., milk) are present in the stomach. It cannot be taken with iron products because a chemical reaction occurs preventing absorption. Although tetracycline can increase sun sensitivity, it does not turn the skin purple. Patients who take tetracycline do not need to avoid eating bananas or foods that are high in potassium.

The nurse is caring for a patient who had a severe, acute, previously unseen adverse effect of a drug in Phase III testing. The patient asks, After all the testing done on this drug, didn't they know this adverse effect could occur? What is the nurse's best response? (Select all that apply.) A) Pharmaceutical companies sometimes underreport problems to make more money. B) Your response to this medication will be reported to the drug company and the Food and Drug Administration (FDA). C) When a drug begins to be used by a large clinical market, new adverse effects may be found. D) The pharmaceutical company weighs the benefits of the drug with the severity of adverse effects. E) After a drug reaches phase III testing it is considered an accepted drug and will not be recalled.

Ans: B, C Feedback: When a new and unexpected adverse effect occurs, especially one of a serious nature, it is reported to the drug company who reports it to the FDA immediately. When a large number of people begin using the drug in phase III studies, it is not unusual to identify adverse effects not previously noted. It would be both unprofessional and inaccurate to imply that pharmaceutical companies put profit ahead of patient concern because lawsuits would remove any potential profit if a drug proves harmful. The FDA is responsible for weighing risk versus benefit in deciding whether to allow the drug to move to the next phase of testing. Drugs found to have serious adverse effects can be removed from the market at any time.

The nurse administers an intravenous medication with a half-life of 24 hours but recognizes what factors in this patient could extend the drug's half-life? (Select all that apply.) A) Gastrointestinal disease B) Kidney disease C) Liver disease D) Cardiovascular disease E) Route of administration

Ans: B, C, D Feedback: Kidney disease could slow excretion and extend the drug's half-life. Liver disease could slow metabolism resulting in an extended half-life. Cardiovascular disease could slow distribution resulting in a longer half-life. Gastrointestinal disease would not impact half-life because the medication was injected directly into the bloodstream. Route of administration would not extend half-life because IV injection eliminates the absorption step in the process.

Federal guidelines state that when advertising a drug, if the company states what the drug is used for, what other information must also be included in the advertisement? (Select all that apply.) A) Symptoms B) Contraindications C) Adverse effects D) Precautions E) Cost

Ans: B, C, D Feedback: If a drug advertisement states what the drug is used for, it must also state contraindications, adverse effects, and precautions. The advertisement does not have to state symptoms or cost.

Because of the amount of care now being done in the home care setting, it is imperative that the nurse teach the patients what? (Select all that apply.) A) Care givers educational level. B) Generic names of medication C) Over-the-counter (OTC) drugs that need to be avoided D) Alleviation of adverse effects E) How to calculate safe dosages

Ans: B, C, D Feedback: The responsibility of meeting the tremendous increase in teaching needs of patients frequently resides with the nurse. Patients need to know exactly what medications they are taking (generic and brand names), the dose of each medication, and what each is supposed to do. Patients also need to know what they can do to alleviate some of the adverse effects that are expected with each drug (e.g., small meals if gastrointestinal upset is common, use of a humidifier if secretions will be dried and make breathing difficult), which OTC drugs or alternative therapies they need to avoid while taking their prescribed drugs, and what to watch for that would indicate a need to call the health care provider.

The nurse is caring for a child who has been newly diagnosed with asthma. What environmental modifications should the nurse encourage the parents to make to help their child avoid future attacks? (Select all that apply.) A) Make sure the child begins herbal therapy as soon as possible. B) Avoid crowded areas as much as possible. C) Keep the child away from any known allergens. D) Encourage the child to use a broom to sweep the bedroom every day. E) Keep the child away from areas that are filled with cigarette smoke.

Ans: B, C, E Feedback: Parents need to be encouraged to take measures to prevent acute attacks, including avoidance of known allergens, smoke-filled rooms, and crowded or dusty areas. OTC drugs and herbal remedies should be avoided if possible. The child should not be sweeping the bedroom because this will produce a lot of dust.

The nursery has just admitted a new infant born 1 hour ago. While performing an assessment, the nurse suspects the infant may have hypoglycemia based on which of the following assessment data? Select all that apply. A) Heel stick glucose value of 50 mg/dL B) Infant having periods of apnea requiring physical stimulation C) Muscle twitching noted while lying in crib undisturbed by nurses D) Hyperactive reflexes noted especially when crying E) Poor suck reflex resulting in an inability to feed properly

Ans: B, C, E Feedback: In neonates, glucose levels stabilize to a value of 50 mg/dL or higher within the first 3 hours of life. Concentrations below 45 mg/dL should be considered abnormal. Signs and symptoms of neonatal hypoglycemia include cyanosis, apnea, hypothermia, hypotonia, poor feeding, lethargy, and seizures.

A 14-year-old boy has experienced a pronounced growth spurt over the last several months. While discussing this with his parents, the nurse educates what normal male growth patterns contain. Of the following, which are accurate statements to relay to the parents? Select all that apply. A) Most males will complete their growth spurt by age 16. B) It is not usual for their son to gain up to 30 kg in weight. C) With parathyroid hormone involvement, your son may be at risk for fractures . D) Expect the thorax to become broader and for the pelvis to remain narrow. E) Some children have stunted growth in their arms or legs.

Ans: B, D Feedback: In males, they may continue to gain height until 18 to 20 years of age and gain from 7 to 30 kg of weight. Parathyroid hormone does not have roles that relate to the adolescent growth spurt. In males, the thorax becomes broader and the pelvis remains narrow. In girls, the opposite occurs. Growth in the arms, legs, hands, feet, and neck is followed by increases in the hip and chest months later.

Place the following stages of the hydrogen ion elimination and bicarbonate conservation in the proximal tubules of the nephrons in the ascending chronological order. Use all the options. A) CO2 and H2O are produced. B) H+ is secreting into the tubular fluid. C) Carbonic acid is produced. D) H+ combines with filtered HCO3-.

Ans: B, D, C, A Feedback: Renal regulation of pH involves the secretion of hydrogen ions into the tubular fluid, the combining of hydrogen ions with bicarbonate yielding carbonic acid, followed by the decomposition of carbonic acid into carbon dioxide and water.

As a consequence of a long-standing lung disease, a client is in a chronic state of hypoxia. Which of the following phenomena would the client's care team be most justified in anticipating? Select all that apply. A) Metabolic alkalosis B) Increased erythropoietin production C) Pulmonary vasodilation D) Hyperventilation E) Personality changes

Ans: B, D, E Feedback: Increased production of erythropoietin, hyperventilation, and cognitive and personality changes are all associated with hypoxemia. Acidosis, not alkalosis, and vasoconstriction rather than vasodilation are likely to occur.

Upon admission to the ICU, a patient with a history of cor pulmonale will likely be exhibiting which of the following clinical manifestations of right heart failure? Select all that apply. A) Fine crackles throughout both lung fields B) +4 pitting edema in lower extremities C) Expectorating copious amounts of frothy, pink sputum D) Jugular vein distension E) Altered level of consciousness

Ans: B, D, E Feedback: Signs of right-sided HF include venous congestion (jugular vein distension), peripheral edema (+4 pitting edema in feet), shortness of breath, and productive cough. Altered level of consciousness may occur as the result of carbon dioxide retention. Fine crackles in all lung fields and frothy, pink sputum are common in left-sided HF.

The nursing instructor is discussing the use of sympathomimetics in patients who have acute bronchospasm. The instructor shares with the students that this classification of drugs is contraindicated or only used with great caution in patients with what disorders? (Select all that apply.) A) Hypothyroidism B) Cardiac disease C) Kidney disease D) Diabetes mellitus E) Peripheral vascular disease

Ans: B, D, E Feedback: Before administering a sympathomimetic the nurse should assess for possible contraindications or cautions that include any known allergies to any drug in this class, cigarette use, cardiac disease, vascular disease, arrhythmias, diabetes, and hyperthyroidism. Sympathomimetics have no known adverse effects in hypothyroidism or kidney disease.

Which of the following residents of a long-term care facility is most likely to be exhibiting the signs and symptoms of chronic obstructive pulmonary disease (COPD)? A) A 79-year-old lifetime smoker who is complaining of shortness of breath and pain on deep inspiration B) An 81-year-old smoker who has increased exercise intolerance, a fever, and increased white blood cells C) An 81-year-old male who has a productive cough and recurrent respiratory infections D) An 88-year-old female who experiences acute shortness of breath and airway constriction when exposed to tobacco smoke

Ans: C Feedback: Productive cough and recurrent respiratory infections are associated with COPD, while pain, fever, and increased white cells are not common signs and symptoms of COPD. Acute shortness of breath and bronchoconstriction are associated with asthma.

Drugs do not metabolize the same way in all people. For what patient would a nurse expect to assess for an alteration in drug metabolism? A) A 35-year-old woman with cervical cancer B) A 41-year-old man with kidney stones C) A 50-year-old man with cirrhosis of the liver D) A 62-year-old woman in acute renal failure

Ans: C Feedback: The liver is the most important site of drug metabolism. If the liver is not functioning effectively, as in patients with cirrhosis, drugs will not metabolize normally so that toxic levels could develop unless dosage is reduced. A patient with cervical cancer or kidney stones would not be expected to have altered ability to metabolize drugs so long as no liver damage existed. The patient with renal failure would have altered excretion of the drugs through the renal system but metabolism would not be impacted.

In the early morning, an African American woman brings her 5-year-old son to the emergency room. The boy is wheezing, is short of breath, and has a dry cough. The mother states that he has always been very healthy. He went to bed with only a slight cold and a runny nose but woke her with his coughing shortly after 4 AM. His symptoms worsened so dramatically that she brought him to the hospital. The care team would most likely suspect that he has A) respiratory syncytial virus. B) influenza. C) asthma. D) pneumonia.

Ans: C Feedback: Although the child may have an infectious disease, his symptoms and the timing of them (both in terms of his age and the time of symptom onset) are classic for asthma. They are not as closely associated with RSV, influenza, or pneumonia.

Which of the following clinical findings would be most closely associated with a client who has interstitial lung disease in comparison to chronic obstructive pulmonary disease (COPD)? A) Audible wheezing on expiration B) Reduced expiratory flow rates C) Decreased tidal volume D) Normal forced expiratory volume

Ans: C Feedback: Because it takes less work to move air through the airways at an increased rate than it does to stretch a stiff lung to accommodate a larger tidal volume, interstitial lung disease is commonly associated with an increased respiratory rate but decreased tidal volume. Wheezing and decreased expiratory flow rate are more closely associated with COPD.

A 66-year-old male presents to the emergency room accompanied by his wife who claims that he has been acting confused. The man is complaining of a sudden onset of severe weakness and malaise and has a dry cough and diarrhea. His temperature is 102.8°F, and his blood work indicates his sodium level at 126 mEq/L (normal 135 to 145 mEq/L). Based on this assessment, the nurse suspects the patient has A) bronchopneumonia. B) Mycoplasma pneumonia. C) Legionella pneumonia. D) pneumococcal pneumonia.

Ans: C Feedback: Confusion, dry cough, diarrhea, and hyponatremia are associated with Legionnaire disease and less so with bronchopneumonia, Mycoplasma pneumonia, or pneumococcal pneumonia.

A 14-year-old boy, appearing to be intoxicated, is brought to the emergency room by ambulance. The EMTs report that the boy has denied consuming anything out of the ordinary, but an open antifreeze container was found in the boy's room. Which of the following is likely to be used to treat the patient's symptoms? A) Gastric lavage B) Syrup of ipecac C) Fomepizole D) Sodium bicarbonate

Ans: C Feedback: Ethylene glycol is found in products ranging from antifreeze and deicing solutions to carpet and fabric cleaners. It tastes sweet and is intoxicating—the factors that contribute to its abuse potential. A lethal dose is approximately 100 mL. It is rapidly absorbed from the intestine, making treatment with either gastric lavage or syrup of ipecac ineffective. Fomepizole, with specific indications for ethylene glycol poisoning, was recently approved by the U.S. Food and Drug Administration.

A child with rhinosinusitis should be monitored for complications. Which of the following assessment findings would alert the nurse that a complication is developing? A) Purulent nasal discharge B) Temperature of 100.8°F C) Periorbital edema D) Complaints of headache

Ans: C Feedback: Expected s/s of acute viral rhinosinusitis include facial pain, headache, purulent nasal discharge, decreased sense of smell, and fever. Complications can lead to intracranial and orbital wall problems. Facial swelling over the involved sinus, abnormal extraocular movements, protrusion of the eyeball, periorbital edema, or changes in mental status may indicate intracranial complications.

A client with poorly controlled diabetes mellitus presents to the emergency department with suspected ketoacidosis. Which of the following diagnostic results would most likely confirm this diagnosis? A) Low O2 levels, increased anion gap, base excess B) High ammonia levels, decreased anion gap, high potassium C) Increased anion gap, base deficit D) Decreased anion gap, decreased urine ammonium level

Ans: C Feedback: Increased CO2 levels, an increased anion gap, and a base deficit are all associated with an acidotic state. Base excess, low oxygen, high potassium, high ammonia, and decreased anion gap would not suggest acidosis.

A short, nonsmoking 44-year-old male presents to the emergency room with left- sided chest pain and a cough. He states that the pain started abruptly and worsens with deep breathing and coughing. He denies recent injury. Assessment includes shallow respirations with a rate of 36, normal breath sounds, and no cyanosis. Which condition is most likely causing his symptoms? A) Myocardial infarction B) Spontaneous pneumothorax C) Pleuritis related to infection D) Obstructive atelectasis

Ans: C Feedback: Pleuritis, which frequently accompanies infections that cause cough, is unilateral, starts abruptly, and is worsened by coughing or deep breathing. The client's shallow, rapid breathing may be due to anxiety but also is a way of maintaining adequate air intake while avoiding deep breathing, which exacerbates the pain of pleuritis. His cough may be an indication of infection, especially as he is not a smoker. The pain of myocardial infarction is not worsened by deep breathing or coughing. Spontaneous pneumothorax would be very unlikely in a short, nonsmoking middle-aged man. Tachypnea might indicate obstructive atelectasis, but normal breath sounds and lack of cyanosis argue against it.

While assessing a premature infant born at 25 weeks' gestation, the neonatal intensive care unit (NICU) nurse would suspect which diagnosis when the infant displays poor muscle tone, apnea, and a new onset of somnolence? A) Hydrocephalus B) Airway obstruction C) Intraventricular hemorrhage D) Sepsis

Ans: C Feedback: Prematurity is a risk for IVH. Clinical manifestations are determined by the level of involvement. The most common symptoms are poor muscle tone, lethargy, apnea, decreased hematocrit, and somnolence.

Which of the following statements made by parents of high schoolers would be a cause for the concern the child may be thinking about suicide? A) "My child seems to eat all the time. He tells me that all of his friends are eating a lot as well." B) "My child seems to go shopping at the mall every day after school with her friends. I think they hang out at the mall." C) "My child has never had problems in school until now. He is failing classes and getting in trouble." D) "My child used to talk to me about anything. Now she spends most of her time in her room texting friends."

Ans: C Feedback: Risk factors for suicide in adolescents include substance abuse, personal or family history of depression, anxiety disorders, problems at school, problems communicating with parents, having a friend who committed suicide, and family ownership of a handgun.

Due to rapid neural growth, a child can begin to control the bowel and bladder sphincters by what age? A) 12 months B) 18 months C) 2 years D) 4 years

Ans: C Feedback: The cephalocaudal proximodistal principle is followed as myelinization of the cortex, brain stem, and spinal cord is completed. The spinal cord is usually completely myelinated by 2 years of age. At that time, control of anal and urethral sphincters and motor skills of locomotion can be achieved.

Following several days in an acidotic state, a hospital patient has returned to desired pH. Which of the following processes could have contributed to the resolution of the patient's health problem? A) Exchange of Na+ and H+ ions B) Selective renal secretion and reabsorption of CO2 C) The phosphate and ammonia buffer systems in the renal tubules D) Excretion of HCO3- by the kidneys

Ans: C Feedback: The phosphate and ammonia buffer systems are secondary, but important, processes that contribute to the maintenance of appropriate pH. The kidneys must reabsorb nearly all the body's stores of HCO3- to maintain homeostasis, and they are not involved in CO2 control. Hydrogen-potassium exchange, not hydrogen-sodium exchange, is a component of acid-base control.

The neonatal ICU nurse is aware that type II alveolar cells produce surfactant, and they usually develop at how many weeks of gestation? A) 17 to 18 weeks B) 19 to 20 weeks C) 24 to 28 weeks D) 34 to 38 weeks

Ans: C Feedback: Type II alveolar cells begin to develop at approximately 24 weeks. These cells produce surfactant, a substance capable of lowering the surface tension of the air-alveoli interface. By the 28th to 30th week, sufficient amounts of surfactant are available to prevent alveolar collapse when breathing begins.

A nurse is performing a 5-minute Apgar score on a newborn female. Which of the following characteristics of the infant's current condition would not be reflected in the child's Apgar score? A) The baby's heart rate is 122 beats/minute. B) The infant displays a startle reflex when the crib is accidentally kicked. C) The child's temperature is 35.0°C (95°F) by axilla. D) The infant's skin is pink in color.

Ans: C Feedback: While heart rate, color, and presence or absence of crying are all assessment criteria in the determination of an Apgar score, temperature is not a parameter that is measured.

Which of the following assessment findings of a male infant 14 hours postpartum would be considered abnormal and would require further assessment and possible intervention? A) The baby's first stool appears to contain blood. B) The child is unable to breathe through his mouth. C) The baby's skin has a yellowish orange hue. D) The child's suck is weak when placed at his mother's breast.

Ans: C Feedback: While not an uncommon event in early postnatal life, jaundice requires further assessment and possibly intervention. Meconium often contains blood, and young infants are exclusive nose breathers. A child's suck is frequently weak before it becomes established in the days to follow.

The nurse is talking with a group of nursing students who are doing clinical hours on the unit. A student asks if all intramuscular (IM) drugs are absorbed the same. What factor would the floor nurse tell the students to affect absorption of the IM administration of drugs? A) Perfusion of blood to the subcutaneous tissue B) Integrity of the mucous membranes C) Environmental temperature D) Blood flow to the gastrointestinal tract

Ans: C Feedback: A cold environmental temperature can cause blood vessels to vasoconstrict and decreases absorption or in a hot environment vasodilate and increase absorption of IM medications. Blood flow to the subcutaneous tissues interferes with subcutaneous injection and blood flow to the gastrointestinal (GI) tract causes alterations in absorption for oral medications. The condition of mucous membranes can interfere with sublingual (under the tongue) and buccal (in the cheek) administration of drugs.

How has the patient's access to drug information changed the way the patient interacts with the nurse and other health care providers? A) Patients share information from research reports with health care providers. B) Patients are contacting drug companies to see what their latest reports say. C) Patients are more likely to challenge the health care provider with their own research. D) Patients are more likely to self-prescribe and not obtain prescriptions from their health care provider

Ans: C Feedback: Access to consumer advertising, mass media health reports, and the Internet influence some patients to request specific treatments, to question therapy, and to challenge the health care provider. Consumers do not generally read research reports from medical facilities and contact drug companies to see what their reports say, and they cannot self-medicate because many of these drugs require a prescription to obtain them.

After completing a course on pharmacology for nurses, what will the nurse know? A) Everything necessary for safe and effective medication administration B) Current pharmacologic therapy; the nurse will not require ongoing education for 5 years. C) General drug information; the nurse can consult a drug guide for specific drug information. D) The drug actions that are associated with each classification of medication

Ans: C Feedback: After completing a pharmacology course nurses will have general drug information needed for safe and effective medication administration but will need to consult a drug guide for specific drug information before administering any medication. Pharmacology is constantly changing, with new drugs entering the market and new uses for existing drugs identified. Continuing education in pharmacology is essential to safe practice. Nurses tend to become familiar with the medications they administer most often, but there will always be a need to research new drugs and also those the nurse is not familiar with because no nurse knows all medications.

A patient has recently moved from Vermont to Southern Florida. The patient presents to the clinic complaining of dizzy spells and weakness. While conducting the admission assessment, the patient tells the nurse that he have been on the same antihypertensive drug for 6 years and had stable blood pressures and no adverse effects. Since his move, he has been having problems and he feels that the drug is no longer effective. The clinic nurse knows that one possible reason for the change in the effectiveness of the drug could be what? A) The impact of the placebo effect on the patient's response. B) The accumulative effect of the drug if it has been taken for many years. C) The impact of the warmer environment on the patient's physical status. D) Problems with patient compliance with the drug regimen while on vacation.

Ans: C Feedback: Antihypertensive drugs work to decrease the blood pressure. When a patient goes to a climate that is much warmer than usual, blood vessels dilate and the blood pressure falls. If a patient is taking an antihypertensive drug and moves to a warmer climate, there is a chance that the patient's blood pressure will drop too low, resulting in dizziness and feelings of weakness. Even mild dehydration could exacerbate these effects. Most antihypertensives are metabolized and excreted and do not accumulate in the body. Patients must be very compliant with their drug regimen on vacation. After several years on an antihypertensive drug, the effects of that drug are known; therefore, the placebo effect should not be an issue.

A 34-year-old male client has diagnoses of liver failure, ascites, and hepatic encephalopathy secondary to alcohol abuse. The client's family is questioning the care team as to why his abdomen is so large even though he is undernourished and emaciated. Which of the following statements most accurately underlies the explanation that a member of the care team would provide the family? A) An inordinate amount of interstitial fluid is accumulating his abdomen. B) The transcellular component of the intracellular fluid compartment contains far more fluid than normal. C) Normally small transcellular fluid compartment, or third space, is becoming enlarged. D) Gravity-dependent plasma is accumulating in his peritoneal cavity.

Ans: C Feedback: Ascites is characterized by an accumulation of fluid in the transcellular component of the ECF, not ICF. The fluid is not categorized as belonging to the plasma component of the ECF.

In today's health care environment there is often more contact between the patient and the nurse than between the patient and the physician. How does this increased patient contact impact drug therapy? A) Choosing the best medication to treat the patient's condition B) Assessing the patient's preferred communication strategies C) Assessing the therapeutic success of the drug therapy D) Reducing dosage quickly when adverse effects arise

Ans: C Feedback: Because the nurse has the greatest direct and continued contact with the patient, the nurse is in the best position to detect minute changes that ultimately determine the course of drug therapy therapeutic success or discontinuation because of adverse or unacceptable responses. The nurse does not choose the medication or reduce dosage because it is outside the scope of practice of the nurse. Communication strategies used by patients are not related to drug therapy.

The nurse, providing patient teaching about home medication use to an older adult, explains that even when drugs are taken properly they can produce negative or unexpected effects. What are these negative or unexpected effects called? A) Teratogenic effects B) Toxic effects C) Adverse effects D) Therapeutic effects

Ans: C Feedback: Negative or unexpected effects are known as adverse or side effects. Teratogenic effects are adverse effects on the fetus and not a likely concern for an older adult. Toxic effects occur when medication is taken in larger than recommended dosages caused by an increase in serum drug levels. Therapeutic effects are the desired actions for which the medication is prescribed.

The pharmacology instructor questions the students as to which classification of drugs is commonly found in over-the-counter (OTC) combination cold medications? A) Stimulants B) Opioids C) Oral decongestants D) Antitussives

Ans: C Feedback: Oral decongestants are found in many OTC cold and flu preparations so that care must be taken to avoid inadvertent overdose when more than one such drug is used. Opioids are only available by prescription; stimulants and antitussives are not generally found in OTC combination cold medications.

The patient tells the nurse that he or she has begun ordering his or her medications over the Internet because it is cheaper. What statement made by the nurse in response to this information is accurate? A) All drugs are manufactured with the same quality controls. B) Any drug that is shipped into this country is safe to use. C) Foreign drugs may have the same name as domestic drugs, but they are not the same drug. D) If you order from Canada or Mexico, the drugs are safe because they undergo testing.

Ans: C Feedback: The Food and Drug Administration has begun checking these drugs when they arrive in this country and have found many discrepancies between what was ordered and what is in the product, as well as problems in the storage of these products. Some foreign brand names are the same as brand names in this country but are associated with different generic drugs. Options A, B, and D are incorrect because not all drugs are manufactured the same and they are not always safe coming from another country.

A 7-year-old boy fell off a wood pile while playing. He has been admitted to the intensive care unit with multiple broken bones and internal bleeding. What should the nurse know about drug therapy in this type of patient? A) Adverse effects may be decreased. B) Therapeutic effects may be increased. C) Pharmacodynamics may be altered. D) Pharmacokinetics remain the same.

Ans: C Feedback: The child's developmental age will influence pharmacokinetics and pharmacodynamics; the immature liver may not metabolize drugs in the same way and the kidneys may not work as efficiently as those of an adult. Adverse effects might be increased and therapeutic effects may be decreased.

A health educator is performing a health promotion workshop with the staff of a large, urban homeless shelter, and a component of the teaching centers around tuberculosis. One of the staff members comments, "Anyone who's had contact with tuberculosis in the past can give it to any of the other residents of the shelter, even if they didn't get sick themselves." How could the educator best respond to this comment? A) "Many people do manage to fight off the infection, but you're right: they can still spread it by coughing or sneezing." B) "If someone has been previously exposed to tuberculosis, they are particularly infectious because they are often unaware of the disease." C) "Actually, people who have the latent form of the disease won't be sick and can't spread it either." D) "There isn't any real risk of them spreading it, but we would like to vaccinate everyone who's had any contact with it in the past."

Ans: C Feedback: Contact with M. tuberculosis without the development of progressive primary tuberculosis results in a latent infection that is not communicable. Vaccination is not a common intervention in the United States.

When educating a student who lives in a crowded apartment and diagnosed with tuberculosis, the college school nurse will emphasize, A) "Once your fever goes away, you can stop taking the streptomycin injection." B) "If isoniazid makes you nauseous, we can substitute something milder." C) "To destroy this bacterium, you must strictly adhere to a long-term drug regimen." D) "You will have to wear an N95 mask while on campus at all times."

Ans: C Feedback: Success of chemotherapy for prophylaxis and treatment of tuberculosis depends on strict adherence to a lengthy drug regimen that includes isoniazid (INH), rifampin, ethambutol, pyrazinamide, and streptomycin (or some combination of these).

An ECG technician is performing an ECG on a hospital patient who has developed hypokalemia secondary to diuretic use. Which of the following manifestations of the client's health problem will the technician anticipate on the ECG? A) Irregular heart rate and a peaked T wave B) A low T wave and an absent P wave C) A prominent U wave and a flattened T wave D) A narrow QRS complex and an absent U wave

Ans: C Feedback: ECG changes associated with hypokalemia include a prominent U wave and a flattening of the T wave. Atrial fibrillation, a low P wave, and the absence of a U wave are not associated with hypokalemia.

What needs to happen to the protein drug complex for the drugs to reach the cells where the drug can act? A) The protein drug complex must break itself into smaller pieces to enter the capillaries. B) The binding site on the protein picks up a chemical to make it soluble in the serum. C) The drug must break away from the protein-binding site and float freely. D) The drug must be dissolved in the plasma so it can enter the capillaries and then the tissues.

Ans: C Feedback: Most drugs are bound, to some extent, to proteins in the blood to be carried into circulation. The protein drug complex is relatively large and cannot enter into capillaries and then into tissues to react. The drug must be freed from the protein's binding site at the tissues. This occurs without the introduction of another chemical or by dissolving in it plasma.

A 6-year-old child weighing 52 pounds has had a loading dose of aminophylline (Truphylline). The nurse is ready to administer the regular prescribed dose of 4 mg/kg every 4 hours for three doses. How many milligram will the patient receive in 12 hours? A) 44 mg B) 94 mg C) 284 mg D) 344 mg

Ans: C Feedback: First, using the formula: 2.2 pounds and 52 pounds: times kg, determine the child's weight in kg (52/2.2 = 23.64). Next, using the formula of amount of drug prescribed times weight in kg, determine the amount the child will receive in one dose (4 times 23.64 = 94.56). To determine the mg in a 12-hour period multiply 94.56 times 3 = 283.68. Round to 284 mg. Options A, B, and D are not correct.

What would be the nurse's best response if a patient calls the clinic and reports that he has had a persistent cough for 2 weeks and asks the nurse for a recommendation for a cough medicine? A) Look for dextromethorphan as an ingredient in any OTC cough preparation. B) The doctor could order an antihistamine, which might dry up your secretions and stop the cough. C) You should come to the clinic to be evaluated. A cough that lasts that long might be an indication of an underlying medical problem. D) Drink a lot of fluids and take aspirin, which should reduce the irritation in your throat.

Ans: C Feedback: A cough that has persisted for 2 weeks could be indicative of an underlying medical condition that should be addressed before treating the cough. The patient should be asked to come in for an evaluation. Dextromethorphan could help stop the cough, but suppressing the cough might not be in the patient's best interest. Drying the mucosa with an antihistamine could aggravate the cough. Aspirin would not be indicated for relieving throat irritation.

The nurse is caring for a patient who does not have a respiratory disorder but has been prescribed acetylcysteine. What is an additional indication for acetylcysteine (Mucomyst)? A) Conversion of cardiac dysrhythmias B) Treatment of peptic ulcer disease C) Antidote for acetaminophen poisoning D) Decreased bronchospasm

Ans: C Feedback: Acetylcysteine is used orally to protect liver cells from being damaged during episodes of acetaminophen toxicity because it normalizes hepatic glutathione levels and binds with a reactive hepatotoxic metabolite of acetaminophen. Acetylcysteine is not used for the conversion of cardiac dysrhythmias, for treatment of peptic ulcer disease, or for decreasing bronchospasm.

It is important for the nurse to be aware of what related to the way drugs are marketed? A) The adverse effects the advertisements do not mention B) What magazines and Web sites contain the advertisement C) What patients are seeing in the advertisements about these drugs D) The name of the cheerful, happy models who are advertising these drugs

Ans: C Feedback: As the marketing power for prescription drugs continues to grow, the nurse must be constantly aware of what patients are seeing, what the ads are claiming, and the real data behind the indications and contraindications for these hot drugs. The Food and Drug Administration regulates the information that needs to be contained within medication ads. Where the patient saw the ad and the actors in the ads are unimportant.

The nurse is preparing to administer cefadroxil 1 g PO. The medication is supplied in 500-mg tablets. How many tablets will the nurse administer? A) 0.5 tablet B) 1 tablet C) 2 tablets D) 3 tablets

Ans: C Feedback: Convert 1 g to mg by multiplying 1 g times 1,000 mg. There are 500-mg in each tablet. Dividing the 1000 mg prescribed dosage by 500-mg available dosage, the answer is two tablets.

The physician has ordered 30 mg of Demerol IM for relief of a severe migraine headache. The package insert reads meperidine hydrochloride (Demerol) 50 mg/mL. How many milliliters would the nurse administer? A) 1.6 B) 1 C) 0.6 D) 0.5

Ans: C Feedback: Filling in the information from the problem: 50 mg/1 mL = 30 mg/X mL. Cross-multiply yielding 0.6 mL required to administer a 30-mg dosage

A patient with an acute myocardial infarction is admitted to the coronary care unit. The physician has ordered heparin 25,000 units in 250 mL normal saline to infuse at a rate of 600 units/h. The nurse sets the infusion pump to deliver how many milliliters in an hour? A) 8 B) 7 C) 6 D) 5

Ans: C Feedback: First determine the number of units per mL = 25,000 units/250 mL = 100 units/1 mL. Next use the ratio and proportion method to determine the number of milliliters needed to supply 600 units/h. 100 units/1 mL = 600 units/X. Cross-multiplying yields 6 mL needed every hour to supply the required dose.

The nurse is providing an in service on alternative therapies for peers and explains that the term alternative therapies includes what? A) Holistic drug therapy B) Hospice care C) Nondrug measures D) Home care

Ans: C Feedback: Herbal medicines and alternative therapies are found in ancient records and have often been the basis for discovery of an active ingredient that is later developed into a regulated medication. Today, alternative therapies can also include non-drug measures, such as imaging and relaxation. Options A, B, and D are not included in alternative therapies.

The nurse is gathering assessment data from a medication history of a 38-year-old man with four children. What assessment information would be most important in providing care for this patient? A) The medication history of the patient's mother and/or father B) The name of the patient's pharmacy C) Insurance, financial support, and stability for the patient and his family D) The last time the patient was hospitalized

Ans: C Feedback: In this situation, insurance, financial support, and stability would be the most important data and may determine compliance with future drug therapy. The medication history of the patient's parents could indicate a pattern of overall attitude about drug therapy but is not the priority concern. The last time the patient was hospitalized could indicate whether the patient seeks medical care when appropriate or if he self-medicates, contributing to the nurse's knowledge of this individual but this is not the priority concern. The name of the pharmacy would be unnecessary unless the nurse anticipates having to call a prescription in to the pharmacy for the prescriber.

An important concept taught by the nurse when providing medication teaching is the need to provide a complete list of medications taken to health care providers to avoid what? A) Spending large amounts of money on medications B) Allergic reactions to medications C) Drug drug interactions D) Critical concentrations of medications in the body

Ans: C Feedback: It is important that all health care providers have a complete list of the patient's medications to avoid drug drug interactions caused by one provider ordering a medication, unaware of another medication the patient is taking that could interact with the new prescription. Using the same pharmacist for all prescriptions will also help to prevent this from happening. Informing the provider of all medications taken will not reduce costs of medications, which is best accomplished by requesting generic medications. Allergies should be disclosed to all health care providers as well, but this is not why it is important to provide a complete list of medications taken. Critical concentrations are desirable because that is the amount of drug needed to cause a therapeutic effect, or, in other words, to have the effect the drug is prescribed for.

A patient who has chronic bronchial asthma has had a mast cell stabilizer prescribed. What drug would the physician prescribe? A) Ipratropium or budesonide B) Isoetharine or montelukast C) Nedocromil or cromolyn D) Aminophylline or caffeine

Ans: C Feedback: Nedocromil and cromolyn are mast cell stabilizers used in the treatment of asthma. Aminophylline and caffeine are xanthines. Ipratropium is an anti-cholinergic drug and budesonide is a corticosteroid. Isoetharine is a sympathomimetic drug and montelukast is a leukotriene receptor antagonist.

According to Center for Disease Control and Prevention (CDC) recommendations, what is the role of the nurse in preparing for the possibility of bioterrorism? A) Post updated information on signs and symptoms of infections caused by biological agents B) Provide guidelines for treating patients exposed to, or potentially exposed to, biological agents C) Remain current on recognition and treatment of infections caused by biological weapons D) Advocate for increased funding for research involving bioterrorism and patient treatment

Ans: C Feedback: Nurses need to remain current about recognition of and treatment for those exposed to biological weapons because nurses are often called upon to answer questions, reassure the public, offer educational programs, and serve on emergency preparedness committees. The CDC posts updated information on signs and symptoms of infections caused by biological agents that nurses would read. The CDC also provides guidelines for how to treat patients exposed to biological agents and the nurse must remain current on this information. Although nurses could advocate for funding, this is not usually the role of the nurse.

A nurse is caring for a 15-year-old patient with cystic fibrosis who has been prescribed dornase alfa (Pulmozyme) to relieve the buildup of secretions and keep airways open and functioning longer. What will the nurse instruct the patient to do concerning the use of this drug? A) Use in home nebulizer up to four times a day if needed. B) Stop all other medications while using the drug. C) Store the drug in the refrigerator, protected from light. D) Expect a severe headache after each use.

Ans: C Feedback: Patients using dornase alfa should be cautioned to store the drug in the refrigerator, protected from light. Heat and light can cause the drug to break down and can decrease its therapeutic value. The drug should only be used up to two times a day and the patient should continue all other medications while using it. Dornase alfa is only a palliative therapy that improves respiratory symptoms. A severe headache is not associated with this drug.

The nursing instructor is teaching the lab students the best position for the administration of nasal sprays. What position would the instructor teach the students? A) Supine B) Semi-Fowler's C) High Fowler's D) Side-lying

Ans: C Feedback: Teach the patient to sit upright and press a finger over one nostril to close it. This body position is important to prevent excessive amounts of the medication running down the back of the throat. It needs to be in direct contact with the greatest amount of nasal mucosa and the high Fowler's position provides that.

The nurse is reviewing the results of the patient's laboratory tests. What must the nurse keep in mind when reviewing these results related to medication administration? A) The patient's emotional response to the disease process B) The timing of the last dose of medication relative to when blood was drawn C) The possibility of a drug laboratory test interaction D) A change in the body's responses or actions related to the drug

Ans: C Feedback: The body works through a series of chemical reactions. Because of this, administration of a particular drug may alter results of tests that are done on various chemical levels or reactions as part of a diagnostic study. This druglaboratory test interaction is caused by the drug being given and not necessarily by a change in the body's responses or actions. The patient's emotional response or timing of the last dose is not important in drug-laboratory interactions.

The patient has a diagnosis of multiple sclerosis and is taking the drug interferon beta-1a (Rebif). The patient takes this drug by subcutaneous injection three times a week. The dosage is 44 mcg per injection. If the patient takes an injection on Monday, how much of the drug would still be in the patient's system when she takes her next injection on Wednesday, assuming the half-life of the drug is 24 hours? A) 22 mcg B) 16.5 mcg C) 11 mcg D) 5.5 mcg

Ans: C Feedback: The half-life of a drug is the time it takes for the amount of drug in the body to decrease to 1 half the peak level it previously achieved. On Tuesday, there would be 22 mcg remaining in the body, so option A is incorrect. On Wednesday 11 mcg would remain, so option C is the correct answer. At 12 hours before taking the next dose on Wednesday, there would be 16.5 mcg remaining. If the injection were not taken on Wednesday, 12 hours after the dose was due, there would be 5.5 mcg remaining.

The patient drinks 18 ounces of fluid at lunchtime. How many milliliters of intake will the nurse document? A) 1.7 mL B) 0.6 mL C) 540 mL D) 54 mL

Ans: C Feedback: 1 ounce = 30 mL. Using the ratio-and-proportion method: 1 oz/30 mL = 18 oz/X. Cross-multiply to determine the patient drank 540 mL of fluid.

While administering a medication that the nurse has researched and found to have limited effectiveness, the patient tells the nurse, I have read all about this drug and it is such a wonder drug. I'm so lucky my doctor prescribed it because I just know it will treat my problem. The nurse suspects this drug will be more effective than usual for this patient because of what effect? A) Cumulative effect B) First-pass effect C) Placebo effect D) Cross-tolerance effect

Ans: C Feedback: A drug is more likely to be effective if the patient thinks it will work than if the patient believes it will not work. This is called the placebo effect. If a drug is taken in successive doses at intervals that are shorter than recommended, or if the body is unable to eliminate a drug properly, the drug can accumulate in the body, leading to toxic levels and adverse effects. This is a cumulative effect. First- pass effect addresses the reduction of available drug when taken orally due to metabolism in the liver before the drug reaches the bloodstream. Cross-tolerance is resistance to drugs within the same class

A pharmacology student asks the instructor what an accurate description of a drug agonist is. What is the instructor's best response? A) A drug that reacts with a receptor site on a cell preventing a reaction with another chemical on a different receptor site B) A drug that interferes with the enzyme systems that act as catalyst for different chemical reactions C) A drug that interacts directly with receptor sites to cause the same activity that a natural chemical would cause at that site D) A drug that reacts with receptor sites to block normal stimulation, producing no effect

Ans: C Feedback: Agonists are drugs that produce effects similar to those produced by naturally occurring neurotransmitters, hormones, or other substances found in the body. Noncompetitive antagonists are drugs that react with some receptor sites preventing the reaction of another chemical with a different receptor site. Drug enzyme interactions interfere with the enzyme systems that stimulate various chemical reactions.

A 70-year-old patient has just received a drug that can cause sedation. What would be the priority nursing diagnosis for this patient? A) Noncompliance: Cost of the drug B) Deficient knowledge: Unfamiliar with drug therapy C) Risk for injury: Related to adverse effects of the drug D) Ineffective health maintenance: Need for medication

Ans: C Feedback: Because of the patient's age and that the medication causes sedation, the highest priority nursing diagnosis is related to maintaining the patient's safety. Safety for the patient is the nurse's number one concern. There is nothing indicated related to the cost of the drug or the risk of noncompliance for this patient. Deficient knowledge will need to be addressed but it is not the priority when compared with patient safety. There is no indication the patient's need for this medication is related to an ineffective health maintenance issue.

Why does the nurse need to be alert for any indication of an allergic reaction in patients? A) To obtain early warning of noncompliance in drug therapy B) To increase the effectiveness of a specific medication C) To maintain the patient's safety during drug therapy D) To reduce the risk of adverse effects during drug therapy

Ans: C Feedback: Being alert to adverse effects what to assess and how to intervene appropriately can increase the effectiveness of a drug regimen, provide for patient safety, and improve patient compliance. Indications of allergic reactions would not indicate noncompliance or improve effectiveness of a specific medication. Indications of allergic reaction would indicate an adverse effect and would not reduce the risk.

A nurse calculates the pediatric patient's medication dosage using Clark's rule and uses what formula? A) Infant's age in months/150 months times the average adult dose B) Child's age in years/child's age in years plus 12 times the average adult dose C) Weight of child in pounds/150 pounds times the average adult dose D) Surface area in square meters/1.73 times the average adult dose

Ans: C Feedback: Clark's rule uses the child's weight to calculate the dose and assumes the adult dose is based on a 150- pound person. Fried's rule applies to a child younger than 1 year of age and assumes that an adult dose would be appropriate for a child who is 12.5 years (150 months) old. Young's rule applies to children 1 to 12 years of age. Surface area calculation of a child's dose is determined with the use of a nomogram including the child's height and weight.

A patient presents at the emergency department in acute respiratory distress. A quick assessment by the triage nurse indicates that the patient experienced difficulty breathing immediately after taking Combivent for the first time. The nurse suspects that the patient may be allergic to what? A) Aspirin B) Penicillin C) Peanuts D) Ragweed pollen

Ans: C Feedback: Combivent is a combination drug of ipratropium and albuterol. The propellant used to make ipratropium has a cross-sensitivity to the antigen that causes peanut allergies. Aspirin, penicillin, and ragweed pollen are not associated with this drug.

The nurse is caring for a patient with cystic fibrosis who is receiving dornase alfa by nebulizer to help thin secretions. What statement by the patient indicates a need for further instruction? A) This medication will loosen up the sticky mucus that is in my lungs. B) I will try to cough after I receive this medication. C) I'm glad that this medication will solve all of my problems. D) I will continue to use postural drainage and take my enzymes as ordered.

Ans: C Feedback: Cystic fibrosis patients who receive dornase alfa should be cautioned about the need to continue all therapies for their cystic fibrosis because dornase alfa is only a palliative therapy that improves respiratory symptoms, and other therapies, such as coughing, postural drainage and enzymes, are still needed.

When the nurse reads in the drug handbook the section related to recommended dosage, it is important to remember that this suggested dosage is based on what? A) A 40-year-old man B) An average-sized adult C) A 150-pound adult male D) A healthy young adult

Ans: C Feedback: Drug studies base the therapeutic dosage, or that dose needed to reach a critical concentration, on the physiology of a 150-pound healthy adult male. Testing is not routinely done in women because of the potential for unknown effects on the ova. Testing would not be done on an obese adult or older adult because of the potential for underlying disease, altered metabolism, or reactions to the drug. Children and adolescents are never used for testing due to ethical concerns

The patient tells the nurse about a new drug being tested to treat the disease she was diagnosed with and asks the nurse whether the doctor can prescribe a medication still in the preclinical phase of testing. What is the nurse's best response? A) The doctor would have to complete a great deal of paperwork to get approval to prescribe that drug. B) Sometimes pharmaceutical companies are looking for volunteers to test a new drug and the doctor could give them your name. C) Drugs in the preclinical phase of testing are only tested on animals and so would not be available to you. D) Drugs in the preclinical phase of testing are given only to healthy young men and so would not be available to you.

Ans: C Feedback: During the preclinical phase of testing drugs are tested on animals and are not available to patients. In phase I, the drug is tested on volunteers who are usually healthy young men. It is only in phase III studies that the drug is made available to prescribers who agree to closely monitor patients getting the medication.

The patient with diabetes is also taking ephedrine to treat asthma that causes her blood sugar to increase. The patient asks the nurse, Why does this medication make my blood sugar go up? What is the nurse's best response? A) The active ingredient in ephedrine is mixed with sugar. B) Ephedrine is a placebo containing only sugar. C) Stored glycogen is broken down by ephedrine, which is causing higher blood sugar levels. D) Insulin is inactivated by ephedrine so it cannot work to control sugar levels.

Ans: C Feedback: Ephedrine breaks down stored glycogen, which then enters the bloodstream as glucose and causes an increase in serum blood glucose, or blood sugar, levels. Ephedrine is not mixed with sugar and is not a placebo. Ephedrine has no effect on insulin.

The nurse admits a patient to the unit and learns the patient has recently been diagnosed with chronic renal failure but has not informed the primary care provider of this diagnosis. What is the nurse's first priority? A) Administer medications ordered immediately. B) Maintain the patient's confidentiality. C) Call the admitting physician immediately. D) Provide teaching about chronic renal failure.

Ans: C Feedback: Knowledge of the patient's diagnosed renal failure is essential to proper medication ordering because some dosages will need to be decreased whereas other medications may be contraindicated in this patient. The nurse does not breach confidentiality when sharing information that impacts needed care to the primary care provider. Teaching about renal failure may be provided at some point but it is not the priority in this situation. Medications should not be administered until they are appropriately adjusted by a health care provider who is aware of the renal failure diagnosis.

An 80-year-old patient presents at the clinic for a follow-up appointment. She is taking a macrolide antibiotic and is experiencing tinnitus. The nurse is talking with family members about home care for the patient. What should the nurse include in her instructions regarding home care? A) Keep the patient in a prone position when in bed. B) Eliminate salt from the patient's diet. C) Provide protective measures to prevent falling or injury. D) Monitor exposure to sunlight.

Ans: C Feedback: Macrolide antibiotics can cause severe auditory nerve damage, which can cause dizziness, ringing in the ears (tinnitus), and loss of balance and hearing. The patient would be at high risk for injury due to falls. Usually a person who is dizzy is unable to lie flat and needs to recline with the head elevated. Salt and sunlight are not a component of this patient's presenting complaint.

The Kardex record of a male patient who is prescribed antihistamines for treating an allergy reads as follows: Age: 32; Profession: Carpenter; Lifestyle & diet: Lives alone, average smoker, nonalcoholic, no food preferences, practices yoga; Medical history: Suffers from hay fever, recent urinary tract infection that has been treated successfully. What information from the Kardex is likely to have the greatest implication in educating the patient about antihistamine administration? A) The patient's age B) The patient's smoking habit C) The patient's profession D) The patient's medical history

Ans: C Feedback: Most antihistamines cause drowsiness, so the nurse should advise the patient not to operate machinery or perform tasks that require alertness when taking antihistamines (e.g., climbing ladders, working on rooftops, standing on iron supports at the top of a building). Because the patient is not an older adult, his age has no implications on the therapy. Although encouraging the patient to make better lifestyle choices is an important part of the patient's plan of care, this information is not related to administration of antihistamines. There is nothing in the documented medical history that is significant to antihistamine use.

The patient asks the nurse why generic drugs would be used and voices concerns that only the brand name product will be safe. What is the nurse's best response? A) Generic drugs are often less expensive. B) Some quality control problems have been found with generic drugs. C) Most generic drugs are very safe and can be cost effective as well. D) Although initial cost is higher for a brand name it may cost less in the long run.

Ans: C Feedback: Most generic medications are completely safe and may be identical to the brand name drug except generic medications are often less expensive, but this does not address the patient's concern about safety. Although some quality control issues have occurred in the past, this does not address the patient's concerns regarding safety or explain why generic drugs are prescribed and used. Although some doctors believe initial cost is higher but will cost less over time, this response also does not address the patient's concerns.

When assessing a patient before starting a drug regimen, why would the nurse consider it important to assess baseline kidney function? A) To anticipate adverse effects of drugs B) To determine patient's baseline electrolyte levels C) To determine patient's ability to excrete the drug D) To determine patient's ability to metabolize the drug

Ans: C Feedback: Patients with kidney or liver disease require very cautious medication administration, often needing dosages to be decreased and contraindicating some drugs altogether. The patient's renal status will indicate the ability to excrete the drug. Liver function is needed to assess metabolism. Electrolyte levels would be indicated by serum blood test results, not by studies of kidney function. The nurse's goal is to prevent or minimize adverse effects of drugs, not to anticipate them

While studying for the test, the nursing student encounters the following drug: papaverine (Pavabid). What does the nursing student identify the name Pavabid as? A) The generic name B) The chemical name C) The brand name D) The chemical and generic name

Ans: C Feedback: Several clues indicate the brand name including capitalization of the first letter in the name and in parentheses. Generic names are not capitalized; chemical names are descriptions of the chemistry of the medication resulting in complicated names

The nurse is writing a plan of care for a patient newly admitted to the floor with an asthma attack that occurred while exercising. What would be the most appropriate intervention for this patient? A) Assist patients with moderate to severe asthma in obtaining a home nebulizer unit B) Try to prevent or reduce panic, which may initiate bronchospasm C) Teach patient to use an inhaler before exercising D) Monitor peak flow rates, especially in children

Ans: C Feedback: Teach patient who use one of these drugs for exercise-induced asthma to use it 30 to 60 minutes before exercising to ensure peak therapeutic effects when they are needed. The most important intervention would be to use a bronchodilator as prophylaxis for a patient with exercise-induced asthma. It would not be monitoring peak flow rates, trying to prevent or reduce panic, or assisting patient in obtaining a home nebulizer unit.

The nurse is teaching a group of patients with allergic rhinitis about the use of their medications. What would be the most essential information to give these patients about preventing possible drug interactions? A) Over-the-counter (OTC) medications are safe to use. B) Use only one pharmacy so the pharmacist can check drug interactions. C) Read drug labels before taking OTC medications. D) Ask the pharmacy tech for assistance in selecting an OTC medication.

Ans: C Feedback: Teach patients to read the OTC labels to avoid inadvertent overdose. It would be inappropriate to teach the patient to use only one pharmacy for OTC medications. OTC medications are generally safe to use if used correctly. Asking the pharmacy tech for help in selecting an OTC medication is appropriate but not the most essential information to give the patients.

An inhaled sympathomimetic drug has been ordered for a teenage athlete who has exercise-induced asthma. What should the patient be instructed to do? A) Use the inhaler every day at the same time each day. B) Use the inhaler as soon as the symptoms start. C) Use the inhaler 30 to 60 minutes before exercising to ensure peak therapeutic levels when needed. D) Use the inhaler 2 to 3 hours before exercising to ensure peak effectiveness.

Ans: C Feedback: Teaching a patient about using an inhaled sympathomimetic for management of exercise-induced asthma should include instructions to use the inhaler 30 to 60 minutes before exercising to ensure therapeutic levels when needed. The inhaler would not be used daily and waiting until symptoms occur will be too late for prevention. Options B and D are not correct.

The nurse, working on the maternity unit, receives a call from a pregnant woman asking how she can know whether a medication is safe to take while pregnant. What is the nurse's best response? A) You can take any drug indicated as a Category A. B) No medications should be taken during pregnancy. C) Never take medication until you receive approval from your health care provider. D) Most medications are safe but you need to weigh benefit against risk.

Ans: C Feedback: The best response to a pregnant woman asking about medication usage is to talk with her obstetric practitioner because the best advice will come from someone who knows their health and pregnancy history. While Category A drugs have no known risk, they may be contraindicated by the woman's health condition or pregnancy issues and many pregnant women would not know what it means to be a Category A drug. Medications can be helpful during pregnancy if taken safely and appropriately. Although risk benefit needs to be weighed, it should occur with advice from the obstetric practitioner.

A patient has an important presentation to make in 4 hours and he needs relief from the congestion of seasonal rhinitis. The patient calls the nurse, explains the situation, and tells the nurse that he cannot afford to be drowsy. Which antihistamine would be a good choice for this patient? A) Diphenhydramine (Benadryl) B) Dexchlorpheniramine (Polaramine) C) Loratadine (Claritin) D) Hydroxyzine (Atarax)

Ans: C Feedback: The first-generation antihistamines, including diphenhydramine, dexchlorpheniramine, and hydroxyzine, are associated with drowsiness. Loratadine is one of the second-generation antihistamines, which have fewer anticholinergic effects and are less likely to cause drowsiness.

The patient is a 6-year-old child who is taking 125 mg of amoxicillin every 6 hours. Assuming that the half-life of Amoxicillin is 3 hours, how much Amoxicillin would be in the child's body at the time of the next administration of the drug? A) 62.5 mg B) 46.875 mg C) 31.25 mg D) 15.625 mg

Ans: C Feedback: The half-life of a drug is the time it takes for the amount of drug in the body to decrease to 1 half the peak level it previously achieved. Option A would occur at 3 hours after the original dose of amoxicillin. Option B would occur 4 1/2 hours after the original dose. Option C would occur at 6 hours after the original dose. Option D would occur at 7 1/2 hours after the original dose.

The nurse administers amoxicillin 500 mg. The half-life of this drug is approximately 1 hour. At what point would the drug level in the body be 62.5 mg if the drug was not administered again? A) 1 hours after the original dose B) 2 hours after the original dose C) 3 hours after the original dose D) 4 hours after the original dose

Ans: C Feedback: The half-life of a drug is the time it takes for the amount of drug in the body to decrease to one-half of the peak level it previously achieved. At a dose of 500 mg the drug level would be 250 mg in 1 hour, 125 mg in 2 hours, 62.5 mg in 3 hours, and 31.25 mg in 4 hours so the correct answer is 3 hours.

The pharmacology instructor is talking to the nursing students about potassium-sparing diuretics and how they can lead to hyperkalemia, indicated by what assessment finding? A) Urine output of 1,500 mL/24 hours B) Blood pressure of 98/60 C) Potassium level of 5.9 mEq/L D) Calcium level of 11.4 mg/dL

Ans: C Feedback: The normal range of serum potassium for an adult is 3.5 to 5.0 mEq/L. A level higher than 5.0 mEq/L can indicate hyperkalemia. Normal urinary output is between 1,500 and 2,000 cc per day. Urinary output below 1,000 mL per day would include oliguria and would indicate hyperkalemia. A decrease in blood pressure and pulse can indicate hypokalemia. Hyperkalemia refers to an elevated potassium level and not an elevated calcium level.

You are evaluating the discharge teaching you have done with your patient concerning drug therapy. What statement from the patient would indicate that teaching had been effective? A) I have to take three pills each day and I can take them at the time that fits my schedule. B) I should take the white pill in the morning because the doctor wants me to take it. C) I will add the names and dosages of these new drugs to my medication list in my wallet. D) I have prescriptions at different pharmacies. I shop around for the best price for each drug.

Ans: C Feedback: The patient needs to recognize the importance of keeping an updated list of all current medications and the need to share this list with all health care providers to avoid drug-drug interactions. The patient should understand exactly when to take medications, why that medication is being taken, and how to take it safely. Patients should be encouraged to use a single pharmacy because this will add another layer of safety because the pharmacy will know all drugs being prescribed to this patient.

The nurse admits an older adult patient to the emergency room with reports of shortness of breath on exertion and a productive cough. The nurse reviews the patient's current medications and the patient says, I take one pink pill every morning. The nurse asks the name of the drug and the patient says she doesn't know. The patient cannot supply the name of the drug or the purpose in taking it either. This happens with four other medications the patient says she takes. What is an appropriate nursing diagnosis for this patient? A) Acute confusion B) Risk-prone health behavior C) Ineffective health maintenance D) Noncompliance

Ans: C Feedback: This patient is demonstrating ineffective health maintenance because knowledge of what medications are being taken and why they are being taken is essential to her health. Patients should be instructed to carry an up-to-date list of current medications to share with all health care providers. She is taking her medication as ordered according to what she said so that would eliminate risk-prone health behaviors and noncompliance. There is no indication the patient is confused.

A 79-year-old female patient presents at the clinic complaining of constipation for 1 week. The nurse practitioner prescribes Milk of Magnesia 2 teaspoons by mouth as needed for relief of constipation. How many milliliter will the nurse administer? A) 30 mL B) 7.5 mL C) 10 mL D) 15 mL

Ans: C Feedback: Use the ratio and proportion method to convert from household system to metric system. 5 mL/1 tsp = X mL/2 tsp. Cross-multiplying yields 10 mL = 2 tsp.

A patient with diabetic ketoacidosis is to receive a continuous infusion of regular insulin. The physician orders 1 L of 5% dextrose and water solution to run at 150 mL/h once the patient's blood glucose has reached 250 mg/dL. The drip factor of the tubing is 15 gtt/mL. How many drops per minute will the nurse deliver? A) 0.25 gtt/min B) 62 gtt/min C) 37 gtt/min D) 250 gtt/min

Ans: C Feedback: Using the information from this problem: 37.5 can be rounded to 37 or 38 gtt/min.

When involved in phase III drug evaluation studies, what responsibilities would the nurse have? A) Working with animals who are given experimental drugs B) Choosing appropriate patients to be involved in the drug study C) Monitoring and observing patients closely for adverse effects D) Conducting research to determine effectiveness of the drug

Ans: C Feedback: Phase III studies involve use of a drug in a vast clinical population in which patients are asked to record any symptoms they experience while taking the drugs. Nurses may be responsible for helping collect and analyze the information to be shared with the Food and Drug Administration (FDA) but would not conduct research independently because nurses do not prescribe medications. Use of animals in drug testing is done in the preclinical trials. Select patients who are involved in phase II studies to participate in studies where the participants have the disease the drug is intended to treat. These patients are monitored closely for drug action and adverse effects. Phase I studies involve healthy human volunteers who are usually paid for their participation. Nurses may observe for adverse effects and toxicity.

A 73-year-old male patient is being discharged home today. The discharge order reads: Take 10 mL of guaifenesin (Robitussin) PO q4h. This over-the-counter pharmaceutical comes in bottles with plastic measuring caps. How much should the nurse teach the patient to take at home? A) 1/2 tsp B) 1 tsp C) 2 tsp D) 1 tbsp

Ans: C Feedback: The nurse is responsible for teaching the patient the correct drug dosage. You should teach the patient that 5 mL = 1 tsp, so 10 mL (5 × 2) = 2 tsp; therefore Options A, B, and D are incorrect. It is important that patients be taught how to take their medication using measurement systems they are familiar with and know how to use

The nurse is caring for a patient who experienced a severe headache. When the prescribed number of over-the-counter pain relievers did not work the patient said she took double the dosage an hour later. The nurse recognizes this patient is at greatest risk for what? A) An allergic reaction B) Anaphylactic reaction C) Poisoning D) Sedative effects

Ans: C Feedback: This patient has taken an overdosage of the medication. Poisoning occurs when an overdose of a drug damages multiple body systems, leading to the potential for fatal reactions. Allergic and anaphylactic reactions can occur with any drug administration but this is not the patient's greatest risk. More information about the exact type of medication would be needed to determine whether sedative effects are likely.

The physician prescribes 250 mg of a drug. The information on the drug vial says the concentration is 500 mg/mL. How much of the drug will the nurse prepare? A) 0.25 mL B) 0.33 mL C) 0.5 mL D) 0.75 mL

Ans: C Feedback: To determine amount to prepare: 500 mg/1 mL = 250 mg/X. Cross-multiply to determine the nurse will prepare 0.5 mL

During a period of extreme excess fluid volume, a renal dialysis patient may be administered which type of IV solution to shrink the swollen cells by pulling water out of the cell? A) 0.9% sodium chloride B) 5% dextrose and water C) 3% sodium chloride D) Lactated Ringer solution

Ans: C Feedback: When cells are placed in a hypotonic solution, which has a lower effective osmolality than the ICF, they swell as water moves into the cell, and when they are placed in a hypertonic solution, which has a greater effective osmolality than the ICF, they shrink as water is pulled out of the cell.

What aspect of pharmacology does a nurse study? (Select all that apply.) A) Chemical pharmacology B) Molecular pharmacology C) Impact of drugs on the body D) The body's response to a drug E) Adverse and anticipated drug effects

Ans: C, D, E Feedback: Nurses study pharmacology from a pharmacotherapeutic level, which includes the effect of drugs on the body, the body's response to drugs, and both expected and unexpected drug effects. Chemical and molecular pharmacology (Options A and B) are not included in nursing pharmacology courses.

A patient, diagnosed with asthma, has been prescribed tiotropium (Spiriva). What should the nurse teach the patient about this drug? (Select all that apply.) A) It makes you fatigued. B) You need to stay out of direct sunlight. C) It is an anticholinergic. D) You only need to take it once a day. E) It has a rapid onset of action and a long duration.

Ans: C, D, E Feedback: Tiotropium is the first drug approved for once-daily maintenance treatment of bronchospasm associated with chronic obstructive pulmonary disease (COPD). Patients who cannot tolerate the sympathetic effects of the sympathomimetics might respond to the anticholinergic drugs ipratropium (Atrovent) or tiotropium. Tiotropium has a rapid onset of action and a long duration, with a half-life of 5 to 6 days. Tiotropium is not associated with fatigue or photosensitivity.

The nurse would expect to see an adjusted dosage in what patients? (Select all that apply.) A) Young adult women B) Middle-aged men C) Infants D) Neonates E) Older adults

Ans: C, D, E Feedback: Patients at the extremes of the age spectrum children and older adult soften require dosage adjustments based on the functional level of the liver and kidneys and the responsiveness of other organs. The child's age and developmental level will also alert the nurse to possible problems with drug delivery, such as an inability to swallow pills or follow directions related to other delivery methods. The adult, whether male or female, would not require altered dosage unless a secondary condition such as renal or hepatic alterations existed.

A physician is providing care for a child who has a diagnosis of cystic fibrosis (CF). Place the following events in the etiology of CF in ascending chronological order. Use all the options. A) Airway obstruction B) Recurrent pulmonary infections C) Impaired Cl- transport D) Decreased water content of mucociliary blanket E) Increased Na+ absorption

Ans: C, E, D, A, B Feedback: CF is associated with impaired Cl- transport and a consequent increase in Na+ absorption. These result in a lowered water content of the mucociliary blanket making it more viscid. These changes to the mucociliary blanket cause airway obstruction and, ultimately, pulmonary infections.

A 77-year-old lifetime smoker has been diagnosed with a tumor in his lung at the site of an old tubercle scarring site, located in a peripheral area of his bronchiolar tissue. What is this client's most likely diagnosis? A) Squamous cell carcinoma B) Small cell lung cancer C) Large cell carcinoma D) Adenocarcinoma

Ans: D Feedback: Adenocarcinoma is associated with the periphery of the lungs, often at the site of scarring, and can occur in alveolar or bronchiolar tissue. Squamous cell carcinoma, small cell lung cancer, and large cell carcinoma are less commonly associated with these traits.

A 10-year-old boy has a body mass index that places him in the 96th percentile for his age and gender. While educating the parents about obesity, the nurse should emphasize that his weight may predispose him to the development of A) scoliosis. B) respiratory infections. C) gastrointestinal disorders. D) type 2 diabetes.

Ans: D Feedback: Adolescent obesity is associated with an increased risk of type 2 diabetes. He is less likely to face a heightened risk of scoliosis, respiratory infections, or GI disorders.

A hospital patient's arterial blood gases indicate normal levels of oxygen and increased carbon dioxide. The patient's respiratory rate is 12 breaths/minute (normal 14 to 20 breaths/minute) with all other vital signs within normal range. While not evident from assessment and diagnostics, the patient's kidneys are minimizing both H+ excretion and HCO3- reabsorption. What is this client's most likely diagnosis? A) Respiratory alkalosis B) Metabolic acidosis C) Respiratory acidosis\ D) Metabolic alkalosis

Ans: D Feedback: In response to increased bicarbonate, the client is hypoventilating to increase carbon dioxide partial pressure. As well, renal compensation is aimed at lowering pH by both reducing H+ excretion and HCO3- reabsorption. The given data are incongruent with the other major acid-base imbalances.

A woman has recently determined that she is pregnant, and her clinician believes that the conception occurred around 8 weeks prior. Since the embryo is in the third stage of embryonic development, which of the following events and processes in growth and development would be expected to be taking place? A) Transition from a morula to a blastocyst B) Ossification of the skeleton and acceleration of body length growth C) Rapid eye movement and early support of respiration D) Formation of upper limbs and opening of the eyes

Ans: D Feedback: Limb formation and eye opening are associated with the third stage of the embryonic stage of development. The transition from a morula to a blastocyst occurs before the second week of gestation, while ossification of the skeleton and acceleration of body length growth do not take place until the early fetal period. Rapid eye movement and early pulmonary function emerge during the 26th through 29th weeks.

In the neurotrauma unit, a teenager with a closed head injury related to an automobile accident is experiencing high intracranial pressure (ICP). He is intubated and is on a ventilator. One treatment for this is to allow him to progress into which acid-base imbalance in an attempt to lower ICP? A) Metabolic acidosis B) Metabolic alkalosis C) Respiratory acidosis D) Respiratory alkalosis

Ans: D Feedback: Respiratory alkalosis is seen as a treatment with the ventilator with intubated people experiencing high intracranial pressure (ICP) in order to attempt to lower the ICP.

As part of a public health initiative, a nurse is teaching a group of older adults about ways to promote and maintain their health. Recognizing that the common cold is a frequent source of ailment, the nurse is addressing this health problem. Which of the following teaching points about the common cold is most accurate? A) "You shouldn't be taking antibiotics for a cold until your doctor has confirmed exactly which bug is causing your cold." B) "It's important to both cover your mouth when you cough or sneeze and encourage others to do so, since most colds are spread by inhaling the germs." C) "Scientists don't yet know exactly what virus causes the cold, and there is not likely to be a vaccine until this is known." D) "Use caution when choosing over-the-counter drugs for your cold; most people do best with rest and antifever medications."

Ans: D Feedback: The efficacy of over-the-counter cold remedies is minimal, and all have a risk of unwanted side effects; rest and antipyretics are normally sufficient since cold viruses are normally self-limiting. No cold-causing virus will respond to antibiotics, and most colds are spread by the fingers. There is no one specific virus that causes the common cold, and numerous different viruses cause similar symptoms.

The exasperated parents of a 4-month-old infant with colic have asked their health care provider what they can do to alleviate their child's persistent crying. Based on their concerns, the nurse should educate/discuss with the parents which of the following? A) Encouraging them to walk away from the infant when they can no longer tolerate it B) Recommending them to reduce the amount of commercial formula and increase breast-feeding C) Discussing the use of prescribed antiflatulent medication that will help more than changing the formula D) Demonstrating how to use a soothing voice and slow rocking back and forth as a way to calm the infant

Ans: D Feedback: The lack of a single etiologic factor makes treatment of colic difficult. The incidence is similar with both breast-feeding and formula, and while anti flatulents are sometimes used, the problem is not always attributable to intestinal gas. Even though it is a common problem that does resolve with time, parents need support. Nonpharmacologic interventions include soothing voices, singing, swaddling, and slow rhythmic rocking.

Which of the following infants most likely requires medical intervention? A) A 2-day-old baby boy who has caput succedaneum B) An infant 4 hours postpartum who has visible coning of his head following vaginal delivery C) A girl 3 days postpartum with noticeable unilateral cephalhematoma D) A male infant whose vertex delivery resulted in a brachial plexus injury

Ans: D Feedback: While caput succedaneum, cephalhematoma, and head coning are all frequently able to resolve independently, a brachial plexus injury is likely to require treatment and rehabilitation.

The nurse is caring for a patient who is taking an adrenergic bronchodilator. In what disease process should adrenergic bronchodilators be used cautiously? A) Liver failure B) Renal failure C) Respiratory failure D) Heart failure

Ans: D Feedback: Adrenergic drugs cause cardiac stimulation. Patients with liver failure, renal failure, or respiratory failure do not need to use adrenergic bronchodilators cautiously.

The increasing number of patients who go to their health care provider and request a drug they have seen advertised on television or in a magazine has created what continuing challenge to health care providers? A) Treating infections appropriately B) Treating sicker patients C) Prescribing cost-effectively D) Staying knowledgeable about drug therapy

Ans: D Feedback: As the marketing power for prescription drugs continues to grow, the health care provider must be constantly aware of what patients are seeing (or reading), what the commercials and ads are promising, and the real data behind the indications and contraindications for these hot drugs. It is a continuing challenge to stay up-to-date and knowledgeable about drug therapy.

The clinic nurse is caring for a patient who has been prescribed fexofenadine for hay fever. When the nurse is assessing this patient's medication history, what drug would make the nurse question the order for fexofenadine? A) Tetracycline B) Penicillin C) Gentamicin D) Ketoconazole

Ans: D Feedback: Drugdrug interactions vary among antihistamines. For example, anticholinergic effects may be prolonged if diphenhydramine is taken with a monoamine inhibitor and the interaction of fexofenadine with ketoconazole or erythromycin may raise fexofenadine concentrations to toxic levels. This is not a concern with tetracycline, penicillin, or gentamicin.

The nurse is explaining how medications work to a group of peers and explains that disruption of a single step in any enzyme system disrupts what? A) Cell life B) Cell membrane C) Cell receptor sites D) Cell function

Ans: D Feedback: If a single step in one of the many enzyme systems is blocked, normal cell function is disrupted. Cell life and cell membrane may be impacted by disruption of some enzymes but not all enzymes. Receptor sites would not be disrupted by disruption in a single step in the enzyme system.

A premature newborn is being treated for respiratory distress syndrome. The nurse teaches the parents about what adverse effect that can occur with the use of lung surfactants? A) Kidney dysfunction B) Cardiac arrhythmias C) High fever D) Collapsed lung

Ans: D Feedback: Lung surfactants used therapeutically can cause many adverse effects including pneumothorax (collapsed lung), hypotension, pulmonary leak, hyperbilirubinemia, and sepsis. Other adverse effects may occur in the infant related to the degree of immaturity of the child's system and may not be related to the drug therapy. Options A, B, and C are not correct.

A nurse is working as a member of a research team involved in exploring the unique response to drugs each individual displays based on genetic make-up. What is this area of study is called? A) Pharmacotherapeutics B) Pharmacodynamics C) Pharmacoeconomics D) Pharmacogenomics

Ans: D Feedback: Pharmacogenomics is the area of study that includes mapping of the human genome. In the future, medical care and drug regimens may be personally designed based on a patient's unique genetic make- up. Pharmacotherapeutics is the branch of pharmacology that deals with the uses of drugs to treat, prevent, and diagnose disease. Pharmacodynamics involves how a drug affects the body. Pharmacoeconomics includes the costs involved in drug therapy.

The nurse is caring for a patient who is receiving gentamicin, 250 mg and fluconazole (Diflucan), 500 mg at the same time. The nurse knows that if these two drugs competed with each other for protein- binding sites, what would this do? A) Make the patient gentamicin deficient B) Make the patient fluconazole deficient C) Counteract any positive benefit the drugs would have D) Alter the effectiveness of both drugs

Ans: D Feedback: Some drugs compete with each other for protein-binding sites, altering effectiveness or causing toxicity when the two drugs are given together. Nothing in the scenario would indicate that the patient would be either Gentamicin or Diflucan deficient, nor does it indicate that these drugs cannot be given together because they would counteract each other

The nurse is caring for a patient scheduled for surgery this morning who is not to be given anything orally. The nurse reviews the medication administration record and finds the patient has an important medication due but it is supposed to be given orally. What is the nurse's best action? A) Give the medication with a small sip of water. B) Give the medication via a different route. C) Hold the medication and put a note on the front of the chart for the surgeon. D) Call the ordering health care provider and clarify administration.

Ans: D Feedback: The nurse would consult with the ordering provider to determine whether the medication should be held, given by another route, or taken with a sip of water. Administering the medication with a small sip of water could cause the cancellation of the procedure, either because of the sip of water or because the medication may interfere with anesthesia. The nurse cannot change the route of administration without an order. Holding the medication would constitute a drug error because the medication was not given on time.

While rock climbing, a 22-year-old male has endured a severe head injury. Which of the following statements best captures expected clinical manifestations and treatments for his immediate condition? A) Oxygen therapy is likely to decrease his respiratory drive and produce an increase in PCO2. B) Cheyne-Stokes breathing is likely but will respond to bronchodilators. C) The client is unlikely to respond to supplementary oxygen therapy due to impaired diffusion. D) Hypoventilation may exist, resulting in increased PCO2 and hypoxemia that may require mechanical ventilation.

Ans: D Feedback: Brain injuries and accompanying hypoventilation are often associated with increased PCO2 and by hypoxemia that responds to oxygen therapy. Persons with COPD are more vulnerable to diminished respiratory drive secondary to oxygen therapy, while Cheyne-Stokes breathing is not identified as a likely consequence of brain injury. Impaired alveolar diffusion is not an aspect of the client's injury.

Around 3 weeks after razing an old chicken house, a 71-year-old retired farmer has developed a fever, nausea, and vomiting. After ruling out more common health problems, his care provider eventually made a diagnosis of histoplasmosis. Which of the following processes is most likely taking place? A) Toxin production by Histoplasma capsulatum is triggering an immune response. B) Antibody production against the offending fungi is delayed by the patient's age and the virulence of the organism. C) Spore inhalation initiates an autoimmune response that produces the associated symptoms. D) Macrophages are able to remove the offending fungi from the bloodstream but can't destroy them.

Ans: D Feedback: Disseminated histoplasmosis results from the inability of macrophages of the reticuloendothelial system to destroy the fungi. Fungi do not produce toxins, and antibody production and autoimmune responses are not involved in the pathophysiology of this fungal infection.

A 77-year-old female diagnosed with chronic obstructive pulmonary disease (COPD) is experiencing impaired gas exchange and CO2 retention, despite a rapid respiratory rate. Which of the following pathophysiological principles would her health care team expect if her compensatory mechanisms are working? A) Arterial blood gas sampling indicates a pH in the range of 7.45 to 7.55. B) Her kidneys are likely to reabsorb H+ and secrete HCO3- C) Her body will produce excess metabolic CO2. D) Her kidneys will adapt with an increase in plasma HCO3- and her pH will decrease.

Ans: D Feedback: Respiratory acidosis is accompanied by renal adaptation with a more marked increase in plasma HCO3- and a lesser decrease in pH. Her pH is likely below 7.35, and the likely renal response involves the reabsorption of HCO3- and secretion of H+. Excess CO2 production is not a common manifestation of obstructive lung disease.

Which of the following statements best captures the etiology of the acute response phase of extrinsic (atopic) asthma? A) IgG production is heightened as a consequence of exposure to an allergen. B) Airway remodeling results in airflow limitations. C) Epithelial injury and edema occur along with changes in mucociliary function. D) Chemical mediators are released from presensitized mast cells.

Ans: D Feedback: The acute response phase of extrinsic asthma is characterized by the release of chemical mediators from mast cells that have been sensitized. Epithelial injury and edema, as well as airway remodeling, are not associated with the acute phase, and IgE, not IgG, is primarily involved in asthma.

The nurse needs to consider teratogenic effects of medications when caring for what population of patients? A) Older adults B) Patients with a history of cancer C) Children D) Young adult women

Ans: D Feedback: A teratogen is a drug that can harm the fetus or embryo so the nurse would consider the teratogenic properties of medications when caring for woman of child-bearing age including adolescents and young adult women. Teratogens have no impact on older adults or children. Carcinogens are chemicals that cause cancer.

A 22-year-old patient calls the clinic and tells the nurse that she has been depressed and is thinking about taking St. John's wort but wants to know if it is safe first. The nurse begins by questioning what other medications the patient takes and would be concerned about a drug-alternative drug interaction if the patient is also taking what type of medication? A) Antihistamines B) Analgesics C) Antibiotics D) Oral contraceptives

Ans: D Feedback: St. John's wort can interact with oral contraceptives that alter drug metabolism, which can decrease the effectiveness of the contraceptive. Analgesics, antibiotics, and antihistamines can be taken in combination with St. John's wort without known adverse effects.

Which of the following situations would be most deserving of a pediatrician's attention? A) The mother of an infant 2 days postpartum notes that her baby has intermittent periods of hyperventilation followed by slow respirations or even brief periods of apnea. B) A volunteer in the nursery notes that one of the infants, aged 2 weeks, appears unable to breathe through his mouth, even when his nose is congested. C) A neonate is visibly flaring her nostrils on inspiration. D) A midwife notes that a newborn infant's chest is retracting on inspiration and that the child is grunting.

Ans: D Feedback: Retraction and grunting indicate a significant increase in the work of breathing that can be indicative of respiratory distress syndrome, a situation that would require medical intervention. Periods of hyperventilation interspersed with reduced breathing rates are common during the transition to postpartum ventilation, and infants are commonly unable to mouth breathe. Nostril flaring could be a sign of dyspnea, but it can also be a compensatory mechanism that the infant uses to increase oxygen intake; this situation would not be considered as serious as an infant who has chest retractions and grunting.

The nursing instructor is discussing the administration of nasal spray with the nursing students. What information is most important to include in this discussion? A) Finish the bottle of nasal spray to clear the infection effectively. B) Nasal spray can be shared between family members only. C) Administer the nasal spray in a prone position. D) Overuse of nasal spray may cause rebound congestion.

Ans: D Feedback: An adverse effect that accompanies frequent or prolonged use of decongestants is rebound vasodilation, clinically called rhinitis medicamentosa. The reflex reaction to vasoconstriction is a rebound vasodilation, which often leads to prolonged overuse of decongestants. The patient should hold his or her head back for maximum distribution of the spray. Only an individual patient should use the bottle of medication.

A patient has orders to receive 3,000 mL of IV fluid at a rate of 150 mL/h. If the infusion starts at 08:00, when would it be finished? A) 20:00 B) 23:00 C) 01:00 D) 04:00

Ans: D Feedback: Amount of fluid to infuse = 3,000 mL; rate of infusion = 150 mL. 3,000 ÷ 150 mL = 20 hours to infuse. There are 24 hours in a day 20 hours = 4 hours. The infusion will complete in 4 hours before 08:00 (08:00 04:00 = 04:00 hour) so the infusion completes at 04:00.

What disorders would the pharmacology instructor tell the nursing students may be exacerbated by the use of nasal decongestants? A) Pneumonia B) Rheumatoid arthritis C) Acid reflux D) Hypothyroidism

Ans: D Feedback: Assess for possible contraindications or cautions; any history of allergy to the drug or a component of the drug vehicle; glaucoma, hypertension, diabetes, thyroid disease, coronary disease, and prostate problems, all of which could be exacerbated by the sympathomimetic effects. Nasal decongestants do not appear to exacerbate pneumonia, rheumatoid arthritis, or acid reflux.

What statement by a 61-year-old patient who is to take an antitussive with codeine indicates that the nurse's teaching has been effective? A) I will take this medication anytime I start to cough. B) This medication may make me anxious and nervous. C) I should call the physician if I develop nausea, diarrhea, or stomach cramps while taking this medication. D) This medication can cause drowsiness, so I will avoid driving or using power equipment while I take it.

Ans: D Feedback: Codeine is a CNS depressant and should not be combined with driving or heavy machinery activities. Antitussives are not intended to be taken with every coughing episode because the patient may overdose on the medication. The medication usually makes the patient drowsy rather than nervous and anxious. Codeine may cause GI upset, although it is usually constipating; some patients may complain of nausea and stomach distress while taking this medication.

What statement by the patient leads the nurse to believe that he needs additional instruction regarding his nasal decongestant? A) I will blow my nose before instilling the nasal spray. B) I will report any dizziness, drowsiness, or rapid pulse. C) I will drink 2,000 to 3,000 mL of fluid daily. D) I will use it only when I have nasal discharge.

Ans: D Feedback: Decongestants decrease overproduction of secretions by causing local vasoconstriction to the upper respiratory tract (See Table 54.2). This vasoconstriction leads to a shrinking of swollen mucous membranes and tends to open clogged nasal passages, providing relief from the discomfort of a blocked nose and promoting drainage of secretions and improved airflow. The patient must understand proper administration, which includes clearing the nasal passages before inhaling the medication and increasing fluid intake and reporting adverse effects. The medication must be used on a regular basis to be effective. Option B is a distracter.

A patient presents to the emergency department (ED) having an acute asthma attack. An ED physician has ordered a sympathomimetic (epinephrine). The nurse expects what as the therapeutic effect of this drug? A) Decrease the inflammatory response in the airways B) Reduce the surface tension within the alveoli allowing for gas exchange C) Inhibit the release of histamine and slow-reacting substance of anaphylaxis (SRSA) to prevent the allergic asthmatic response D) Cause dilation of the bronchi with increased rate and depth of respiration

Ans: D Feedback: Epinephrine will cause the bronchi to dilate and also cause the rate and depth of respiration to increase. Inhaled steroids decrease the inflammatory response and lung surfactants reduce the surface tension within the alveoli. Mast cell stabilizers inhibit the release of histamine and SRSA to prevent the allergic response. Options A, B, and C are not correct.

During what phase of the nursing process would the nurse be required to consider the efficacy of nursing interventions related to drug therapy? A) Assessment B) Nursing diagnosis C) Interventions D) Evaluation

Ans: D Feedback: Evaluation allows the nurse to review what has changed since intervening to determine whether the nursing care has had a positive, therapeutic effect moving the patient toward a more healthful life. If outcomes have not improved, the nurse begins again at the assessment phase of the nursing process with the goal of changing the plan of care to improve outcomes. The patient's response to the drug and occurrence of adverse drug effects indicate the effectiveness of the nursing interventions related to drug therapy. Assessment involves a systematic, organized collection of data concerning a patient. A nursing diagnosis indicates actual or potential alterations in patient function based on the assessment of the clinical situation. Interventions are actions taken to meet the patient's needs, such as administration of drugs

What does the lipid solubility of the drug influence? A) Absorption of the drug B) Metabolism of the drug C) Excretion of the drug D) Distribution of the drug

Ans: D Feedback: Factors that can affect distribution include the drug's lipid solubility and ionization and the perfusion of the reactive tissue. The lipid solubility of a drug does not influence absorption, metabolism, or excretion

Discharge planning for patients leaving the hospital should include instructions on the use of over-the- counter (OTC) drugs. Which comment by the patient would demonstrate a good understanding of OTC drugs? A) OTC drugs are safe and do not cause adverse effects if taken properly. B) OTC drugs have been around for years and have not been tested by the Food and Drug Administration (FDA). C) OTC drugs are different from any drugs available by prescription and cost less. D) OTC drugs could cause serious harm if not taken according to directions

Ans: D Feedback: It is important to follow package directions because OTCs are medications that can cause serious harm if not taken properly. OTCs are drugs that have been determined to be safe when taken as directed; however, all drugs can produce adverse effects even when taken properly. They may have originally been prescription drugs that were tested by the FDA or they may have been grandfathered in when the FDA laws changed. OTC education should always be included as a part of the hospital discharge instructions

The patient calls the clinic nurse and says, I looked this medication up on the Internet after it was prescribed yesterday and there is nothing in the literature about this drug being used to treat my disorder. Should I still take it? What is the nurse's best response? A) No, stop taking it immediately until I can consult with the doctor because it is obvious a mistake was made. B) Oh, that's okay. Go ahead and take it because the doctor wouldn't order it if he or she didn't think it would be effective. C) It is quite common for drugs to be found to have positive effects for a condition not originally intended so it is safe to take. D) Let me talk with the physician about why this medication was ordered for you and I will call you back.

Ans: D Feedback: Off-label use is relatively common because new information is gathered when the drug is used by large numbers of people that may indicate another condition for which the drug is effective. However, if the nurse does not know for a fact that the drug prescribed is the right drug for the patient's condition, it is always best to consult with the prescriber to make sure the patient is taking the right drug and to avoid a medication error. The medication may be perfectly safe so the patient should not be told the doctor made a mistake.

A patient tells the nurse that a friend has recommended the use of caffeine to treat the patient's asthma. The nurse counsels the patient to begin treatment immediately with the prescribed medication for what reason? A) Caffeine can aggravate the drugs used to treat asthma. B) Most natural products are less toxic or more potent than traditional asthma medications. C) Natural products decrease the adverse effects associated with adrenergic bronchodilators. D) Delays in appropriate treatment can have serious, even fatal, consequences.

Ans: D Feedback: The xanthines, including caffeine and theophylline, come from a variety of naturally occurring sources. These drugs were formerly the main treatment choices for asthma and bronchospasm. However, because they have a relatively narrow margin of safety, and they interact with many other drugs, they are no longer considered the first-choice bronchodilators. Delays in appropriate treatment can have serious, even fatal, consequences. Natural products do not decrease the adverse effects associated with adrenergic bronchodilators. Natural products have not been proven to be less toxic or more potent than prescribed asthma medications. Caffeine does not aggravate drugs used to treat asthma, but it can have an additive effect

A patient visits the clinic and is diagnosed with acute sinusitis. To promote sinus drainage, what medication might be ordered? A) Topical nasal steroid decongestants B) First-generation antihistamines C) Second-generation antihistamines D) Topical decongestants

Ans: D Feedback: Topical decongestants are sympathomimetics, meaning that they imitate the effects of the sympathetic nervous system to cause vasoconstriction, leading to decreased edema and reduced inflammation of the nasal membranes. They are available as nasal sprays that are used to relieve the discomfort of nasal congestion that accompanies the common cold, sinusitis, and allergic rhinitis. Topical nasal steroid decongestants are used for the treatment of allergic rhinitis and to relieve inflammation after the removal of nasal polyps. First- and second-generation antihistamines are not ordered for sinusitis.

An 81-year-old patient with congestive heart failure has been sent to a cardiologist who prescribes digoxin (Lanoxin) 0.125 mg PO every morning. The pharmacy dispenses pills that contain 0.25 mg of Lanoxin. How many pills should the nurse teach the patient to take every morning? A) 2 B) 1.5 C) 1 D) 0.521.

Ans: D Feedback: 0.25 mg/1 tablet = 0.125 mg dose/X. Cross-multiply to determine 0.5 or 1/2 of a tablet is to be taken daily. The nurse may request the pharmacy dispense a different concentration to prevent the patient from having to cut the tablet in half.

The home health nurse is caring for an elderly patient with benign prostatic hypertrophy. An anticholinergic drug has been prescribed. What would be the nurse's priority teaching point for this patient? A) Urinary incontinence may develop. B) Bladder hypertonia may develop. C) An increased dosage may be required. D) Empty the bladder before taking the drug.

Ans: D Feedback: A patient with an enlarged prostate who takes an anticholinergic drug may develop urinary retention or even bladder paralysis when the drug's effects block the urinary sphincters, so anticholinergic drugs are avoided whenever possible. However, if the medication is needed, the patient must be taught to empty the bladder before taking the drug. A reduced dosage also may be required to avoid potentially serious effects on the urinary system but this would not be a teaching point for the patient because the provider will make that decision. Hypotonia, not hypertonia, is more likely to occur. Urinary incontinence is not a likely effect in this case.

Oral antidiabetic drugs can cause alterations in glucose metabolism. Patients who are taking these drugs would need to be observed for what? A) Increased urination B) Deep Kussmaul's respirations C) Thirst and hot or flushed skin D) Confusion and lack of coordination

Ans: D Feedback: Antidiabetic medications decrease blood glucose levels. If levels fall too low, symptoms of hypoglycemia would include confusion and lack of coordination. Elevated blood glucose levels can occur when the patient does not take the medications. With inadequate dosage, hyperglycemia can occur, resulting in increased urination in an attempt to eliminate serum glucose, deep Kussmaul's respirations to reduce blood pH by eliminating carbon dioxide, thirst, and hot or flushed skin

A 27-year-old man is admitted to the emergency department (ED) after a serious motorcycle accident. The patient has a head injury, abrasions covering the left side of his body, a broken left femur, and internal injuries that are not fully assessed at this time. He is transferred from the ED to the intensive care unit (ICU). The nurse who is going to care for this patient in the ICU knows that a priority responsibility in regard to drug therapy is what? A) Support vital functions B) Continue curative treatment C) Institute life-saving treatment D) Monitor patient's response

Ans: D Feedback: Because the nurse has the greatest direct and continued contact with the patient, the nurse is in the best position to detect minute changes that ultimately determine the course of drug therapy therapeutic success or discontinuation because of adverse or unacceptable responses. The nurse would support vital functions, continue curative treatment, and institute life-saving treatment, but these actions occur regardless of drug therapy.

The physician writes an order for oxazepam for a 6-year-old child. The nurse verifies that there is no established dosage for children 6 to 12 years of age for oxazepam. The nurse knows that the usual adult dose is 10 mg tid. What would the nurse calculate the appropriate dose to be? A) 0.03 mg tid B) 0.3 mg tid C) 1.8 mg tid D) 3.3 mg tid

Ans: D Feedback: Because the nurse knows only the child's age, the nurse would need to use Young's rule to determine the appropriate dosage. The formula for Young's rule is: Child's dose = child's age in years ÷ (child's age + 12) × average adult dose. Using the information provided in the question: Dose = 6 ÷ (6 + 12) × 10 mg = 6 ÷ 18 × 10 = 0.33 × 10 = 3.3.

The pharmacology instructor is explaining the difference between bronchodilators and anti-inflammatory drugs. How does an anti-inflammatory drug reduce bronchoconstriction? A) Increases ability to metabolize medication B) Decreases formation of mucus secretions C) Increases reactivity to medication D) By decreasing airway hyperreactivity to various stimuli

Ans: D Feedback: Bronchodilators, or antiasthmatics, are medications used to facilitate respirations by dilating the airways. They are helpful in symptomatic relief or prevention of bronchial asthma and for bronchospasm associated with chronic obstructive pulmonary disease (COPD). Reducing inflammation prevents and reduces bronchoconstriction by decreasing airway hyperreactivity to various stimuli that decreases mucosal edema and formation of mucus secretions that narrow airways. Anti-inflammatory drugs do not increase the ability to metabolize medication or increases reactivity to medication.

The telephone triage nurse receives a call from a patient asking for a prescription for a narcotic to manage his surgical pain. The nurse explains that narcotic prescriptions must be written and cannot be called in to the pharmacy. The patient says, Why are narcotics so difficult to get a prescription for? What is the nurse's best response? A) The Drug Enforcement Agency (DEA) determines the risk for addiction and the Food and Drug Administration (FDA) enforces their control. B) The increase in the number of drug addicts has made the rules stronger. C) The Centers for Disease Control and Prevention (CDC) regulates use of controlled substances to reduce the risk of injury. D) Controlled substances like narcotics are controlled by the FDA and the DEA.

Ans: D Feedback: Controlled substances are controlled by the FDA and the DEA: the DEA enforces control while the FDA determines abuse potential. Regulations related to controlled substances have remained strict and specific and have not been significantly impacted by substance abusers. The CDC is not involved in control of narcotics and other controlled substances

A patient, newly diagnosed with chronic obstructive pulmonary disease (COPD), calls the clinic and asks the nurse to explain what the newly prescribed medications are for. What would be the most appropriate response by the nurse? A) The medications that have been ordered for you are what the physician thinks will help you the most. B) The medications that have been ordered for you are to help you breathe easier. C) The medications that have been ordered for you are designed to work together to help you feel better. D) The medications that have been ordered for you are to help relieve the inflammation and promote dilation of the bronchi.

Ans: D Feedback: Drug treatment of asthma and COPD aims to relieve inflammation and promote bronchial dilation. The other options do not give the patient information about the use of these new medications.

What patient populations would the nurse expect is most likely to be prescribed a drug for an off-label use? A) Adolescent and middle-aged adult patients B) Patients with diabetes or heart disease C) Obstetric and neonatal patients D) Pediatric and geriatric patients

Ans: D Feedback: Drugs being used for an off-label purpose are commonly prescribed for pediatric and geriatric populations due to the lack of drug trial information and minimal premarket testing. Often a trial-and- error method is used in treating both the pediatric and geriatric populations when only adult information is known. The geriatric population responds to medication more like children because of their decreased ability to metabolize medications. Adolescents, especially later adolescents, use medications similarly to young adults as do middle-aged adults. Patients with different diagnoses are often involved in drug testing including those with diabetes and heart disease. Drugs are discouraged for use in obstetric patients

It is important for the nurse to evaluate the efficacy of what parameter when evaluating the drug therapy of a patient? A) Appropriateness of drug dosages B) Compliance C) Caregivers' knowledge level D) Nursing interventions

Ans: D Feedback: During the evaluation phase of care, nurses evaluate how effective care has been in meeting outcome goals. Appropriateness of drug dosages should be determined before administering the medication and not when evaluating their effects. Often, compliance cannot be evaluated until the nurse evaluates the effectiveness of therapy and finds the drug is not performing as expected, at which time the nurse may question the patient about whether medications are being taken as ordered. Caregivers' knowledge level is an assessment providing data that will determine the teaching plan.

The pediatric nurse is caring for a child who weighs 44 pounds. The physician has ordered methylprednisolone sodium succinate (Solu-Medrol), 0.03 mg/kg/d IV in normal saline. How many milligrams of medication will the nurse prepare? A) 6.5 B) 6 C) 0.65 D) 0.6

Ans: D Feedback: First convert the child's weight to kilograms by dividing 44 pounds by 2.2 kg/1 pound = 20 kg. Multiply the dosage times the child's weight: 20 kg × 0.03 mg/kg/d = 0.6 mg/d

You are volunteering in the medical tent of a road race on a hot, humid day. A runner who has collapsed on the road is brought in with the following symptoms: sunken eyes, a body temperature of 100°F, and a complaint of dizziness while sitting to have his blood pressure taken (which subsides upon his lying down). These are signs of a fluid volume deficit. Which of the following treatments should be carried out first? A) Offer water by mouth. B) Begin cooling of his body by ice packs. C) Give him a transfusion of FFP. D) Give him an electrolyte solution by mouth.

Ans: D Feedback: Fluid volume deficit results in postural hypotension (dizziness while upright) due to decreased blood volume. Sunken eyes and elevated temperature also point to a fluid volume deficit. The most important action to take is to replace fluid; however, pure water would be a mistake, since without accompanying electrolytes such as sodium, hyponatremia (water retention and a decrease in serum osmolality) could result. Thus, an oral electrolyte solution is recommended; in more severe cases, an IV would be appropriate.

A nurse is assessing the patient's home medication use. After listening to the patient list current medications, the nurse asks what priority question? A) Do you take any generic medications? B) Are any of these medications orphan drugs? C) Are these medications safe to take during pregnancy? D) Do you take any over-the-counter medications?

Ans: D Feedback: It is important for the nurse to specifically question use of over-the-counter medications because patients may not consider them important. The patient is unlikely to know the meaning of orphan drugs unless they too are health care providers. Safety during pregnancy, use of a generic medication, or classification of orphan drugs are things the patient would be unable to answer but could be found in reference books if the nurse wishes to research them.

The nurse is providing health teaching to a newly diagnosed asthmatic patient. The patient has been prescribed theophylline. What is contraindicated with the use of this drug? A) Using insulin B) Taking anti-inflammatory drugs C) Exercising D) Smoking cigarettes

Ans: D Feedback: Nicotine increases the metabolism of xanthines in the liver so that xanthine dosage must be increased in patients who continue to smoke while using xanthines. In addition, extreme caution must be used if the patient decides to decrease or discontinue smoking because severe xanthine toxicity can occur. Bronchoconstriction is not caused by using insulin and anti-inflammatory drugs do not cause bronchoconstriction. Exercise with a physician's supervision is encouraged in individuals with asthma.

A 70-year-old man is being treated for chronic obstructive pulmonary disease (COPD) with theophylline (Theo-Dur). What will be a priority assessment by the nurse? A) Ingestion of fatty foods B) Weight C) Activity level D) Use of nicotine

Ans: D Feedback: Nutritional status, weight, and activity level would be important for a nurse to know about a COPD patient. However, it would be most important for the nurse to know whether the patient smokes or uses tobacco in other ways or smoking cessation methods that involve nicotine. Nicotine increases the metabolism of theophyllines; the dosage may need to be increased to produce a therapeutic effect.

A patient tells the clinic nurse that he or she has been taking over-the-counter (OTC) Pepcid to relieve acid indigestion for several years. This is the first time the patient has ever reported this issue to a health care provider. As part of the teaching plan for this patient, the nurse explains what risk associated with not sharing OTC drug use with the provider? A) The OTC drug could be more expensive than seeking health care advice. B) The drug could mask symptoms of a serious problem that is undiagnosed. C) Use of the drug could cause a rebound effect of Pepcid. D) The drug could interact with several cold medicines.

Ans: D Feedback: OTC drugs allow patients to self-diagnose and treat routine signs and symptoms without seeing a health care provider. This self-prescribed treatment, however, could mask a more serious underlying medical problem and result in a poor outcome for the patient. The issues of drug rebound and drug interaction need to be considered, but the safety issue related to self-diagnosis and self-prescription presents the greatest risk to the patient. Patients should always be encouraged to discuss the use of OTC products with their health care provider.

The nurse is caring for a patient who takes several drugs. What patient would the nurse monitor most closely because of an increased risk for adverse effects of medications? A) The school-aged child B) The obese middle-aged man C) The adolescent D) The newborn infant

Ans: D Feedback: Patients most likely to have adverse drug reactions include the very young or very old due to physiologic characteristics peculiar to these age groups. Therefore, the newborn infant would be at greater risk than the school-aged child who is at greater risk than the adolescent or middle-aged man.

The nurse is writing a plan of care for a patient who is exhibiting Parkinson-like syndrome. What would be an appropriate intervention if, on assessing the patient, the nurse finds the patient is having difficulty swallowing? A) Thicken all liquids. B) Keep the patient NPO (not taking anything orally). C) Give only soft or pureed foods. D) Provide small, frequent meals.

Ans: D Feedback: Provide small, frequent meals if swallowing becomes difficult. Keeping the patient NPO would be inappropriate because these effects often result from medications that will be taken throughout the patient's life. Soft or pureed foods are often more difficult to swallow than more rigid foods. Thickening liquids would only be necessary if the dysphagia, or difficulty swallowing, continued to progress

The nurse administers erythromycin, a drug that is known to irritate mucosa in the stomach lining. When the patient reports abdominal discomfort after taking the medication, the nurse would classify this discomfort as what type of adverse effect? A) Primary action B) Secondary action C) Hypersensitivity reaction D) Allergic reaction

Ans: D Feedback: Secondary actions are those actions that occur as a result of taking a medication but do not fall under the category of therapeutic action and are often negative. This patient is experiencing a secondary action of erythromycin. Primary actions would be extensions of therapeutic action. Hypersensitivity reaction would be an excessive response to either the primary or secondary effects of a drug. An allergic reaction would be an immune response to the drug.

A 77-year-old man is brought to the clinic by his daughter for a routine follow-up appointment. The daughter tells the nurse that her father is only taking half the prescribed dosage of several of his medications. What effect would the nurse explain could result from this behavior? A) Increased risk of primary actions B) Dermatologic reaction C) Superinfection D) Reduced therapeutic effect

Ans: D Feedback: Taking too little of the medication would mean that therapeutic levels are not being reached and the drugs will be less effective at lower dosages. Primary actions are the result of overdose, which is not the case in this patient who is taking too little of the drug. Dermatologic reactions are not likely if the patient is taking too little of the drug unless the drug is treating a dermatologic problem, which is not indicated by the question. Superinfection would only result if the patient was taking an antibiotic, which is not indicated by the question.

The patient calls the clinic and talks to the nurse saying, I found the same drug the provider prescribed on the Internet and it is much cheaper. Is it safe for me to order my drug from this site? What is the nurse's best response? A) It is usually safe to order drugs from Internet Web sites if it is a reliable site. B) Most drugs ordered online come from another country and are safely used there. C) The drug you get will be the same chemical prescribed but the dosage may differ. D) The Food and Drug Administration (FDA) has issued warnings to consumers about the risk of taking unregulated drugs.

Ans: D Feedback: The FDA has begun checking these drugs when they arrive in this country and have found many discrepancies between what was ordered and what is in the product, as well as problems in the storage of these products. Some foreign brand names are the same as brand names in this country but are associated with different generic drugs. The FDA has issued many warnings to consumers about the risk of taking some of these drugs without medical supervision, reminding consumers that they are not protected by U.S. laws or regulations when they purchase drugs from other countries.

The clinic nurse is talking with a patient about information concerning a drug her or she bought online. What is the nurse's responsibility to the patient concerning this information? A) Encourage the patient to seek information about drugs from a pharmacist. B) Explain that information obtained from the Internet is not always accurate. C) Offer the patient a drug reference guide to read and learn more about the drug. D) Interpret the information and explain it in terms that the patient will understand.

Ans: D Feedback: The Internet can be a good reference for drug information. However, the amount and reliability of the information can be overwhelming. The nurse should always try to interpret the information and explain it in terms that the patient will understand. A pharmacist is a good resource person but may not be able to teach from a holistic perspective. Drug reference guides may be hard for the patient to understand and he or she would still need someone to interpret the information

A nurse is caring for a patient who has been receiving a drug by the intramuscular route but will receive the drug orally after discharge. How does the nurse explain the increased dosage prescribed for the oral dose? A) Passive diffusion B) Active transport C) Glomerular filtration D) First-pass effect

Ans: D Feedback: The first-pass effect involves drugs that are absorbed from the small intestine directly into the portal venous system, which delivers the drug molecules to the liver. After reaching the liver, enzymes break the drug into metabolites, which may become active or may be deactivated and readily excreted from the body. A large percentage of the oral dose is usually destroyed and never reaches tissues. Oral dosages account for the phenomenon to ensure an appropriate amount of the drug in the body to produce a therapeutic action. Passive diffusion is the major process through which drugs are absorbed into the body. Active transport is a process that uses energy to actively move a molecule across a cell membrane and is often involved in drug excretion in the kidney. Glomerular filtration is the passage of water and water-soluble components from the plasma into the renal tubule.

A drug with a half-life of 4 hours is administered at a dosage of 100 mg. How much of the drug will be in the patient's system 8 hours after administration? A) 75 mg B) 50 mg C) 37.5 mg D) 25 mg

Ans: D Feedback: The half-life of a drug is the time it takes for the amount of drug in the body to decrease to 1 half the peak level it previously achieved. Option A would occur 2 hours after administration of the drug. Option B would occur at 4 hours. Option C would occur at 6 hours. Option D would occur at 8 hours after the original administration of the drug.

How can the nurse find the most up-to-date information about emergency preparedness related to bioterrorism agents? A) Read textbooks devoted to the topic. B) Ask coworkers to explain current events. C) Read journal articles about bioterrorism agents. D) Visit the Centers for Disease Control and Prevention (CDC) Web site.

Ans: D Feedback: The most current information will be found on the CDC Web site because new information can be posted immediately whereas textbooks and journal articles take time to print. Coworkers may or may not remain current on emergency preparedness and should not be the primary source of information.

A patient with chronic obstructive pulmonary disease (COPD) presents at the emergency department in acute respiratory distress. The patient's family tells the nurse that the patient's problems began right after the patient took his or her first dose of ipratropium (Atrovent). What would the nurse suspect is the problem? A) An allergy to milk B) Overexertion by the patient C) Patient not taking the medications correctly D) An allergy to soy products

Ans: D Feedback: The use of ipratropium or tiotropium is contraindicated in the presence of known allergy to the drug or to soy products or peanuts (the vehicle used to make ipratropium an aerosol contains a protein associated with peanut allergies) to prevent hypersensitivity reactions. An allergy to milk is not associated with sensitivity to ipratropium. Overexertion would not cause the patient to develop respiratory distress after using ipratropium for the first time. Misuse of the inhaler would not cause respiratory distress.

A 26-year-old male patient with a diagnosis of schizophrenia has been admitted with suspected hyponatremia after consuming copious quantities of tap water. Given this diagnosis, what clinical manifestations and lab results should the nurse anticipate the patient will exhibit? A) High urine specific gravity, tachycardia, and a weak, thready pulse B) Low blood pressure, dry mouth, and increased urine osmolality C) Increased hematocrit and blood urea nitrogen and seizures D) Muscle weakness, lethargy, and headaches.

Ans: D Feedback: Weakness, lethargy, and nausea are noted manifestations of hyponatremia. High urine specific gravity, tachycardia, and a weak, thread pulse are associated with hypernatremia, while low blood pressure, fever, and increased urine osmolality are manifestations of fluid volume deficit. Increased hematocrit and blood urea nitrogen and seizures are also associated with hypernatremia.

The nurse is assessing a diabetic patient who has presented at the clinic reporting several hypoglycemic episodes during the past 3 weeks. The nurse questions the patient about the use of herbal or alternative therapies, suspecting what herbal remedy could cause the hypoglycemic episodes? A) St. John's wort B) Kava C) Fish oil D) Ginseng

Ans: D Feedback: Ginseng is known to decrease blood sugar levels. If the patient used this in combination with his or her oral antidiabetic agent, diet, and exercise, his or her blood sugar could drop below therapeutic levels. St. John's wort interacts with many drugs, but not with antidiabetic agents. Kava is associated with liver toxicity. Fish oil has been associated with decreased coronary artery disease.

A 32-year-old woman is admitted to the unit with a diagnosis of hypovolemia. The nurse is developing a care plan for this patient. What is an appropriate nursing diagnosis to help prevent medication errors? A) Dysfunctional gastrointestinal motility B) Ineffective self-health maintenance C) Risk for injury D) Deficient fluid volume

Ans: D Feedback: Hypovolemia is condition involving fluid volume in the body that is less than required. This would affect drug therapy for this patient and would be an important inclusion in the plan of care. The situation described does not indicate dysfunctional GI motility or ineffective self-health maintenance. A patient who is severely fluid volume deficient might be at risk for falls and injury but more information would be needed to make this determination.

A patient with Parkinson's disease is taking an anticholinergic drug to decrease the tremors and drooling caused by the disease process. The patient complains that he is having trouble voiding. The nurse would explain that this is what? A) A hypersensitive action of the drug B) A primary action of the drug C) An allergic action of the drug D) A secondary action of the drug

Ans: D Feedback: Sometimes the drug dosage can be adjusted so that the desired effect is achieved without producing undesired secondary reactions. But sometimes this is not possible, and the adverse effects are almost inevitable. In such cases, the patient needs to be informed that these effects may occur and counseled about ways to cope with the undesired effects. The situation described is not a hypersensitivity reaction that would indicate an allergic reaction, a primary reaction that would be excessive therapeutic response, or an allergic reaction to the drug.

The nurse is providing teaching to a student nurse about how antidiuretic hormone (ADH) plays a central role in the reabsorption of water by the kidneys. The nursing student is correct to place the following components of the homeostatic action of ADH in the correct sequence. Use all the options. A) Stored ADH is released into circulation. B) ADH is transported along a neural pathway to the posterior pituitary gland. C) Aquaporins are inserted into tubular cell membranes. D) ADH is synthesized by cells in the supraoptic and paraventricular nuclei of the hypothalamus. E) Serum osmolality increases.

Ans: D, B, E, A, C Feedback: ADH is produced in the hypothalamus, sequestered in the pituitary, and is released in response to increased serum osmolality. Its influence on tubular cells is exerted by way of the insertion of aquaporins in the tubular membrane.


Kaugnay na mga set ng pag-aaral

XCEL Chapter 3 Life Insurance Part 2 Exam

View Set

Organizational Behavior: Chapter 3

View Set

Small Business Management Exam 2 - Agheay UCO

View Set

Chemistry H Midterm Exam Practice

View Set

Pratice quiz- Second Proctor exam

View Set

Chapter 10: Depreciation, Impairments, and Depletion

View Set

Chapter 3 Quiz - Technology for Success (Hardware)

View Set

Section 4.2 Spanning Tree Overview

View Set